HyGuru

New Free 120 NBME Step 2 CK Explanations [August 2023]

Picture of Rahul Damania
By Rahul Damania
, MD — Updated August 18, 2023

Key Takeaway

The USMLE updated the Free 120 Step 2 CK questions on July 10, 2023. Below are explanations for every question in Blocks 1-3 (with references).

If you’re studying for USMLE Step 2 CK, check out my Rapid Review Course. I have created active-recall, vignette-based videos that will augment your content review and test taking strategy for Shelf & USMLE Step 2 CK.

Block 1

Question 1

A 6-year-old boy is brought to the emergency department by his mother because of the acute onset 1 hour ago of severe right-sided lumbar pain and three episodes of emesis….

Answer: B. CT Scan of Abdomen Pelvis

  1. Why the correct answer is right:
    • In acute lumbar pain, vomiting, hematuria, and unilateral hydronephrosis with hydroureter, the child likely has a ureteral stone.
    • For USMLE/NBME, recognize that CT of the abdomen and pelvis is the gold standard for detecting and localizing ureteral stones.
  2. Illness script/Test-taking strategy:
    • Ureteral stones can present with colicky pain and hematuria, often showing radiographic evidence of hydronephrosis.
    • Associate sudden flank pain with hematuria to ureteral stones.
  3. Why other answers are wrong:
    • A. Captopril renography: For renal artery stenosis.
    • C. MAG-3 renal scan: Differentiates obstructive vs. non-obstructive dilatation.
    • D-E. Radionuclide cystography & Retrograde pyelography: Evaluates vesicoureteral reflux and urinary tract, respectively, but not ideal for stone detection.
    • F. Ultrasonography of the spinal column: For spinal dysraphism.
    • G. Voiding cystourethrography: Assesses vesicoureteral reflux.

Test-taking tip: On USMLE, the clinical scenario’s specifics guide the test choice. Here, pain + hematuria + hydronephrosis screams “stone,” and CT is the top for stone detection.

Reference:

  • Le, T., Bhushan, V., & Sochat, M. (2023). First Aid for the USMLE Step 1 (p. 67, 207). McGraw-Hill Education.
  • Le, T., Bhushan, V., Ascha, M., Bhardwaj, A., Boushra, M., & Griffin, D. (2023). First Aid for the USMLE Step 2 CK, eleventh edition (11th ed.). (p. 227, 694 – 697). McGraw-Hill Education.

Question 2

An 82-year-old woman comes to the office because of a 1-month history of increasing numbness in her feet. She has no history of serious illness…..

Answer: F. Vitamin B12 (cyanocobalamin) supplementation

  1. Why the correct answer is right:
    • The patient’s symptoms—numbness, absent deep tendon reflexes, decreased vibration, and proprioception—indicate peripheral neuropathy, and given the prolonged intake of calcium carbonate, vitamin B12 malabsorption is suggested.
    • For USMLE/NBME, calcium can bind to vitamin B12 in the stomach, leading to decreased absorption and deficiency.
  2. Illness script/Test-taking strategy:
    • Peripheral neuropathy with sensory deficits combined with a history of prolonged calcium carbonate intake should raise suspicion for vitamin B12 deficiency.
    • Always assess for dietary and medication histories which may influence vitamin B12 levels.
  3. Why other answers are wrong:
    • A. Alcohol cessation: Chronic alcohol use can cause neuropathy and cerebellar, and gait abnormalities.
    • B. Calcium carbonate cessation: May reduce B12 depletion but will not replace already depleted stores.
    • C. Folic acid supplementation: Deficiency can cause megaloblastic anemia but not neuropathy.
    • D. Niacin supplementation: Niacin deficiency causes pellagra presenting with diarrhea, dermatitis, and dementia.
    • E. Vitamin B (thiamine) supplementation: Deficiency leads to Wernicke-Korsakoff syndrome or beriberi.

Test-taking tip: For neurologic findings combined with a specific dietary or medication history, think about the potential nutritional deficiencies. In this case, peripheral neuropathy with calcium carbonate use leads to B12 deficiency.

Reference:

  • Le, T., Bhushan, V., & Sochat, M. (2023). First Aid for the USMLE Step 1 (p. 66, 67, 157, 158). McGraw-Hill Education.
  • Le, T., Bhushan, V., Ascha, M., Bhardwaj, A., Boushra, M., & Griffin, D. (2023). First Aid for the USMLE Step 2 CK, eleventh edition (11th ed.). (p. 293 – 294). McGraw-Hill Education.

Question 3

A 2-year-old girl is brought to the emergency department because of loud breathing, harsh cough, and a hoarse cry since she awoke 1 hour ago. Her parents state that yesterday she had a runny nose. Medical history is unremarkable and she receives no medications…..

Answer: D. Nebulized epinephrine

  1. Why the correct answer is right:
    • The clinical presentation of the 2-year-old girl, characterized by loud breathing, harsh cough, hoarse cry, runny nose, fever, and inspiratory stridor, strongly suggests acute viral croup. Croup is a common pediatric respiratory illness that primarily affects the larynx and trachea, often caused by parainfluenza virus. The key features of croup include a “barking” cough, hoarseness, and inspiratory stridor, especially in the context of a recent upper respiratory tract infection.
    • Nebulized epinephrine is the most appropriate initial pharmacotherapy for moderate to severe croup, which is indicated by the presence of significant stridor at rest and the use of accessory muscles for breathing (evidenced by suprasternal and subcostal retractions). Nebulized epinephrine works by causing vasoconstriction and reducing upper airway edema, thereby relieving the airway obstruction and improving breathing.
  2. Illness script/Test-taking strategy:
    • In cases of pediatric respiratory distress with a clinical picture consistent with croup, it’s crucial to assess the severity based on symptoms like stridor, retractions, and general respiratory status. The choice of therapy hinges on the severity of these symptoms, with nebulized epinephrine being a first-line treatment for more severe cases.
  3. Why other answers are wrong:
    • A. Intravenous dexamethasone: While dexamethasone is used in the treatment of croup, it’s generally not the first-line for immediate relief of severe symptoms.
    • B. Nebulized albuterol: More appropriate for bronchospasm associated with asthma or bronchiolitis, not croup.
    • C. Nebulized budesonide: A corticosteroid used for long-term management of asthma, not acute relief of croup symptoms.
    • E. Oral albuterol: Typically not utilized clinically. 
    • F.Oral prednisone: While oral steroids are used in croup, they do not provide the immediate relief of severe symptoms like nebulized epinephrine.
    • G.Subcutaneous epinephrine: Used in refractory status asthmaticus, not croup.

Test-taking tip: When tackling clinical vignettes, especially in pediatrics, it’s important to closely evaluate the presenting symptoms and their severity. Stridor typically is an upper airway, inspiratory noise, wheezing is a lower airway, expiratory noise.

Reference:

  • Le, T., Bhushan, V., & Sochat, M. (2023). First Aid for the USMLE Step 1 (p. 67, 592). McGraw-Hill Education.
  • Le, T., Bhushan, V., Ascha, M., Bhardwaj, A., Boushra, M., & Griffin, D. (2023). First Aid for the USMLE Step 2 CK, eleventh edition (11th ed.). (p. 611 – 614). McGraw-Hill Education.

Question 4

A 67-year-old man is evaluated in the intensive care unit. He has end-stage pancreatic cancer and was hospitalized three days ago for treatment of pneumonia….

Answer: D Provide palliative therapy only

  1. Why the correct answer is right:
    • The patient is at the end-stage of a terminal illness and presents with acute respiratory failure.
    • He did not formalize his wishes in an advanced directive but verbally expressed his desire to avoid mechanical ventilation to his wife.
    • The USMLE/NBME emphasizes the importance of prioritizing the patient’s expressed wishes, even verbal, over other considerations when no legal document is present.
  2. Illness script/Test-taking strategy:
    • When presented with an ethical scenario on the USMLE, always prioritize the patient’s known wishes.
    • Remember that verbalizing wishes to a close family member often carries significant weight, especially without a formalized document.
  3. Why other answers are wrong:
    • A. Perform endotracheal intubation and begin mechanical ventilation: This goes against the patient’s wishes.
    • B. Perform endotracheal intubation and then consult the hospital ethics committee: Consulting ethics is valid in many cases, but the patient’s wishes are clear here.
    • C. Perform endotracheal intubation only: This goes against the patient’s wishes.
    • E. Seek a court order to assign a legal guardian: A prolonged process, and the situation requires immediate decision-making.

Test-taking tip: The USMLE often tests ethical scenarios where the best decision respects the autonomy and wishes of the patient. It is important to differentiate between expressed patient wishes and surrogate or family preferences.

Reference:

  • Le, T., Bhushan, V., Ascha, M., Bhardwaj, A., Boushra, M., & Griffin, D. (2023). First Aid for the USMLE Step 2 CK, eleventh edition (11th ed.). (p. 637 – 642). McGraw-Hill Education.

Question 5

A 6-year-old boy is brought to the office by his parents as a new patient. Three months ago, he and his 3-year-old sister and 4-year-old brother were adopted from a Russian orphanage …………… is unable to read, even in Russian. Results of receptive language testing are consistent with an average IQ……..

Answer: D Learning disorder

  1. Why the correct answer is right:
    • The child shows specific difficulties in reading (despite average IQ and age-appropriate daily living skills), characteristic of a learning disorder.
    • The USMLE/NBME expects students to recognize that learning disorders can manifest as specific academic challenges even when a general intellectual ability is within normal limits.
  2. Illness script/Test-taking strategy:
    • A history of institutionalization can lead to various developmental issues.
    • When there is a specific deficit like reading difficulty in the presence of normal intellectual functioning and appropriate daily living skills, think of learning disorders.
  3. Why other answers are wrong:
    • A. Autism spectrum disorder: Involve persistent deficits in social communication and interaction.
    • B. Fetal alcohol syndrome: Involves characteristic facial features, growth problems, and CNS abnormalities
    • C. Intellectual developmental disorder: The child has an average IQ, making this diagnosis less likely.
    • E. Post-traumatic stress disorder: Trauma might be suspected, but symptoms do not highlight re-experiencing, avoidance, or increased arousal.
    • F. Reactive attachment disorder: Involves inhibited, emotionally withdrawn behavior toward adult caregivers.

Test-taking tip: Focus on the primary deficits presented in the vignette. The main issue was the reading difficulty in the setting of average IQ and age-appropriate skills, directing towards a learning disorder.

Reference:

  • Le, T., Bhushan, V., Ascha, M., Bhardwaj, A., Boushra, M., & Griffin, D. (2023). First Aid for the USMLE Step 2 CK, eleventh edition (11th ed.). (p. 588, 513t). McGraw-Hill Education.

Question 6

A 77-year-old woman residing in a skilled nursing care facility is examined by the physician one day after staff noticed blood on her bedsheets. She sustained a cerebral infarction eight years ago……

Answer: F. Biopsy of the vulva

  1. Why the correct answer is right:
    • The presentation of a raised, fleshy lesion on the vulva in an elderly woman is concerning for vulvar cancer, and the most definitive way to diagnose it is through histological examination.
    • The USMLE/NBME expects students to prioritize biopsy for suspicious lesions to establish a diagnosis.
  2. Illness script/Test-taking strategy:
    • The presentation of a raised, fleshy lesion on the vulva in an elderly woman is concerning for vulvar cancer, and the most definitive way to diagnose it is through histological examination.
    • The USMLE/NBME expects students to prioritize biopsy for suspicious lesions to establish a diagnosis.
  3. Why other answers are wrong:
    • A. Cytologic evaluation of the vulva: Not the primary method for diagnosing vulvar cancer.
    • B. CT scan of the abdomen and pelvis: Imaging is not the initial step.
    • C. Application of an antifungal cream: No evidence of a fungal infection.
    • D. Application of a corticosteroid cream: No evidence of inflammation.
    • E. Colonoscopy: Not relevant.

Test-taking tip: Prioritize histological diagnosis when dealing with suspicious lesions, as it provides the most accurate information regarding the nature of the lesion and definitive diagnosis.

Reference:

  • Le, T., Bhushan, V., & Sochat, M. (2023). First Aid for the USMLE Step 1 (p. 663). McGraw-Hill Education.
  • Le, T., Bhushan, V., Ascha, M., Bhardwaj, A., Boushra, M., & Griffin, D. (2023). First Aid for the USMLE Step 2 CK, eleventh edition (11th ed.). (p. 502 – 503). McGraw-Hill Education.

Question 7

A 51-year-old woman comes to the office for a health maintenance examination…….
On examination, pedal pulses are absent bilaterally.

Answer: C Serum lipid studies

  1. Why the correct answer is right:
    • The absent pedal pulses and the presence of a femoral bruit indicate peripheral arterial disease (PAD).
    • One of the most modifiable risk factors for PAD is dyslipidemia. As such, assessing and managing lipid levels in these patients is important.
    • The USMLE/NBME expects students to assess modifiable risk factors for diseases with significant morbidity proactively.
  2. Illness script/Test-taking strategy:
    • In PAD patients with risk factors like smoking, it is essential to identify and manage modifiable risk factors, such as dyslipidemia, to prevent disease progression and associated cardiovascular risks.
  3. Why other answers are wrong:
    • A. Pentoxifylline therapy: Used to treat intermittent claudication in PAD, but the patient has no such symptoms.
    • B. Peripheral artery catheterization:  An invasive diagnostic procedure reserved for patients considering revascularization.
    • D. Warfarin therapy: Not the primary therapy for PAD.
    • E. No further management is indicated: Indicated; Given the clinical signs suggestive of PAD, further evaluation and management are certainly indicated.

Test-taking tip: Always consider the most significant modifiable risk factors when dealing with chronic diseases, especially vascular ones. Addressing these can substantially alter the disease’s course.

Reference:

  • Le, T., Bhushan, V., Ascha, M., Bhardwaj, A., Boushra, M., & Griffin, D. (2023). First Aid for the USMLE Step 2 CK, eleventh edition (11th ed.). (p. 81 – 82). McGraw-Hill Education.

Question 8

A 57-year-old woman comes to the emergency department because of a 2-day history of fever, chills, and sore throat. She is on day 6 of a 7-day course of trimethoprim-sulfamethoxazole to treat cystitis……….

Answer: C Adverse effect of trimethoprim-sulfamethoxazole

  1. Why the correct answer is right:
    • Trimethoprim-sulfamethoxazole (TMP-SMX) can cause drug-induced agranulocytosis.
    • The recent initiation of TMP-SMX, fever, sore throat, and a marked decrease in white blood cells with relative neutropenia strongly suggest this adverse reaction.
    • Agranulocytosis is characterized by an acute decrease in circulating granulocytes, primarily neutrophils.
  2. Illness script/Test-taking strategy:
    • Recognize the scenario of a recent medication (especially TMP-SMX) initiation followed by signs of infection and marked leukopenia (especially neutropenia) on labs.
    • Always review potential adverse effects of recent medications when evaluating new symptoms.
  3. Why other answers are wrong:
    • A. Acute mononucleosis: Presents with fever, sore throat, and lymphocytosis. The mononuclear cells in mononucleosis are atypical.
    • B. Acute myelogenous leukemia: Pancytopenia and blast cells on the smear.
    • D. Allergic reaction to lisinopril: Cause angioedema but not neutropenia.
    • E. Myelofibrosis: Chronic condition leading to bone marrow scarring, resulting in pancytopenia and teardrop cells on peripheral smear.
    • F. Sepsis syndrome: Sepsis can cause leukocytosis or leukopenia, but TMP-SMX use suggests drug-induced agranulocytosis.

Test-taking tip: Consider drug side effects in the differential, especially when a drug has recently been started or changed, and the presentation aligns with known adverse reactions.

Reference:

  • Le, T., Bhushan, V., & Sochat, M. (2023). First Aid for the USMLE Step 1 (p. 191). McGraw-Hill Education.
  • Le, T., Bhushan, V., Ascha, M., Bhardwaj, A., Boushra, M., & Griffin, D. (2023). First Aid for the USMLE Step 2 CK, eleventh edition (11th ed.). (p. 749t). McGraw-Hill Education.

Question 9

A 68-year-old man comes to the clinic because of a 6-month history of frequent falls. He sustained a fracture of the left wrist 4 weeks ago during a fall…. Parkinson’s disease treated with carbidopa-levodopa and entacapone……

Answer: B Physical therapy

  1. Why the correct answer is right:
    • Patients with Parkinson’s disease often develop postural instability, characterized by impaired balance and a tendency to fall.
    • This patient is experiencing postural instability.
    • A crucial intervention for these patients is physical therapy, emphasizing balance training and exercises to improve gait.
  2. Illness script/Test-taking strategy:
    • Recognize that postural instability in Parkinson’s patients increases their risk for falls.
    • Physical therapy provides tailored exercises to address this issue, helping improve patients’ balance and mobility.
  3. Why other answers are wrong:
    • A. Biofeedback: Has no consistent benefit in addressing Parkinson’s-related postural instability.
    • C. Pramipexole therapy: A dopamine agonist, does not effectively help in postural instability or falling risk.
    • D. Ropinirole therapy: Similar to pramipexole, it does not directly reduce the risk of falls due to postural instability.
    • E. Rotigotine therapy: Dopamine agonist, no effect on the postural instability in Parkinson’s patients.

Test-taking tip: In managing conditions with multifactorial complications, like Parkinson’s disease, always consider non-pharmacological interventions like physical therapy, especially when the primary concern is mobility or balance.

Reference:

  • Le, T., Bhushan, V., & Sochat, M. (2023). First Aid for the USMLE Step 1 (p. 565, 566). McGraw-Hill Education.
  • Le, T., Bhushan, V., Ascha, M., Bhardwaj, A., Boushra, M., & Griffin, D. (2023). First Aid for the USMLE Step 2 CK, eleventh edition (11th ed.). (p. 397 – 399). McGraw-Hill Education.

Question 10

A 39-year-old man is brought to the emergency department for a 3-hour history of moderate pain and swelling of his right knee and ankle……..

Answer: C Knee – No diagnostic testing indicated, Ankle – X-ray

  1. Why the correct answer is right:
    • The patient’s knee shows no signs of significant injury, meeting no Ottawa Knee Rule criteria for imaging.
    • However, his ankle injury, with tenderness at the distal fibula and restricted movement, meets Ottawa Ankle Rule criteria, warranting an X-ray.
  2. Illness script/Test-taking strategy:
    • Always consider validated clinical decision rules when evaluating trauma to the extremities, like the Ottawa Ankle and Knee Rules.
    • They can guide when imaging is necessary, ensuring appropriate care while avoiding unnecessary radiation exposure.
  3. Why other answers are wrong:
    • A. Knee – X-ray, Ankle – X-ray: The knee findings do not warrant an X-ray based on the Ottawa Knee Rules.
    • B. Knee – X-ray, Ankle – No diagnostic testing indicated: The ankle findings suggest a potential fracture, warranting an X-ray.
    • D. Knee – no diagnostic testing indicated, Ankle – no diagnostic testing indicated: This overlooks the concerning ankle findings that warrant radiographic evaluation.

Test-taking tip: Familiarity with clinical decision rules, like the Ottawa Ankle and Knee Rules, can help streamline decision-making in such trauma scenarios. These rules are evidence-based and can assist in reducing unnecessary imaging.

Reference:

  • Le, T., Bhushan, V., Ascha, M., Bhardwaj, A., Boushra, M., & Griffin, D. (2023). First Aid for the USMLE Step 2 CK, eleventh edition (11th ed.). (p. 319). McGraw-Hill Education.

Question 11

A 39-year-old man comes to the office because of a 3-month history of episodes of headaches, palpitations, and sweating. The episodes occur two to three times monthly. He has hypertension treated with a calcium- channel blocking agent and an ACE inhibitor…….

Answer: D Measurement of plasma and urine catecholamine and metabolite concentrations

  1. Why the correct answer is right:
    • The patient’s presentation of paroxysmal headaches, palpitations, sweating, hypertension, and an adrenal mass suggests a pheochromocytoma, a catecholamine-secreting tumor.
    • The most appropriate next step for a suspected pheochromocytoma is to measure the concentrations of plasma and urine catecholamine (epinephrine, norepinephrine) and metabolite (metanephrine, normetanephrine) to confirm the diagnosis.
  2. Illness script/Test-taking strategy:
    • Recognize that the triad of headache, palpitations, and sweating highly suggests pheochromocytoma, especially in an adrenal mass.
    • The first step in confirming this diagnosis is assessing catecholamine and metabolite levels.
  3. Why other answers are wrong:
    • A. 24-Hour urine collection for Measurement of vanillylmandelic acid and 5-hydroxy indole acetic acid: Metabolites of catecholamines and serotonin, respectively – mainly used to diagnose neuroblastoma and carcinoid syndrome.
    • B. Adrenal venous sampling: Used to differentiate the cause of primary hyperaldosteronism (between an adrenal adenoma and bilateral adrenal hyperplasia).
    • C. Laparoscopic left adrenalectomy and right adrenal biopsy: Prior to surgery, diagnostic confirmation with catecholamine and metabolite testing is essential.
    • E. Transsphenoidal hypophysectomy: Used for pituitary tumors, not adrenal masses.

Test-taking tip: Clinical presentation drives the diagnostic pathway. In this case, the combination of the symptom triad and the finding of an adrenal mass immediately points to pheochromocytoma, making the diagnostic confirmation through catecholamine testing the next best step.

Reference:

  • Le, T., Bhushan, V., & Sochat, M. (2023). First Aid for the USMLE Step 1 (p. 355). McGraw-Hill Education.
  • Le, T., Bhushan, V., Ascha, M., Bhardwaj, A., Boushra, M., & Griffin, D. (2023). First Aid for the USMLE Step 2 CK, eleventh edition (11th ed.). (p. 150). McGraw-Hill Education.

Question 12

A 39-year-old man comes to the office because of a 6-week history of mild fatigue and increased thirst and urination. During this time, he also has had a 6.8-kg (15-Ib) weight loss despite no change in appetite…..

Answer: B Insulin therapy

  1. Why the correct answer is right:
    • Symptoms like (polyuria, polydipsia, weight loss), very high blood glucose, and elevated Hemoglobin A1c level are consistent with diabetes mellitus, most likely type 1, given his age, BMI, and symptom duration.
    • For type 1 diabetes, the beta cells of the pancreas fail to produce enough insulin, which leads to hyperglycemia.
    • These patients are insulin-deficient and must be started on insulin therapy immediately.
  2. Illness script/Test-taking strategy:
    • Recognize the classic presentation of diabetes with the combination of polyuria, polydipsia, and unexplained weight loss.
    • In a lean patient with rapid onset of symptoms, type 1 diabetes is more likely, and insulin is the initial treatment of choice.
  3. Why other answers are wrong:
    • A. Glyburide therapy: It is a sulfonylurea that stimulates the pancreas to release more insulin – used for type 2 diabetes when patients have some residual pancreatic function.
    • C. Metformin therapy: First-line drug for type 2 diabetes
    • D. Pioglitazone therapy: It is a thiazolidinedione used for type 2 diabetes.
    • E. Sitagliptin therapy: It is a dipeptidyl peptidase-4 (DPP-4) inhibitor used for type 2 diabetes.

Test-taking tip: When determining the type of diabetes, consider the clinical presentation, age of onset, and physical characteristics of the patient. Type 1 diabetics are usually younger, leaner, and present acutely, while type 2 diabetics are often older, overweight, and have a more indolent course. The treatment choices differ significantly between the two.

Reference:

  • Le, T., Bhushan, V., & Sochat, M. (2023). First Aid for the USMLE Step 1 (p. 248, 333, 342). McGraw-Hill Education.
  • Le, T., Bhushan, V., Ascha, M., Bhardwaj, A., Boushra, M., & Griffin, D. (2023). First Aid for the USMLE Step 2 CK, eleventh edition (11th ed.). (p. 142 – 144). McGraw-Hill Education.

Question 13

A 62-year-old man has severe shortness of breath 2 days after he was admitted to the hospital for management of an acute myocardial infarction. He initially underwent cardiac catheterization with angioplasty of the left anterior descending artery and is currently being treated with heparin, metoprolol, and aspirin……

Answer: B Acute left ventricular failure

  1. Why the correct answer is right:
    • The patient has acute pulmonary edema (shortness of breath, crackles on examination, decreased oxygen saturation) and hypotension following an acute myocardial infarction.
    • These findings indicate acute left ventricular failure.
    • His symptoms can be attributed to the acute loss of a substantial amount of myocardium, leading to decreased cardiac output and fluid buildup in the lungs.
  2. Illness script/Test-taking strategy:
    • Recognize that the most common complication after an Acute MI is a decline in left ventricular function leading to congestive heart failure and cardiogenic shock.
    • Signs of left heart failure include dyspnea, orthopnea, and crackles on examination due to pulmonary congestion.
  3. Why other answers are wrong:
    • A. Acute dilation of the aortic root: Presemt with severe chest pain and a wide mediastinum on chest x-ray.
    • C. Intravascular hypovolemia: Presents with low blood pressure, increased heart rate, decreased jugular venous distension, and dry mucous membranes.
    • D. New ventricular septal defect: A complication of MI and presents with a new loud holosystolic murmur best heard at the left lower sternal border.
    • E. Ruptured papillary muscle: Sudden onset causing severe mitral regurgitation with a new holosystolic murmur and pulmonary edema.

Test-taking tip: After an acute myocardial infarction, consider complications based on the post-MI time frame. Immediate complications (within 24 hours) include arrhythmias and heart failure, while structural complications like ventricular septal defect and papillary muscle rupture tend to occur days after the event.

Reference:

  • Le, T., Bhushan, V., & Sochat, M. (2023). First Aid for the USMLE Step 1 (p. 316, 700). McGraw-Hill Education.
  • Le, T., Bhushan, V., Ascha, M., Bhardwaj, A., Boushra, M., & Griffin, D. (2023). First Aid for the USMLE Step 2 CK, eleventh edition (11th ed.). (p. 23, 740). McGraw-Hill Education.

Question 14

A 12-year-old girl is brought to the office for a well-child examination. She feels well. She has no history of serious illness and receives no medications. Menarche has not yet occurred……

Answer: E No additional diagnostic steps are indicated

  1. Why the correct answer is right:
    • This 12-year-old girl has not yet exhibited any signs of puberty (breast development or the appearance of pubic hair).
    • The absence of secondary sexual characteristics by age 13 in girls or a delay of more than five years between the initial and complete development of these characteristics is abnormal.
    • In this case, there is still time before considering this a delayed puberty, so no additional diagnostic steps are required.
  2. Illness script/Test-taking strategy:
    • Recognize that the normal age range for the onset of puberty in girls is between 8 and 13 years.
    • Awareness of these age ranges helps differentiate normal from delayed puberty and dictates whether further evaluation is necessary.
  3. Why other answers are wrong:
    • A. Measurement of serum follicle-stimulating hormone and luteinizing hormone concentrations: No evidence of primary ovarian failure.
    • B. Measurement of serum growth hormone and thyroxine concentrations: No growth delay or hypothyroidism.
    • C. MRI of the brain: No suspicion of a central cause for delayed puberty, like a tumor impacting the pituitary or hypothalamus.
    • D. X-ray of the left hand and wrist to determine bone age: May help determine skeletal maturity and predict future growth, but unnecessary.

Test-taking tip: In questions about delayed puberty, always evaluate the patient’s age concerning the typical age range for the onset of puberty.

Reference:

  • Le, T., Bhushan, V., & Sochat, M. (2023). First Aid for the USMLE Step 1 (p. 55). McGraw-Hill Education.
  • Le, T., Bhushan, V., Ascha, M., Bhardwaj, A., Boushra, M., & Griffin, D. (2023). First Aid for the USMLE Step 2 CK, eleventh edition (11th ed.). (p. 470 – 471, 472t, 514). McGraw-Hill Education.

Question 15

A randomized controlled trial is conducted to assess the effectiveness of a new antiarrhythmic drug in patients with recurrent ventricular tachycardia…….

Answer: A Extrapolation of findings beyond data

  1. Why the correct answer is right:
    • The study only demonstrated decreased episodes of recurrent ventricular tachycardia, but the investigators concluded it could decrease mortality from cardiac arrhythmia.
    • For the USMLE/NBME, recognize that concluding mortality based solely on reductions in arrhythmia episodes is an overextension of the findings.
  2. Illness script/Test-taking strategy:
    • Always align the results of a study to its conclusion.
    • If the conclusion extends beyond the study’s results or scope, it is likely an extrapolation of findings.
  3. Why other answers are wrong:
    • B. Insufficient power: Statistically found significant difference (p<0.05) – indicates enough power.
    • C. No information regarding confidence interval: Absence of this info does not lead to the specific overextension made by the authors.
    • D. Selection bias: No evidence of inappropriate or biased selection of participants.

Test-taking tip: On USMLE, when assessing study validity, ensure that conclusions closely match and do not overextend the findings presented. If a study makes broad conclusions not directly supported by its data, suspect extrapolation of findings.


Question 16

A 5-day-old boy is brought to the office for an initial well-child examination. He was born at 40 weeks* gestation and discharged at 60 hours of life. On newborn screening, hemoglobin electrophoresis showed an FS pattern…..

Answer: E Penicillin prophylaxis 

  1. Why the correct answer is right:
    • Prophylactic penicillin is initiated by age two months in infants with SCD to prevent life-threatening infections, particularly Streptococcus pneumoniae.
    • For USMLE/NBME, recognize that an FS pattern on hemoglobin electrophoresis in a newborn typically indicates sickle cell disease (SCD).
  2. Illness script/Test-taking strategy:
    • When encountering a newborn with an FS pattern on electrophoresis, always think of SCD.
    • The USMLE often tests preventive strategies for sickle cell disease, with the early introduction of penicillin prophylaxis being a high-yield point.
  3. Why other answers are wrong:
    • A. Deferoxamine therapy: Used for iron overload, not a primary treatment for SCD.
    • B. Hydroxyurea therapy: May be used in older children with recurrent pain crises but not in neonates.
    • C. Iron supplementation: Not typically given in SCD unless there is a confirmed deficiency.
    • D. Monthly blood transfusions: Used for specific complications (recurrent stroke) but not as a routine measure.
    • F. Vitamin B12 (cyanocobalamin) supplementation: Relevant for B12 deficiency, not SCD.

Test-taking tip: In pediatric questions, the USMLE often emphasizes preventive measures. In SCD, early prevention against infections is vital, making penicillin prophylaxis a high-yield topic.

Reference:

  • Le, T., Bhushan, V., Ascha, M., Bhardwaj, A., Boushra, M., & Griffin, D. (2023). First Aid for the USMLE Step 2 CK, eleventh edition (11th ed.). (p. 567 – 569). McGraw-Hill Education.

Question 17

A 2-year-old girl is brought to the clinic by her parents for an initial examination. Medical records show that she has a cleft palate and ventriculosetal defects that have not been repaired……..

Answer: C Lymphopenia

  1. Why the correct answer is right:
    • For USMLE/NBME, the presence of both a cleft palate and cardiac defect (VSD) in the setting of recurrent infections (pneumonia and otitis media) should prompt consideration of DiGeorge syndrome (22q11.2 deletion syndrome).
    • A hallmark feature of DiGeorge syndrome is T-cell lymphopenia due to thymic hypoplasia or aplasia.
  2. Illness script/Test-taking strategy:
    • Consider DiGeorge syndrome when encountering a patient with congenital anomalies, especially cardiac and palatal defects combined with recurrent infections.
    • Remember: DiGeorge = developmental (cardiac and palatal defects) + immunodeficiency (T-cell lymphopenia).
  3. Why other answers are wrong:
    • A. Eosinophilia: Seen in parasitic infections, allergic reactions, and certain autoimmune conditions.
    • B. Lymphocytosis: Common in viral infections (like infectious mononucleosis).
    • D. Neutropenia: Found in certain congenital conditions, marrow suppression, or aplastic anemia.
    • E. Neutrophilia: Typical of bacterial infections and inflammatory processes.

Test-taking tip: For USMLE, connecting multisystem clinical findings to guide your differential diagnosis is essential. Recurrent infections plus congenital anomalies should lead you to think about syndromic causes with immunodeficiencies, such as DiGeorge syndrome.

Reference:

  • Le, T., Bhushan, V., & Sochat, M. (2023). First Aid for the USMLE Step 1 (p. 533, 543t). McGraw-Hill Education.
  • Le, T., Bhushan, V., Ascha, M., Bhardwaj, A., Boushra, M., & Griffin, D. (2023). First Aid for the USMLE Step 2 CK, eleventh edition (11th ed.). (p. 348). McGraw-Hill Education.

Question 18

A 57-year-old woman comes to the emergency department 1 hour after vomiting bright red blood. She thinks it was approximately 1 cup of blood. She reports that she has felt full sooner than usual during the past month while eating……..

Answer: E Esophagogastroduodenoscopy

  1. Why the correct answer is right:
    • The patient’s presentation of vomiting bright red blood (hematemesis) suggests an upper gastrointestinal bleed.
    • For the USMLE/NBME, recognize that EGD is the first-line diagnostic and therapeutic approach for suspected upper GI bleeding due to its direct visualization of bleeding sources in the esophagus, stomach, and duodenum.
  2. Illness script/Test-taking strategy:
    • When faced with a case of hematemesis, always consider EGD as the best initial diagnostic approach.
    • Consider the location (upper vs. lower GI), nature of bleeding (bright red blood vs. melena), and clinical context.
  3. Why other answers are wrong:
    • A. Abdominal CT scan: Not the first line for gastrointestinal bleeding.
    • B. Octreotide scan: Used to locate sources of ectopic endocrine activity, not direct bleeding.
    • C. Technetium 99m scan: Detects slow GI bleeds, but EGD is preferred for acute bleeds.
    • D. Colonoscopy: Evaluates lower GI tract.
    • F. Mesenteric angiography: Used for intermittent bleeding when other methods fail.

Test-taking tip: On the USMLE, hematemesis should direct your thinking towards an upper GI source, and the primary diagnostic tool for this is EGD due to its dual diagnostic and therapeutic utility.

Reference:

  • Le, T., Bhushan, V., & Sochat, M. (2023). First Aid for the USMLE Step 1 (p. 387). McGraw-Hill Education.
  • Le, T., Bhushan, V., Ascha, M., Bhardwaj, A., Boushra, M., & Griffin, D. (2023). First Aid for the USMLE Step 2 CK, eleventh edition (11th ed.). (p. 211 – 213). McGraw-Hill Education.

Question 19

A 57-year-old woman comes to the emergency department 1 hour after vomiting bright red blood. She thinks it was approximately 1 cup of blood. She reports that she has felt full sooner than usual during the past month while eating………. After intravenous administration of fluids and nasogastric tube lavage, the patient undergoes esophagogastroduodenoscopy……..

Answer: D Endoscopic hemostatic therapy

  1. Why the correct answer is right:
    • The patient’s presentation of sudden hematemesis, mid-epigastric tenderness, and positive stool occult blood suggests upper GI bleeding.
    • For USMLE/NBME, immediate endoscopic hemostatic therapy is the appropriate management if an active source of bleeding or a high-risk lesion is identified on esophagogastroduodenoscopy.
  2. Illness script/Test-taking strategy:
    • Sudden upper GI bleeding, especially with findings of high-risk lesions on endoscopy, should prompt consideration for endoscopic intervention.
  3. Why other answers are wrong:
    • A. Octreotide therapy: Primarily for variceal bleeding due to portal hypertension.
    • B. Omeprazole therapy: For peptic ulcer disease, not for bleeding.
    • C. Tyrosine kinase inhibitor therapy: Used in gastrointestinal stromal tumors.
    • E. Endoscopic biopsy: Biopsy is indicated to diagnose lesions.
    • F. Endoscopic resection: Used for removal of tumors or polyps.

Test-taking tip: On the USMLE, focus on the urgency of the clinical scenario. In active upper GI bleeding, stabilizing the patient and controlling the source of bleeding are priorities.

Reference:

  • Le, T., Bhushan, V., & Sochat, M. (2023). First Aid for the USMLE Step 1 (p. 387). McGraw-Hill Education.
  • Le, T., Bhushan, V., Ascha, M., Bhardwaj, A., Boushra, M., & Griffin, D. (2023). First Aid for the USMLE Step 2 CK, eleventh edition (11th ed.). (p. 211 – 213). McGraw-Hill Education.

Question 20

A 32-year-old man comes to the office because of a 2-week history of persistent nonproductive cough occurring in short bursts. During the past week, he has had bilateral chest pain with severe coughing…….

Answer: B Azithromycin

  1. Why the correct answer is right:
    • The patient’s presentation suggests post-infectious cough resulting from a prior upper respiratory tract infection, frequently caused by Bordetella pertussis (whooping cough).
    • For the USMLE/NBME, it is crucial to remember that macrolides, such as azithromycin, is the treatment for pertussis in adults.
  2. Illness script/Test-taking strategy:
    • Think of pertussis when a patient presents with a protracted cough (often > two weeks), especially if associated with paroxysms, post-tussive vomiting, or inspiratory “whoop.”
    • Although the “whoop” is less common in adults, the persistent cough can lead to complications like rib fractures.
  3. Why other answers are wrong:
    • A. Albuterol: Used for bronchospasm, common in asthma or COPD.
    • C. Levofloxacin: A fluoroquinolone antibiotic for more severe bacterial infections.
    • D. Omeprazole: PPI for GERD.
    • E. Prednisone: For inflammatory conditions.

Test-taking tip: On the USMLE, when encountering a persistent cough after a URI, especially post-tussive vomiting, always consider pertussis in your differential, even in adults.

Reference:

  • Le, T., Bhushan, V., & Sochat, M. (2023). First Aid for the USMLE Step 1 (p. 183). McGraw-Hill Education.
  • Le, T., Bhushan, V., Ascha, M., Bhardwaj, A., Boushra, M., & Griffin, D. (2023). First Aid for the USMLE Step 2 CK, eleventh edition (11th ed.). (p. 554 – 555). McGraw-Hill Education.

Question 21

A 1-week-old female newborn is brought to the office for a follow-up examination after newborn screening showed a serum TSH concentration of 40 wU/mL (N=1-20). She has been breastfeeding well……

Answer: B Levothyroxine therapy

  1. Why the correct answer is right:
    • The neonate has an elevated TSH and a low free thyroxine (FT) concentration, indicating primary hypothyroidism.
    • For USMLE/NBME, it is crucial to know that congenital hypothyroidism is commonly caused by thyroid dysgenesis and warrants early initiation of levothyroxine therapy to prevent intellectual disability.
  2. Illness script/Test-taking strategy:
    • Recognize the key findings of elevated TSH and low FT in neonates to indicate primary hypothyroidism.
    • Early identification and treatment are vital to preventing irreversible developmental and intellectual delays.
  3. Why other answers are wrong:
    • A. Hydrocortisone therapy: Not indicated for primary hypothyroidism.
    • C. Monthly thyroglobulin measurement: Not the primary marker for thyroid function.
    • D. Monthly TSH and FT measurements: Monitoring is essential, but immediate treatment is crucial.
    • E. Radioactive iodine uptake scan: Not safe or indicated in neonates.
    • F. Ultrasonography of the thyroid gland: Can reveal structural anomalies, No role in treatment.

Test-taking tip: On USMLE, primary hypothyroidism in neonates presents with increased TSH and decreased thyroxine levels. Immediate treatment takes precedence over serial monitoring to prevent developmental issues.

Reference:

  • Le, T., Bhushan, V., & Sochat, M. (2023). First Aid for the USMLE Step 1 (p. 344 – 345). McGraw-Hill Education.
  • Le, T., Bhushan, V., Ascha, M., Bhardwaj, A., Boushra, M., & Griffin, D. (2023). First Aid for the USMLE Step 2 CK, eleventh edition (11th ed.). (p. 134-135). McGraw-Hill Education.

Question 22

A 27-year-old man comes to the emergency department because of a 4-week history of progressive shortness of breath and swelling of the lower extremities. During this time, he also has had shortness of breath during the night while lying down……..

Answer: D Methamphetamine

  1. Why the correct answer is right:
    • This patient’s presentation of shortness of breath, exertional chest pain, hypertension, severe gingivitis (“meth mouth”), and findings suggestive of congestive heart failure are characteristic of chronic methamphetamine use.
    • The USMLE/NBME expects recognition of the systemic manifestations of drug abuse, particularly methamphetamine’s cardiovascular and dental impacts.
  2. Illness script/Test-taking strategy:
    • Methamphetamine abuse can lead to cardiomyopathy and heart failure.
    • When presenting a young patient with cardiovascular symptoms combined with dental pathology, always consider methamphetamine use.
  3. Why other answers are wrong:
    • A. Cocaine: may cause myocardial ischemia and infarction but is not linked with heart failure or gingivitis.
    • B. Heroin: More associated with pulmonary complications like non-cardiogenic pulmonary edema.
    • C. Methadone: Used for opioid dependence; can cause respiratory depression.
    • E. Toluene: Inhalant abuse cause multi-organ damage, but not the specific findings described.

Test-taking tip: On USMLE, Always look for a combination of systemic findings. Here, cardiovascular and dental issues pointed strongly toward methamphetamine.

Reference:

  • Le, T., Bhushan, V., & Sochat, M. (2023). First Aid for the USMLE Step 1 (p. 590, 593). McGraw-Hill Education.
  • Le, T., Bhushan, V., Ascha, M., Bhardwaj, A., Boushra, M., & Griffin, D. (2023). First Aid for the USMLE Step 2 CK, eleventh edition (11th ed.). (p. 34 – 40). McGraw-Hill Education.

Question 23

A 22-year-old man is brought to the emergency department by his girlfriend 30 minutes after she found him unconscious in his home. She last saw him yesterday, and he appeared well. He has a history of cocaine and heroin use………

Answer: A Acute kidney injury

  1. Why the correct answer is right:
    • This young patient with a history of drug abuse presents with stupor, bradypnea, and a cold, cyanotic limb, suggesting opioid overdose and non-traumatic rhabdomyolysis (evidenced by the markedly elevated creatine kinase).
    • A common complication of rhabdomyolysis, as per USMLE/NBME content, is myoglobin-induced AKI.
  2. Illness script/Test-taking strategy:
    • In the setting of drug overdose with accompanying evidence of muscle injury (elevated CK), always consider rhabdomyolysis and its potential renal implications.
    • For the USMLE, connect high CK in an overdose patient with rhabdomyolysis, leading to AKI.
  3. Why other answers are wrong:
    • B. Acute liver failure: Acetaminophen overdose.
    • C. Cardiac arrhythmia: Electrolyte imbalances like hyperkalemia are not the primary risk from rhabdomyolysis.
    • D. Hypocalcemia & E. Hypophosphatemia: Electrolyte disturbances can arise from rhabdomyolysis, but AKI is the most immediate and life-threatening complication.

Test-taking tip: On the USMLE, be alert to clinical overdose scenarios. While multiple complications may arise, always prioritize immediate life-threatening risks based on lab findings and clinical presentation.

Reference:

  • Le, T., Bhushan, V., & Sochat, M. (2023). First Aid for the USMLE Step 1 (p. 623). McGraw-Hill Education.
  • Le, T., Bhushan, V., Ascha, M., Bhardwaj, A., Boushra, M., & Griffin, D. (2023). First Aid for the USMLE Step 2 CK, eleventh edition (11th ed.). (p. 341 & 745t). McGraw-Hill Education.

Question 24

A 31-year-old woman comes to the office because of a 3-month history of intermittent nonbloody diarrhea. The first episode began 5 days after a camping trip in rural Virginia with her family. During the trip, she drank from a spring-fed pond that she says was “clean.”………

Answer: E Giardia lamblia

  1. Why the correct answer is right:
    • This woman’s non-bloody diarrhea, which started after drinking from a spring-fed pond, is consistent with giardiasis, a protozoal infection.
    • For the USMLE/NBME, always link untreated freshwater exposure with Giardia lamblia as a potential etiology.
  2. Illness script/Test-taking strategy:
    • Giardiasis often follows ingestion of cyst-contaminated water. Symptoms include non-bloody diarrhea, bloating, flatulence, and sometimes weight loss.
    • On exams, a history of freshwater exposure (especially in rural settings) is a key clue for this protozoan infection.
  3. Why other answers are wrong:
    • A. Campylobacter jejuni: Causes bloody diarrhea and is linked with poultry consumption.
    • B. Clostridioides difficile: Associated with antibiotic use leading to pseudomembranous colitis.
    • C. Entamoeba histolytica: Cause dysentery and liver abscess in some cases.
    • D. Escherichia coli: Several pathogenic strains, but freshwater exposure is not a typical source.
    • F. Salmonella enteritidis: Linked with raw eggs or poultry consumption, it causes fever and bloody diarrhea.
    • G. Shigella dysenteriae: Produces dysentery

Test-taking tip: On USMLE, environmental exposure often holds clues to diagnosis. Freshwater exposure should lead one to consider Giardia among the differentials.

Reference:

  • Le, T., Bhushan, V., & Sochat, M. (2023). First Aid for the USMLE Step 1 (p. 152). McGraw-Hill Education.

Question 25

A 27-year-old woman comes to the office because of a 3-day history of constant mild pain in her left lower abdomen. Vigorous movement makes the pain worse. She has not had nausea. She is otherwise healthy and takes no medications. Menses occur at regular 28-day intervals and last 4 to 6 days………

Answer: E Observation only

  1. Why the correct answer is right:
    • The clinical presentation and ultrasonography findings indicate a probable ruptured ovarian cyst, common in reproductive-aged women.
    • For USMLE/NBME, recall that uncomplicated ovarian cyst rupture usually presents with sudden mild-to-moderate pain and may have associated free fluid without significant hemodynamic changes.
  2. Illness script/Test-taking strategy:
    • A reproductive-aged woman presenting with unilateral pelvic pain, adnexal tenderness, and ultrasound findings of an adnexal cyst and free fluid should raise suspicion for a ruptured ovarian cyst.
    • Always correlate the timing of the menstrual cycle with symptoms, as mid-cycle is common for ruptures.
  3. Why other answers are wrong:
    • A. Antibiotic therapy: Treatment for PID, but without signs of infection,
    • B. Appendectomy: For acute appendicitis.
    • C. CT scan-guided aspiration: Used for uncertain pelvic masses; the diagnosis is clear here.
    • D. Laparoscopy: An invasive procedure reserved for cases where torsion, ectopic pregnancy, or severe hemorrhage is suspected.

Test-taking tip: For USMLE, associate unilateral pelvic pain in a reproductive-aged woman with either ovarian cyst rupture or torsion. Differentiate based on clinical severity and associated findings.

Reference:

  • Le, T., Bhushan, V., & Sochat, M. (2023). First Aid for the USMLE Step 1 (p. 665). McGraw-Hill Education.
  • Le, T., Bhushan, V., Ascha, M., Bhardwaj, A., Boushra, M., & Griffin, D. (2023). First Aid for the USMLE Step 2 CK, eleventh edition (11th ed.). (p. 227). McGraw-Hill Education.

Question 26

A 78-year-old man is evaluated in the hospital following admission 8 hours ago for treatment of aspiration pneumonia. He has a 5-year history of intermittent dysphagia resulting in spontaneous regurgitation of undigested food and liquids. Over-the-counter antacids………A 78-year-old man is evaluated in the hospital following admission 8 hours ago for treatment of aspiration pneumonia. He has a 5-year history of intermittent dysphagia resulting in spontaneous regurgitation of undigested food and liquids. Over-the-counter antacids………

Answer: D Zenker diverticulum

  1. Why the correct answer is right:
    • The patient’s presentation of longstanding intermittent dysphagia associated with regurgitation of undigested food, without weight loss or heartburn, is characteristic of Zenker diverticulum.
    • On USMLE/NBME, recognize that Zenker diverticulum can lead to food retention and predispose to aspiration.
  2. Illness script/Test-taking strategy:
    • Elderly patients with isolated symptoms of regurgitation of undigested food should make you think of Zenker diverticulum, a false diverticulum at the pharyngoesophageal junction.
    • The absence of systemic symptoms like weight loss is a hint against malignancy.
  3. Why other answers are wrong:
    • A. Achalasia: Presents with dysphagia to solids and liquids, chest discomfort, and weight loss.
    • B. Esophageal cancer: Presents with progressive dysphagia (initially to solids, then liquids), weight loss, and possibly odynophagia.
    • C. Hiatal hernia: Mostly asymptomatic but present with heartburn and regurgitation that improves with antacids.

Test-taking tip: On the USMLE, the regurgitation of undigested food without esophageal symptoms suggests a proximal (pharyngeal) source of dysphagia, like Zenker diverticulum.

Reference:

  • Le, T., Bhushan, V., & Sochat, M. (2023). First Aid for the USMLE Step 1 (p. 391). McGraw-Hill Education.
  • Le, T., Bhushan, V., Ascha, M., Bhardwaj, A., Boushra, M., & Griffin, D. (2023). First Aid for the USMLE Step 2 CK, eleventh edition (11th ed.). (p. 209). McGraw-Hill Education.

Question 27

A 36-year-old woman, gravida 2, para 1, at 39 weeks gestation, is admitted to the hospital in labor. She reports painful contractions that occur every 2 to 3 minutes and last 60 seconds. She has not had vaginal bleeding………

Answer: C Expectant management

  1. Why the correct answer is right:
    • The clinical scenario describes a woman in active labor with a reassuring fetal heart rate (as indicated by moderate variability, spontaneous accelerations, and absence of decelerations).
    • For the USMLE/NBME, it is essential to recognize that there is no rush to expedite delivery in the absence of fetal distress or maternal indications.
  2. Illness script/Test-taking strategy:
    • A reassuring fetal heart rate with good cervical progress in active labor suggests a normal labor process.
    • When there are no signs of fetal distress or maternal complications, allowing the labor to progress naturally is the best course of action.
  3. Why other answers are wrong:
    • A. Advising the patient to begin pushing: Reserved for when the cervix is fully dilated (10 cm).
    • B. Amnioinfusion: Alleviate umbilical cord compression when variable decelerations are seen.
    • D.Forceps-assisted vaginal delivery: Indicated for specific fetal mal presentations or non-reassuring fetal status when the cervix is fully dilated.
    • E.Immediate cesarean delivery: Needed for emergencies like cord prolapse, uterine rupture, or a sustained non-reassuring fetal status.

Test-taking tip: On USMLE, the emphasis is often on normal processes and natural progression. Unless there is a clear indication, invasive interventions are not favored.

Reference:

  • Le, T., Bhushan, V., Ascha, M., Bhardwaj, A., Boushra, M., & Griffin, D. (2023). First Aid for the USMLE Step 2 CK, eleventh edition (11th ed.). (p. 454 – 456). McGraw-Hill Education.

Question 28

A 17-year-old girl comes to the office because of a 3-month history of daily moderate epigastric pain that is more severe after meals. Three weeks ago, she had watery diarrhea for 2 days, which resolved spontaneously……….

Answer: B Endoscopy

  1. Why the correct answer is right:
    • This teen’s epigastric pain worsened by meals, without any alarming features (e.g., weight loss, anemia, vomiting), suggests peptic ulcer disease or gastritis.
    • For the USMLE/NBME, it is crucial to understand that endoscopy is the best diagnostic modality to visualize and biopsy gastric mucosa to identify these conditions.
  2. Illness script/Test-taking strategy:
    • Consider peptic ulcer disease or gastritis in patients with mealassociated epigastric pain, especially when they do not have systemic symptoms.
    • For the USMLE, remember endoscopy as the go-to diagnostic tool.
  3. Why other answers are wrong:
    • A. CT scan of the abdomen: More useful for assessing structural abnormalities or masses.
    • C. Examination of the stool for ova and parasites: Useful for suspected parasitic infections.
    • D.Stool culture: Pertinent for bacterial gastrointestinal infections.
    • E.Upper gastrointestinal series: Can identify ulcers but less sensitive than endoscopy.

Test-taking tip: When a case presents with isolated gastrointestinal symptoms without systemic or alarming features, consider localized mucosal diseases, and remember that the most direct way to view the mucosa is via endoscopy.

Reference:

  • Le, T., Bhushan, V., & Sochat, M. (2023). First Aid for the USMLE Step 1 (p. 386). McGraw-Hill Education.
  • Le, T., Bhushan, V., Ascha, M., Bhardwaj, A., Boushra, M., & Griffin, D. (2023). First Aid for the USMLE Step 2 CK, eleventh edition (11th ed.). (p. 214 – 216). McGraw-Hill Education.

Question 29

A 77-year-old woman is brought to the emergency department by her son because of a 3-day history of pain in her right wrist. The patient has advanced dementia, Alzheimer’s type and cannot provide her medical history. She is completely dependent on her son………

Answer: C Contact adult protective services

  1. Why the correct answer is right:
    • The clinical presentation suggests elder abuse, a key topic for USMLE/NBME.
    • The presence of multiple bruises over both forearms and wrists in an elderly patient with advanced dementia dependent on a single caregiver is concerning.
    • The lack of a clear mechanism for the fracture further raises suspicions.
  2. Illness script/Test-taking strategy:
    • For USMLE, multiple unexplained bruises or injuries in a dependent elderly patient should immediately raise suspicion for possible elder abuse, especially in the setting of neglect, such as poor hygiene.
  3. Why other answers are wrong:
    • A. Admit the patient to the hospital: Might temporarily protect the patient but not address the root cause.
    • B & D. Arrange placement/provide phone numbers: These might be potential next steps after addressing the immediate concern.
    • E. Recommend evaluation at a geriatric clinic: Does not directly tackle the issue of potential abuse.

Test-taking tip: On USMLE, when faced with evidence of potential abuse or neglect, the priority is to ensure the potential victim’s safety before addressing other health concerns.

Reference:

  • Le, T., Bhushan, V., & Sochat, M. (2023). First Aid for the USMLE Step 1 (p. 273). McGraw-Hill Education.
  • Le, T., Bhushan, V., Ascha, M., Bhardwaj, A., Boushra, M., & Griffin, D. (2023). First Aid for the USMLE Step 2 CK, eleventh edition (11th ed.). (p. 193). McGraw-Hill Education.

Question 30

A 9-year-old girl is brought to the clinic for a routine examination. She has a 4-year history of asthma. Her only medication is inhaled albuterol with a spacer as needed. She uses it only occasionally and has not used it at all during the past 4 weeks……….

Answer: B Add fluticasone by metered-dose inhaler with a spacer to the regimen

  1. Why the correct answer is right:
    • This child displays features of poorly controlled asthma, evidenced by nighttime coughing episodes and exercise-induced symptoms.
    • The USMLE/NBME expects students to recognize that adding a daily inhaled corticosteroid, like fluticasone, is the primary step in longterm asthma control.
  2. Illness script/Test-taking strategy:
    • Nighttime coughing and activity limitation are two hallmarks of poor asthma control.
    • An inhaled corticosteroid is indicated when symptoms occur more than twice a week or cause nighttime awakening more than twice a month.
  3. Why other answers are wrong:
    • A. Oral theophylline: An older treatment with significant side effects, not a first-line for children.
    • C. Salmeterol: LABA – not be used alone in asthma due to severe exacerbation risk; always combined with a corticosteroid.
    • D. Oral prednisone: Used for acute exacerbations.
    • E. No change: Her symptoms indicate poorly controlled asthma, necessitating a change in management

Test-taking tip: On USMLE, for asthma, always think “control” over “relief.” If symptoms are uncontrolled with a short-acting beta-agonist, move to daily control with an inhaled corticosteroid.

Reference:

  • Le, T., Bhushan, V., & Sochat, M. (2023). First Aid for the USMLE Step 1 (p. 708). McGraw-Hill Education.
  • Le, T., Bhushan, V., Ascha, M., Bhardwaj, A., Boushra, M., & Griffin, D. (2023). First Aid for the USMLE Step 2 CK, eleventh edition (11th ed.). (p. 626 – 628). McGraw-Hill Education.

Question 31

A 15-year-old girl is brought to the office because of a 3-day history of diffuse abdominal pain, fever, and vomiting. She has cerebral palsy, situs inversus, and severe scoliosis. She uses a wheelchair for ambulation………

Answer: D Premature closure

  1. Why the correct answer is right:
    • Premature closure refers to accepting a diagnosis before it has been fully verified
    • Given the patient’s complex medical background, the physician quickly attributed her symptoms to gastroenteritis without considering or ruling out more severe diagnoses.
  2. Illness script/Test-taking strategy:
    • Avoid anchoring bias in complex patients or those with atypical presentations (settling on an initial diagnosis without considering other possibilities).
    • USMLE often uses these scenarios to test the ability to think broadly about differential diagnoses.
  3. Why other answers are wrong:
    • A. Latent error: Refers to system-based problems that may lead to errors.
    • B. Near miss: An incident that could have led to harm but did not.
    • C. Non-preventable error: Mistakes that cannot be foreseen or prevented.
    • E. Systems failure: Breakdowns in organizational processes leading to errors.

Test-taking tip: For USMLE, recognize that complex patients, especially with rare conditions like situs inversus, may not present typically. Avoid being influenced by recent similar diagnoses and always verify before concluding.


Question 32

A 43-year-old man comes to the office because of a 2-week history of fatigue and mild pain in his right armpit. He has not had a fever or weight loss. Medical history is remarkable for type 2 diabetes mellitus and hypercholesterolemia……….

Answer: B. Cat-scratch disease

  1. Why the correct answer is right:
    • The presence of tender regional lymphadenopathy (axillary and epitrochlear nodes) in a patient with recent contact with cats points to cat-scratch disease.
    • For USMLE/NBME, remember that Bartonella henselae infection is often associated with cat scratches or bites.
  2. Illness script/Test-taking strategy:
    • The cat-scratch disease commonly presents with localized lymphadenopathy and a history of cat contact, particularly with kittens.
    • Consider it, especially when the clinical picture does not suggest a more aggressive systemic disease (like fever or weight loss).
  3. Why other answers are wrong:
    • A. Castleman disease: Presents with systemic symptoms with lymphadenopathy.
    • C. Hidradenitis suppurativa: Chronic skin condition with painful, recurrent abscesses in the axilla and groin.
    • D. T-cell lymphoma: Aggressive; present with systemic symptoms and widespread lymphadenopathy.
    • E. Tuberculosis: Commonly manifests with systemic symptoms and lung findings, not localized tender lymphadenopathy.

Test-taking tip: On the USMLE, use the patient’s clinical presentation combined with epidemiological hints (e.g., cat ownership) to narrow down the diagnosis.

Reference:

  • Le, T., Bhushan, V., & Sochat, M. (2023). First Aid for the USMLE Step 1 (p. 147). McGraw-Hill Education.
  • Le, T., Bhushan, V., Ascha, M., Bhardwaj, A., Boushra, M., & Griffin, D. (2023). First Aid for the USMLE Step 2 CK, eleventh edition (11th ed.). (p. 742t). McGraw-Hill Education.

Question 33

A 50-year-old woman comes to the office for a health maintenance examination. She feels well. Medical history is unremarkable, and she takes no medications. There is no family history of serious………….

Answer: A Ductal carcinoma in situ

  1. Why the correct answer is right:
    • Pleomorphic microcalcifications, especially in clusters, are a classic radiologic sign of Ductal Carcinoma In Situ (DCIS) on mammography.
    • For the USMLE/NBME, it is essential to identify this as an early, noninvasive form of breast cancer.
  2. Illness script/Test-taking strategy:
    • When encountering mammographic findings on the USMLE, always correlate the specific type of microcalcification with potential pathology
    • Pleomorphic and clustered microcalcifications are hallmarks of DCIS.
  3. Why other answers are wrong:
    • B. Fat necrosis: Presents after trauma or surgery with a palpable mass; may have oil cysts or calcifications.
    • C. Fibroadenoma: Most common benign breast tumor; with a welldefined, solid mass.
    • D. Mastitis: Breast tissue inflammation; with skin changes and redness, not microcalcifications
    • E. Sclerosing adenosis: A benign condition; calcifications but not pleomorphic type.

Test-taking tip: In breast imaging on the USMLE, the specific type and distribution of microcalcifications can provide significant diagnostic clues. Pleomorphic and clustered suggest malignancy.

Reference:

  • Le, T., Bhushan, V., & Sochat, M. (2023). First Aid for the USMLE Step 1 (p. 670). McGraw-Hill Education.
  • Le, T., Bhushan, V., Ascha, M., Bhardwaj, A., Boushra, M., & Griffin, D. (2023). First Aid for the USMLE Step 2 CK, eleventh edition (11th ed.). (p. 493, 494). McGraw-Hill Education.

Question 34

A 42-year-old woman is brought to the emergency department by ambulance 20 minutes after a head-on motor vehicle collision in which she was the restrained driver. On arrival, she is alert and reports chest and abdominal pain……

Answer: F Tube thoracostomy

  1. Why the correct answer is right:
    • Given the trauma history, decreased breath sounds on the right, and an arterial blood gas showing hypoxia, the scenario is consistent with traumatic hemothorax.
    • The USMLE/NBME expects students to know that tube thoracostomy is the immediate management for hemothorax.
  2. Illness script/Test-taking strategy:
    • After trauma, especially MVCs, look for classic signs like decreased breath sounds, chest pain, and hypoxia.
    • Abnormalities consistent with seatbelt signs, like those on the right upper abdomen, also warn of possible internal injury.
  3. Why other answers are wrong:
    • A. Bronchoscopy: For airway visualization and foreign body removal.
    • B. Intubation and mechanical ventilation: For respiratory failure.
    • C. Placement of a thoracic epidural: For pain management in thoracic surgeries.
    • D. Thoracentesis: Useful for pleural effusion diagnosis.
    • E. Thoracotomy: A highly invasive procedure for non-responsive cases.
    • G. Observation only: Inappropriate for such severity.

Test-taking tip: In trauma cases on the USMLE, always align the clinical presentation with imaging and lab findings. Hypoxia plus trauma background usually hints at a pneumothorax or hemothorax.

Reference:

  • Le, T., Bhushan, V., Ascha, M., Bhardwaj, A., Boushra, M., & Griffin, D. (2023). First Aid for the USMLE Step 2 CK, eleventh edition (11th ed.). (p. 756). McGraw-Hill Education.

Question 35

A 68-year-old woman returns to the office for a follow-up examination. One month ago, a urinalysis showed protein during the work-up of a suspected urinary tract infection. At that time, the patient was prescribed a 3-day course of oral cephalexin……….

Answer: E No additional testing is indicated

  1. Why the correct answer is right:
    • This elderly woman had transient proteinuria during a urinary tract infection, which has since resolved.
    • For USMLE/NBME, recognize that transient proteinuria (due to conditions like fever, exercise, or UTI) does not necessitate further evaluation once resolved.
  2. Illness script/Test-taking strategy:
    • UTIs can cause transient proteinuria.
    • The key is to reassess proteinuria after the infection has resolved.
    • If the repeat urinalysis post-treatment is normal, no further evaluation for proteinuria is needed.
  3. Why other answers are wrong:
    • A. Determination of urine albumin: creatinine ratio & B. 24-Hour urine collection: For quantifying proteinuria, but not required if repeat urinalysis post-UTI treatment is normal.
    • C. Ultrasonography of the kidneys: For structural abnormalities of the kidneys or to evaluate for obstruction.
    • D. Urine sulfosalicylic acid method: Detects protein in the urine but is redundant if the standard dipstick test is negative.

Test-taking tip: On USMLE, transient conditions causing proteinuria often resolve after treating the underlying condition. It is essential to recheck urinalysis before diving into extensive work-ups.

Reference:

  • Le, T., Bhushan, V., & Sochat, M. (2023). First Aid for the USMLE Step 1 (p. 179). McGraw-Hill Education.
  • Le, T., Bhushan, V., Ascha, M., Bhardwaj, A., Boushra, M., & Griffin, D. (2023). First Aid for the USMLE Step 2 CK, eleventh edition (11th ed.). (p. 706 – 711). McGraw-Hill Education.

Question 36

A 42-year-old woman comes to the office because of a 2-week history of sore throat and fullness in her neck. She has no history of serious illness and takes no medications. She appears anxious……….

Answer: D Subacute thyroiditis

  1. Why the correct answer is right:
    • The patient’s presentation with a sore throat, fullness in the neck, tender anterior neck palpation, absence of oropharyngeal exudates, and systemic symptoms like fever and tachycardia are consistent with subacute thyroiditis.
    • USMLE/NBME often presents subacute thyroiditis as a painful thyroid condition following an upper respiratory infection.
  2. Illness script/Test-taking strategy:
    • Subacute thyroiditis, also called De Quervain’s thyroiditis, often presents with neck pain (often mistaken for pharyngitis), tender thyroid gland, systemic symptoms, and transient thyrotoxicosis.
    • The key is to differentiate it from other throat/neck pathologies.
  3. Why other answers are wrong:
    • A. Acute mononucleosis: Fever, pharyngitis, lymphadenopathy, and atypical lymphocytes in the blood.
    • B. Gastroesophageal reflux disease: May cause a sore throat due to acid reflux but no tender neck fullness.
    • C. Laryngitis: Hoarseness and voice change but no tender neck fullness.
    • E. Tracheitis: Tracheal inflammation due to bacterial infection with a cough, high fever, and stridor.

Test-taking tip: For USMLE, consider subacute thyroiditis when a patient has neck pain with a tender thyroid gland but lacks classic signs of bacterial or viral pharyngitis.

Reference:

  • Le, T., Bhushan, V., & Sochat, M. (2023). First Aid for the USMLE Step 1 (p. 213 & 345). McGraw-Hill Education.
  • Le, T., Bhushan, V., Ascha, M., Bhardwaj, A., Boushra, M., & Griffin, D. (2023). First Aid for the USMLE Step 2 CK, eleventh edition (11th ed.). (p.135). McGraw-Hill Education.

Question 37

A 45-year-old man comes to the office because of a 6-month history of difficulty sleeping, nervousness, and fatigue. He goes to bed at 11 pm every night, falls asleep within 15 minutes, then sleeps “fitfully.” After he awakens, he has mild neck pain and feels tired, edgy, and tense……….

Answer: A Buspirone

  1. Why the correct answer is right:
    • This patient has generalized anxiety disorder (GAD), evidenced by excessive worry about multiple areas (finances and job) for more than six months, causing significant distress or impairment (fatigue, muscle tension).
    • For USMLE/NBME, recognize that buspirone is a 2nd-line non-addictive anxiolytic suitable for patients, especially those with a history of substance abuse like this patient. 1st line therapy is CBT + SSRI.
  2. Illness script/Test-taking strategy:
    • GAD presents excessive worry over several domains, difficulty controlling the worry, and associated symptoms such as restlessness, fatigue, concentration problems, irritability, muscle tension, and sleep disturbance.
    • For the USMLE, always link the history of substance use with safer, non-addictive options like buspirone for anxiety.
  3. Why other answers are wrong:
    • B. Clonazepam: A benzodiazepine, not ideal due to the potential for dependence.
    • C. Diphenhydramine: An antihistamine; sedating but not primarily for anxiety.
    • D. Imipramine: A tricyclic antidepressant, not the first-line for GAD.
    • E. Quetiapine: An antipsychotic not primarily for GAD.
    • F. Temazepam: A benzodiazepine; has potential for dependence.

Test-taking tip: For GAD on the USMLE, particularly with comorbid substance use disorder, steer clear from potentially addictive substances and consider the safest therapeutic options.

Reference:

  • Le, T., Bhushan, V., & Sochat, M. (2023). First Aid for the USMLE Step 1 (p. 583). McGraw-Hill Education.
  • Le, T., Bhushan, V., Ascha, M., Bhardwaj, A., Boushra, M., & Griffin, D. (2023). First Aid for the USMLE Step 2 CK, eleventh edition (11th ed.). (p. 594 – 595). McGraw-Hill Education.

Question 38

A 17-year-old boy is brought to the emergency department because of a 1-hour history of progressive difficulty breathing and a 2-hour history of an itchy, red rash over his trunk, arms, and legs. He has not used new soaps, detergents, or lotions during the past week…………

Answer: C Administration of epinephrine

  1. Why the correct answer is right:
    • The patient’s symptoms of itchy rash, respiratory distress, and recent exposure to a new medication (amoxicillin) indicate an anaphylactic reaction.
    • For USMLE/NBME purposes, remember that anaphylaxis is a severe, systemic allergic reaction requiring immediate treatment, and epinephrine is the first-line therapy.
  2. Illness script/Test-taking strategy:
    • Anaphylaxis presents with a combination of cutaneous (itchy, red rash), respiratory (difficulty breathing), and often cardiovascular (hypotension) symptoms following exposure to a trigger.
    • Recognize the triad quickly in exams.
  3. Why other answers are wrong:
    • A. Administration of albuterol: For bronchospasm due to asthma or COPD; not first-line for anaphylaxis.
    • B. Administration of diphenhydramine: For allergic reactions but not acute anaphylaxis, it can be adjunctive but not primary therapy.
    • D. Complete blood count: Does not address the acute situation.
    • E. Observation only: Anaphylaxis is life-threatening and requires immediate intervention.

Test-taking tip: In USMLE questions, prioritize life-saving measures when facing a rapid-onset allergic reaction involving multiple systems. Anaphylaxis always demands quick epinephrine administration.

Reference:

  • Le, T., Bhushan, V., & Sochat, M. (2023). First Aid for the USMLE Step 1 (p. 110 – 112). McGraw-Hill Education.
  • Le, T., Bhushan, V., Ascha, M., Bhardwaj, A., Boushra, M., & Griffin, D. (2023). First Aid for the USMLE Step 2 CK, eleventh edition (11th ed.). (p. 718t). McGraw-Hill Education.

Question 39

Patient Information
Age: 28 years
Gender: M, self-identified
Race/Ethnicity: unspecified
Site of Care: clinic
History.
Reason for Visit/Chief Concern: follow-up 1 week after undergoing colonoscopy because of a 3-month history of hematochezia and iron deficiency anemia………

Answer: D Smoking cessation

  1. Why the correct answer is right:
    • The patient’s colonoscopy findings and presentation suggest Crohn’s disease, characterized by “skip lesions” (areas of disease interspersed with normal mucosa).
    • For USMLE/NBME, it is crucial to recognize that smoking is a known risk factor for disease exacerbation and poor outcomes in Crohn’s disease.
  2. Illness script/Test-taking strategy:
    • Loose bowel movements, hematochezia, and colonoscopy findings showing ulcers with normal mucosa in-between should lead you to consider Crohn’s disease.
    • Smoking worsens Crohn’s; hence, smoking cessation is often recommended.
  3. Why other answers are wrong:
    • A. Annual screening for lymphoproliferative disorders: Not routinely recommended.
    • B. Discontinuation of citalopram therapy: Citalopram, an SSRI, does not exacerbate Crohn’s symptoms.
    • C. Prophylactic colectomy: More aggressive treatment is not considered before addressing modifiable risk factors like smoking.

Test-taking tip: For USMLE, whenever Crohn’s disease is hinted at or diagnosed, consider smoking cessation a critical intervention due to the known worsening effects of smoking on the disease.

Reference:

  • Le, T., Bhushan, V., & Sochat, M. (2023). First Aid for the USMLE Step 1 (p. 389). McGraw-Hill Education.
  • Le, T., Bhushan, V., Ascha, M., Bhardwaj, A., Boushra, M., & Griffin, D. (2023). First Aid for the USMLE Step 2 CK, eleventh edition (11th ed.). (p. 241). McGraw-Hill Education.

Question 40

A 32-year-old woman is admitted to the hospital because of a 2-year history of intractable seizures. Her boyfriend, who has accompanied her, describes the seizures as episodes of bilateral limb shaking and moving her head from side to side, during which the patient closes her eyes and cries. The episodes last 15 to 20 minutes………

Answer:  A Cognitive behavioral therapy

  1. Why the correct answer is right:
    • The patient’s seizures not correlating with any EEG changes suggest they are psychogenic non-epileptic seizures (PNES) rather than epileptic seizures.
    • PNES can be a physical manifestation of psychological distress.
    • For the USMLE/NBME, recognize PNES based on a history of prior trauma (like abuse) and seizures not associated with EEG changes. CBT is the primary treatment for PNES.
  2. Illness script/Test-taking strategy:
    • For seizures with prolonged duration, retained consciousness, and an absence of postictal confusion or EEG changes, consider PNES.
    • History of trauma or abuse can be a clue.
  3. Why other answers are wrong:
    • B. Hypnotic therapy: Not the primary treatment.
    • C. Increasing the dosage of pregabalin: This patient’s seizures are not epileptic.
    • D. Psychoanalytic therapy: CBT is the evidence-based approach for treatment.
    • E. Surgical resection of the epileptogenic focus: Non-epileptic seizures; no focus to resect.

Test-taking tip: In the context of USMLE, always correlate clinical findings with diagnostic tests. Seizure activity without EEG changes should make you suspect PNES, especially with a background of trauma.

Block 2

Question 41

A surgical intensive care unit observes that the number of days patients have central venous catheters in place and the number of bloodstream infections have been increasing……….

Answer: A Ask clinical providers in the unit to identify what they believe to be the most common factors leading to nonremoval of central venous catheters

  1. Why the correct answer is right:
    • The first step in quality improvement is to understand the root causes of a problem.
    • Engaging healthcare providers in identifying barriers helps develop effective strategies to address the problem.
    • For USMLE/NBME, understand that clinical providers working directly with the patients will have firsthand knowledge about real-world issues and barriers to catheter removal.
  2. Illness script/Test-taking strategy:
    • Central venous catheters can increase the risk of bloodstream infections when they remain in place for prolonged periods.
    • In situations involving quality improvement, always identify root causes before implementing changes.
  3. Why other answers are wrong:
    • B. Changing all central venous catheters every five days: Leads to unnecessary procedures and does not address the root cause.
    • C. Placing stickers: May not address specific reasons for prolonged catheter placement.
    • D. Randomized controlled trial: While RCTs are the gold standard, they are time-consuming and not an efficient initial step in a quality improvement initiative.

Test-taking tip: On USMLE, consider the foundational steps in clinical processes or quality improvement before more elaborate interventions. Identifying problems is a crucial first step.


Question 42

A 25-year-old primigravid woman at 28 weeks’ gestation comes to the office for a routine prenatal visit. At 18 weeks gestation, she had vaginal bleeding that resolved spontaneously. At that time, her blood group was determined to be O, Rhnegative……….

Answer: C Administration of Rh. (D) immune globulin

  1. Why the correct answer is right:
    • RhoD immune globulin is given prophylactically to prevent sensitization of Rh-negative mothers, especially after fetomaternal hemorrhage.
    • A positive anti-D antibody titer indicates potential Rh sensitization, risking fetal hemolytic disease.
    • For USMLE/NBME, remember that subsequent doses of RhoD immune globulin may be needed if there is a risk of re-sensitization.
  2. Illness script/Test-taking strategy:
    • Rh-negative mothers are at risk for sensitization when carrying an Rhpositive fetus; events like vaginal bleeding increase this risk.
    • Tie any evidence of potential Rh sensitization (even a weakly positive anti-D) to the need for RhoD immune globulin in Rh-negative pregnant women.
  3. Why other answers are wrong:
    • A. Repeat serum anti-D antibody titer: Not prevent potential Rh sensitization or complications.
    • B. Ultrasonography of the pelvis does not directly address Rh sensitization risk.
    • D. Amniocentesis: This invasive procedure is not indicated solely here
    • E. Induction of labor: Not indicated at 28 weeks without a compelling medical reason; Rh sensitization alone does not warrant premature delivery.

Test-taking tip: When dealing with Rh-negative mothers in USMLE, consider the risk of Rh sensitization and the protective role of RhoD immune globulin, especially after any event with potential fetomaternal hemorrhage.

Reference:

  • Le, T., Bhushan, V., & Sochat, M. (2023). First Aid for the USMLE Step 1 (p. 411). McGraw-Hill Education.
  • Le, T., Bhushan, V., Ascha, M., Bhardwaj, A., Boushra, M., & Griffin, D. (2023). First Aid for the USMLE Step 2 CK, eleventh edition (11th ed.). (p. 285). McGraw-Hill Education.

Question 43

A 32-year-old man comes to the office because of a 3-month history of a painless rash over his arms. During the past month, he also has had a 3.2-kg (7-Ib) weight loss despite no change in appetite………

Answer: C Kaposi sarcoma

  1. Why the correct answer is right:
    • Kaposi sarcoma (KS) presents a painless, violaceous, macular rash associated with Human Herpesvirus 8 (HHV-8).
    • KS can manifest in immunocompetent and immunocompromised individuals, but the latter is more common in the setting of HIV/AIDS.
    • For USMLE/NBME, a rash accompanied by systemic symptoms such as weight loss hints towards an underlying neoplasm or systemic disease, of which KS is a classic dermatological malignancy.
  2. Illness script/Test-taking strategy:
    • When presented with a violaceous rash and systemic symptoms in a relatively young adult, always consider Kaposi sarcoma.
    • Recognize the classical appearance of KS lesions to identify the diagnosis quickly.
  3. Why other answers are wrong:
    • A. Actinic keratosis: Precancerous lesions caused by sun exposure; presents as rough, scaly patches on sun-exposed skin; no links with weight loss.
    • B. Dermatitis herpetiformis: An intensely pruritic rash associated with celiac disease; clusters of itchy, blistered papules.
    • D. Lichen planus: Manifests as pruritic, polygonal, purple papules and plaques with a white reticular pattern (Wickham’s striae).
    • E. Seborrheic dermatitis: Present as scaly patches, red skin, and stubborn dandruff; not associated with weight loss.

Test-taking tip: On the USMLE, associating the clinical findings with the visual appearance of a condition can lead directly to the correct diagnosis. In this case, a violaceous rash in a young adult with systemic symptoms should quickly point to Kaposi sarcoma.

Reference:

  • Le, T., Bhushan, V., & Sochat, M. (2023). First Aid for the USMLE Step 1 (p. 486). McGraw-Hill Education.
  • Le, T., Bhushan, V., Ascha, M., Bhardwaj, A., Boushra, M., & Griffin, D. (2023). First Aid for the USMLE Step 2 CK, eleventh edition (11th ed.). (p. 121). McGraw-Hill Education.

Question 44

An 84-year-old woman comes to the office because of a 1-month history of malaise. She has not had fever, weight loss, or abdominal pain. She has type 2 diabetes mellitus well controlled………

Answer: B Elder neglect

  1. Why the correct answer is right:
    • The patient has deteriorating self-care (missed appointments, unkempt appearance, elevated glucose) started after her son moved in.
    • For USMLE/NBME, it is essential to recognize that elder neglect can manifest as worsening medical conditions despite no overt signs of physical abuse.
  2. Illness script/Test-taking strategy:
    • Elder neglect can manifest as failure to meet self-care needs, Lack of medical follow-up, and sudden decline in chronic medical conditions.
    • Consider socio-environmental causes for a sudden change in a stable patient’s condition.
  3. Why other answers are wrong:
    • A. Dietary indiscretion: This would not explain the unkempt appearance and missed appointments.
    • C. Major depressive disorder: Though she has a flat affect, there are no overt sadness, anhedonia, or other depressive symptoms.
    • D. Pancreatic malignancy: There is no weight loss, abdominal pain, or other related symptoms typical of pancreatic cancer.
    • E. Urinary tract infection: The urinalysis does not support this diagnosis as there is no nitrite or leukocyte esterase.

Test-taking tip: On the USMLE, consider social and environmental factors when seeing a drastic change in a patient’s health or behavior. In the elderly, neglect can be a subtle but significant reason for this change.

Reference:

  • Le, T., Bhushan, V., Ascha, M., Bhardwaj, A., Boushra, M., & Griffin, D. (2023). First Aid for the USMLE Step 2 CK, eleventh edition (11th ed.). (p. 193). McGraw-Hill Education.

Question 45

A 75-year-old man is admitted to the hospital because of a 3-day history of fever. He has type 2 diabetes mellitus, hypertension, and chronic obstructive pulmonary disease. One year ago, he had a stroke with residual right lower extremity hemiparesis………

Answer: A Inform the son that the decision is the patient’s to make, regardless of the medical team’s Recommendation

  1. Why the correct answer is right:
    • The patient is awake, alert, and conversing, demonstrating intact decision-making capacity.
    • A power of attorney (POA) is only activated when the patient cannot make decisions.
    • The patient’s wishes about his healthcare decisions should be respected as long as he has the capacity.
  2. Illness script/Test-taking strategy:
    • Decision-making capacity requires the ability to understand information, appreciate the situation and its consequences, reason logically, and communicate a choice.
    • Power of attorney does not override the decision of a competent patient.
    • The USMLE/NBME tests the principle of patient autonomy, especially when legal considerations are involved.
  3. Why other answers are wrong:
    • B. The patient’s wishes and rights to make decisions about his care supersede the physical therapist’s Recommendation when he has the capacity.
    • C. Family meetings can be useful but do not precede a competent patient’s decision.
    • D. No evidence in the vignette suggests the patient lacks mental competency; he is conversant and aware.
    • E. Reviewing the POA would not change the fact that a competent patient’s decision is paramount.

Test-taking tip: On the USMLE, always prioritize patient autonomy when decision-making capacity is intact, even with legal instruments like a power of attorney.


Question 46

A 41-year-old woman is being prepared for discharge from the hospital after treatment for acute pancreatitis. She has tolerated oral fluids well for the past 12 hours………

Answer: A Abstinence from alcoholic beverages

  1. Why the correct answer is right:
    • Alcohol consumption is a major risk factor for recurrent acute pancreatitis.
    • Abstaining from alcoholic beverages directly addresses the main etiological factor of her recurrent admissions.
    • For USMLE/NBME, note that alcohol use disorder significantly contributes to acute pancreatitis, and abstinence is the most effective way to prevent recurrences.
  2. Illness script/Test-taking strategy:
    • Recurrent pancreatitis in the context of alcohol use disorder suggests alcohol is the underlying cause.
    • When reducing the risk of a condition’s recurrence, target the primary risk factor, especially if it is modifiable.
  3. Why other answers are wrong:
    • B. Enrollment in a health insurance plan: While insurance can provide access to healthcare, it does not directly address the cause of her pancreatitis.
    • C. Nutritional counseling: Although diet can impact pancreatitis, it is secondary to alcohol in this patient’s context.
    • D. Placement in a women’s shelter: While it may provide her with secure housing, it does not directly mitigate the risk of alcohol-induced pancreatitis unless it offers substance abuse counseling.

Test-taking tip: On USMLE, focus on addressing the primary risk factors of diseases, especially when the clinical vignette provides clear etiological clues, as in this case with alcohol use disorder and pancreatitis.

Reference:

  • Le, T., Bhushan, V., & Sochat, M. (2023). First Aid for the USMLE Step 1 (p. 284). McGraw-Hill Education.
  • Le, T., Bhushan, V., Ascha, M., Bhardwaj, A., Boushra, M., & Griffin, D. (2023). First Aid for the USMLE Step 2 CK, eleventh edition (11th ed.). (p. 264, 265t, 613). McGraw-Hill Education.

Question 47

A 27-year-old man comes to the office because of a 1-month history of depressed mood and fatigue. He reports that it takes him at least 1 hour to fall asleep at night. He has had decreased concentration at work and decreased interest in socializing with his friends. He no longer exercises……….

Answer: E Mirtazapine

  1. Why the correct answer is right:
    • Mirtazapine is an atypical antidepressant effective for depression, plus being sedating, which may help his insomnia.
    • In patients with epilepsy, avoiding drugs that lower seizure threshold is crucial, making Mirtazapine a better choice.
    • For USMLE/NBME, note that Mirtazapine is especially helpful in depressed patients with insomnia, given its sedative properties.
  2. Illness script/Test-taking strategy:
    • The patient’s clinical picture suggests depression with insomnia, requiring an antidepressant that addresses sleep disturbances.
    • When choosing pharmacotherapy, consider comorbidities, such as epilepsy, which can guide or limit medication choices.
  3. Why other answers are wrong:
    • A. Alprazolam: This is a benzodiazepine primarily used for anxiety disorders and not a first-line treatment for depression.
    • B. Aripiprazole: An atypical antipsychotic used as an adjunct for resistant depression but not as a first-line.
    • C. Bupropion: An atypical antidepressant that is activating and can increase the risk of seizures; not ideal for epilepsy.
    • D. Buspirone: Primarily used for generalized anxiety disorder, not as a first-line treatment for depression.

Test-taking tip: On USMLE, patient history and comorbid conditions often suggest the best treatment choice. In this case, epilepsy directs away from certain antidepressants due to seizure risk.

Reference:

  • Le, T., Bhushan, V., & Sochat, M. (2023). First Aid for the USMLE Step 1 (p. 580, 596). McGraw-Hill Education.
  • Le, T., Bhushan, V., Ascha, M., Bhardwaj, A., Boushra, M., & Griffin, D. (2023). First Aid for the USMLE Step 2 CK, eleventh edition (11th ed.). (p. 603 – 607, 611t). McGraw-Hill Education.

Question 48

Patient Information Age: 6 years Sex Assigned at Birth: F Race/Ethnicity: unspecified Site of Care: emergency department History Reason for Visit/Chief Concern: “My daughter’s eyes and legs are swollen.” History of Present Illness: • 2-week history of cough, congestion, and runny nose that seems to be improving • parents report the patient’s eyes seemed swollen 4 days ago…………….

Answer B Serum albumin concentration

  1. Why the correct answer is right:
    • The patient’s presentation of periorbital edema, leg edema, and proteinuria suggests nephrotic syndrome, commonly associated with hypoalbuminemia.
    • Nephrotic syndrome is characterized by massive proteinuria, hypoalbuminemia, edema, and hyperlipidemia.
    • For USMLE/NBME, understand that decreased serum albumin in nephrotic syndrome leads to decreased oncotic pressure and edema.
  2. Illness script/Test-taking strategy:
    • Nephrotic syndrome in children often presents with puffiness around the eyes and swelling of the legs or ankles.
    • Link the clinical features of edema with the pathophysiology of decreased albumin leading to reduced oncotic pressure.
    • Remember that in children, minimal change disease is the most common cause of nephrotic syndrome and is characterized by hypoalbuminemia without significant inflammation or hematuria.
  3. Why other answers are wrong:
    • A. Hemoglobin: Nephrotic syndrome does not cause anemia directly. Hemoglobin would not necessarily be decreased.
    • C. Serum C3 and C4 concentrations: Decreased complement levels are seen in nephritic syndromes.
    • D. Serum triglycerides concentration: Triglycerides are increased, not decreased, in nephrotic syndrome due to altered lipid metabolism.
    • E. Serum urea nitrogen concentration: Urea levels are normal or elevated due to decreased renal perfusion or increased protein catabolism.

Test-taking tip: Consider nephrotic syndrome for pediatric patients with edema, especially periorbital and proteinuria, on the USMLE. In this context, hypoalbuminemia is a key laboratory finding that explains the edematous state.


Question 49

A 57-year-old man is brought to the emergency department by ambulance 30 minutes after he had loss of consciousness. His wife says he was making dinner when he fell on the kitchen floor. He has hypertension, and his only medication is lisinopril. He has no history of operative procedures. being irrigated……….

Answer: B Contact the organ bank for potential donation

  1. Why the correct answer is right:
    • For an unresponsive patient with brainstem findings consistent with brain death, organ donation can be considered.
    • With a ruptured cerebral aneurysm and absent brainstem reflexes, brain death is highly probable, making the patient potentially eligible for organ donation.
    • For USMLE/NBME, recognize that the absence of an advance directive does not preclude organ donation if there is knowledge of the patient’s wishes and family agreement.
  2. Illness script/Test-taking strategy:
    • A patient with signs of brainstem dysfunction (absent corneal reflex, absent eye movements) and radiographic evidence of significant brain injury should raise suspicion for brain death.
    • In organ donation scenarios, consider the clinical context and the patient’s or surrogate decision-makers expressed wishes.
  3. Why other answers are wrong:
    • A. Consult with the hospital ethics committee: The immediate step should be contacting the organ bank.
    • C. Determine if patients are awaiting organ donation in the hospital: The organ allocation process is managed by regional organ procurement organizations, not individual hospitals.
    • D. Lack of advance directive: Does not preclude organ donation, especially if the family or surrogate decision-maker is aware of the patient’s wishes.
    • E. History of hypertension: This does not automatically exclude a patient from being an organ donor.

Test-taking tip: On USMLE, when faced with complex ethical scenarios, always lean towards patient autonomy and previously expressed wishes when known, especially in organ donation.


Question 50

A 20-year-old woman comes to the office because of a 1-year history of a lump on her left ear that appeared 2 months after she had the ear pierced……….

Answer: A Corticosteroid injection

  1. Why the correct answer is right:
    • Keloids result from an overproduction of collagen during wound healing, and the most effective treatment for smaller keloids is intralesional corticosteroid injection.
    • Intralesional corticosteroid injections help reduce the size and itchiness of keloids by decreasing collagen synthesis and increasing collagenase activity.
    • For USMLE/NBME, remember that keloids often occur after ear piercings and can be distinguished from hypertrophic scars as they grow beyond the boundaries of the original wound.
  2. Illness script/Test-taking strategy:
    • A history of lump formation after ear piercing, especially in young adults, strongly suggests keloid formation.
    • Initial management choices on USMLE often favor conservative options that can be performed in an outpatient setting before considering more aggressive interventions.
  3. Why other answers are wrong:
    • B. Cryotherapy: More commonly used for warts or actinic keratoses.
    • C. Laser ablation: Reserved for cases that do not respond to conservative treatments.
    • D. Needle aspiration: Not used for keloids as they are solid and fibrous lesions.
    • E. Wide excision: Surgery can sometimes trigger larger keloids, especially without concurrent therapies.

Test-taking tip: On USMLE, when evaluating skin lesions or growths, consider both the clinical history and the physical characteristics. In the case of keloids, their growth beyond the wound and association with trauma (like piercings) are key features.

Reference:

  • Le, T., Bhushan, V., & Sochat, M. (2023). First Aid for the USMLE Step 1 (p. 214). McGraw-Hill Education.

Question 51

A 77-year-old man is brought to the emergency department because of a 1-hour history of severe, constant abdominal pain and dizziness. He has hypertension well controlled with hydrochlorothiazide. He has smoked two packs of cigarettes daily………

Answer: D Ruptured abdominal aortic aneurysm

  1. Why the correct answer is right:
    • The patient’s presentation of severe abdominal pain, hypotension, and long history of smoking suggests a ruptured abdominal aortic aneurysm (AAA).
    • An abdominal x-ray may show a curvilinear calcification outlining the aneurysmal wall or evidence of retroperitoneal hemorrhage.
    • For USMLE/NBME, key risk factors for AAA include advanced age, male gender, and tobacco use; the rupture can present with severe abdominal pain and hypotension.
  2. Illness script/Test-taking strategy:
    • A patient with significant risk factors (age, male, heavy smoking history) presenting with sudden abdominal pain and hypotension should raise suspicion for a ruptured AAA.
    • Clinical clues on the USMLE will typically lean heavily on presenting symptoms and risk factor assessment.
  3. Why other answers are wrong:
    • A. Acute pancreatitis: Presents with severe epigastric pain radiating to the back and associated with increased serum amylase or lipase.
    • B. Mesenteric ischemia: This may cause severe abdominal pain; the sudden onset of hypotension is less typical.
    • C. Perforated duodenal ulcer: Does not cause hypotension.
    • E. Small bowel obstruction: Not associated with the acute onset of hypotension.

Test-taking tip: On USMLE, always integrate the clinical presentation with imaging findings. Here, sudden abdominal pain in an elderly smoker with hypotension strongly suggests AAA rupture.

Reference:

  • Le, T., Bhushan, V., & Sochat, M. (2023). First Aid for the USMLE Step 1 (p. 305, 306). McGraw-Hill Education.
  • Le, T., Bhushan, V., Ascha, M., Bhardwaj, A., Boushra, M., & Griffin, D. (2023). First Aid for the USMLE Step 2 CK, eleventh edition (11th ed.). (p. 75 – 76, 226). McGraw-Hill Education.

Question 52

A 31-year-old man comes to the clinic for a health maintenance examination prior to employment. He feels well. He has no history of serious illness and takes no medications………

Answer: C DASH diet

  1. Why the correct answer is right:
    • The patient’s elevated blood pressure points toward stage 1 hypertension. Lifestyle interventions, such as the Dietary Approaches to Stop Hypertension (DASH) diet, are the first step in management.
    • Weight loss can reduce blood pressure, and the DASH diet can amplify this beneficial effect.
    • For USMLE/NBME, recognize that the DASH diet, which emphasizes fruits, vegetables, whole grains, poultry, fish, and low-fat dairy products, has been shown to reduce blood pressure and should be advised before drug therapy in mild cases.
  2. Illness script/Test-taking strategy:
    • Elevated blood pressure without end-organ damage or other cardiovascular risk factors is initially managed with lifestyle interventions.
    • On the USMLE, lifestyle modification recommendations often precede drug therapies unless there is a compelling indication.
  3. Why other answers are wrong:
    • A. ACE inhibitor therapy: Medications are typically reserved for stage 2 or 1 hypertension with evidence of end-organ damage or other cardiovascular risk factors.
    • B. Calcium supplementation: DASH diet is a more established recommendation.
    • D. Fish oil supplementation: Omega-3 fatty acids can have cardiovascular benefits but do not reduce blood pressure.
    • E. Thiazide diuretic therapy: Medications are reserved for higher stages of hypertension or evidence of end-organ damage.

Test-taking tip: On USMLE, lifestyle modifications like the DASH diet and weight loss are typically the first Recommendation for patients with early-stage hypertension and no end-organ damage.

Reference:

  • Le, T., Bhushan, V., & Sochat, M. (2023). First Aid for the USMLE Step 1 (p. 321). McGraw-Hill Education.
  • Le, T., Bhushan, V., Ascha, M., Bhardwaj, A., Boushra, M., & Griffin, D. (2023). First Aid for the USMLE Step 2 CK, eleventh edition (11th ed.). (p. 57 – 63). McGraw-Hill Education.

Question 53

Two days after admission to the hospital for treatment of a fractured femur and closed head injury sustained in a motor vehicle collision, an 8-year-old girl has a brief, generalized tonic-clonic seizure. CT scan of the head on admission disclosed no abnormalities………

Answer: A Administer a bolus of intravenous 3% saline

  1. Why the correct answer is right:
    • The patient’s symptoms of generalized tonic-clonic seizure and serum studies indicate severe hyponatremia (Na<125 mEq/L).
    • Rapid correction with hypertonic saline (3% saline) is essential when symptoms are severe, especially when they include seizures.
    • For the USMLE/NBME, remember that severe, symptomatic hyponatremia necessitates immediate treatment to prevent irreversible neurological damage.
  2. Illness script/Test-taking strategy:
    • Severe hyponatremia can cause non-specific symptoms like nausea, malaise, and headache, progressing to seizures, coma, or brain herniation in acute and severe cases.
    • Always correlate the clinical picture with lab findings; in this case, the low serum sodium combined with the recent seizure.
  3. Why other answers are wrong:
    • B. Subcutaneous vasopressin: Used for central diabetes insipidus, not for symptomatic hyponatremia.
    • C. Initiate fosphenytoin therapy: It is an antiepileptic; the cause of the seizure is hyponatremia.
    • D. 0.9% saline at 1.5 times the maintenance: Isotonic saline might not raise sodium levels quickly enough for symptomatic hyponatremia.
    • E. Order EEG: Not the priority when the cause of the seizure is evident.
    • F. Order MRI of the brain: Not necessary when the seizure’s cause is clear from the lab findings.

Test-taking tip: On the USMLE, always prioritize the most immediate life-threatening issue. In this scenario, symptomatic hyponatremia is the primary concern, and its rapid correction takes precedence.

Reference:

  • Le, T., Bhushan, V., & Sochat, M. (2023). First Aid for the USMLE Step 1 (p. 342, 591). McGraw-Hill Education.
  • Le, T., Bhushan, V., Ascha, M., Bhardwaj, A., Boushra, M., & Griffin, D. (2023). First Aid for the USMLE Step 2 CK, eleventh edition (11th ed.). (p. 676, 678). McGraw-Hill Education.

Question 54

A 23-year-old man comes to the emergency department because of a 1-hour history of intermittent coughing spasms that began suddenly while he was attempting to clean mold from his garage with a mixture of acetic acid and cleaning agents. The cough is not productive………..

Answer: A Albuterol therapy

  1. Why the correct answer is right:
    • The patient’s symptoms of coughing spasms, wheezing, and shortness of breath after cleaning with chemicals suggest a reactive airway event or chemical-induced bronchospasm.
    • Albuterol, a short-acting beta-2 agonist, provides rapid bronchodilation, making it the first-line treatment for acute bronchospasm.
    • For the USMLE/NBME, always associate sudden onset wheezing following exposure to irritants or allergens with bronchospasm, and remember that the first-line treatment is bronchodilators like albuterol.
  2. Illness script/Test-taking strategy:
    • Chemical exposures, especially in a confined space like a garage, can lead to inhalation injury manifesting as cough and wheezing.
    • Given the acute onset after a known exposure and the physical exam findings, the diagnosis is likely bronchospasm.
  3. Why other answers are wrong:
    • B. Bronchoscopy: Not indicated as first-line.
    • C. CT scan of the chest: Not immediately required since chest x-ray only shows hyper expansion.
    • D. Intubation: The patient is not in severe respiratory distress, so this invasive step is not warranted.
    • E. Methylprednisolone therapy: Not the first-line immediate treatment for acute bronchospasm.

Test-taking tip: On the USMLE, when faced with a patient exhibiting signs of acute bronchospasm, especially after exposure to potential irritants, consider bronchodilators before considering more invasive or aggressive treatments.

Reference:

  • Le, T., Bhushan, V., & Sochat, M. (2023). First Aid for the USMLE Step 1 (p. 241, 708). McGraw-Hill Education.

Question 55

A 50-year-old man comes to the office because of a 3-day history of mild headache and intermittent confusion. He has a 30-year history of schizophrenia and a 5-year history of emphysema. Medications are clozapine, sertraline, and inhaled beclomethasone and salmeterol………

Answer: D Measurement of urine and plasma osmolarity

  1. Why the correct answer is right:
    • The patient’s presentation includes confusion and a low serum sodium level, indicating hyponatremia, which requires differentiation between the various etiologies.
    • Measuring urine osmolarity helps to differentiate between hypo-osmolar (dilutional) hyponatremia and other causes, while plasma osmolarity further specifies the type of hyponatremia.
    • For USMLE/NBME, assessing urine and plasma osmolarity is the next logical step in evaluating the cause of hyponatremia.
  2. Illness script/Test-taking strategy:
    • Hyponatremia can manifest with non-specific CNS symptoms like headache, confusion, or even seizures in severe cases.
    • Determining whether hyponatremia is due to water retention, salt loss, or a mix of both is essential.
  3. Why other answers are wrong:
    • A. CT scan of the chest: This patient’s confusion is more likely related to hyponatremia.
    • B. Measurement of serum cortisol and TSH concentrations: The patient lacks specific signs of adrenal insufficiency and hypothyroidism.
    • C. Measurement of serum uric acid concentration: Not directly relevant to the evaluation of hyponatremia.
    • E. MRI of the brain: Neurologic symptoms are more likely related to the electrolyte imbalance.

Test-taking tip: On the USMLE, when faced with a patient with neurologic symptoms and confirmed hyponatremia, focus on determining the type and cause of hyponatremia before pursuing other potential etiologies for the neurologic symptoms.

Reference:

  • Le, T., Bhushan, V., & Sochat, M. (2023). First Aid for the USMLE Step 1 (p. 342, 591). McGraw-Hill Education.
  • Le, T., Bhushan, V., Ascha, M., Bhardwaj, A., Boushra, M., & Griffin, D. (2023). First Aid for the USMLE Step 2 CK, eleventh edition (11th ed.). (p. 676, 678). McGraw-Hill Education.

Question 56

A 47-year-old man comes to the office for a routine examination before beginning an exercise program. He feels well. He has a 25-year history of type 1 diabetes mellitus. A grade 1/6, early peaking, systolic ejection murmur is heard best at the left third intercostal space………..

Answer: C. Coronary artery disease

  1. Why the correct answer is right:
    • The exercise stress test reveals a 3-mm ST-segment depression, which is suggestive of myocardial ischemia, indicating the presence of coronary artery disease (CAD).
    • Patients with longstanding type 1 diabetes mellitus, like this individual, are at an increased risk for atherosclerotic disease, including CAD.
    • On the USMLE/NBME, connect longstanding diabetes with an increased risk of macrovascular complications like CAD, especially when presented with suggestive findings.
  2. Illness script/Test-taking strategy:
    • In a diabetic patient, even in the absence of chest pain (due to potential diabetic neuropathy causing silent myocardial ischemia), significant ST-segment changes on a stress test strongly point towards CAD.
    • Always correlate the risk factors (in this case, longstanding diabetes) with the clinical presentation and investigative findings.
  3. Why other answers are wrong:
    • A. Cardiomyopathy: No signs of heart failure, and the murmur does not fit the cardiomyopathy presentation.
    • B. Congenital heart disease: The murmur and the patient’s age make them less likely.
    • D. Valvular heart disease: It does not explain the ST-segment changes.
    • E. Normal cardiac findings: The ST-segment depression is abnormal and suggests underlying ischemia.

Test-taking tip: In the USMLE, when a diabetic patient presents with suggestive findings on an exercise stress test, always consider coronary artery disease as a significant differential, even in the absence of typical anginal symptoms.

Reference:

  • Le, T., Bhushan, V., & Sochat, M. (2023). First Aid for the USMLE Step 1 (p. 305, 308). McGraw-Hill Education.
  • Le, T., Bhushan, V., Ascha, M., Bhardwaj, A., Boushra, M., & Griffin, D. (2023). First Aid for the USMLE Step 2 CK, eleventh edition (11th ed.). (p. 47 – 49). McGraw-Hill Education.

Question 57

A community has created standard-of-care guidelines for ambulatory patients with several common diagnoses. The community’s standard of care for patients with asthma is annual pulmonary function testing. Recent analysis of billing records from one clinic in this community shows that only 34% of patients with asthma at the clinic…………….

Answer: B Diagnosis-driven reminders in patient charts

  1. Why the correct answer is right:
    • Diagnosis-driven reminders in patient charts directly alert the healthcare provider to necessary care steps when the patient is seen.
    • Automated systems (like electronic medical record reminders) targeting specific diagnoses can effectively close care gaps.
    • For USMLE/NBME, always consider interventions that prompt the physician at the point of care as effective measures to improve adherence.
  2. Illness script/Test-taking strategy:
    • Diagnosis-driven reminders are direct, immediate, and actionable, making them efficient in improving the standard of care adherence.
    • Relate electronic health records or chart prompts to improved clinical care outcomes in USMLE scenarios.
  3. Why other answers are wrong:
    • A. Annual chart reviews and feedback: Time-consuming, and feedback may be too delayed to effect change.
    • C. Flyers in waiting room: Patients might not act on them or may forget; they are not provider-directed.
    • D. Testing all patients with respiratory symptoms: Over-testing and not all asthma-related respiratory symptoms.
    • E. Testing post-exacerbation: Too narrow a criterion; misses routine asthma patients who need the test annually.

Test-taking tip: Regarding quality improvement or care standard adherence on the USMLE, solutions that directly influence the provider’s decision-making at the point of care often emerge as the most effective interventions.


Question 58

A 56-year-old man comes to the clinic with his wife because of a 6-month history of restlessness while sleeping. The wife says he motions as if he were throwing or kicking a ball and sometimes punches the pillow during the night; he has hit her twice. Recently, he began talking in his sleep and yelling expletives…………

Answer: E Parkinson’s disease

  1. Why the correct answer is right:
    • The patient’s symptoms describe REM sleep behavior disorder, characterized by acting out vivid, often violent dreams, a risk factor for neurodegenerative disorders.
    • The patient also has constipation, another non-motor symptom often preceding the motor symptoms of Parkinson’s disease.
    • For USMLE/NBME, understand that REM sleep behavior disorder is most strongly associated with the development of synucleinopathies, notably Parkinson’s disease.
  2. Illness script/Test-taking strategy:
    • Recognize REM sleep behavior disorder by the presence of violent dream enactment without paralysis during REM sleep.
    • On the USMLE, when given a scenario of a middle-aged or older patient acting out dreams violently, think of its association with Parkinson’s disease.
  3. Why other answers are wrong:
    • A. Alzheimer’s disease: Although dementia can coexist, REM sleep behavior disorder has a stronger link with Parkinson’s.
    • B. Amyotrophic lateral sclerosis: Not typically associated with REM sleep behavior disorder.
    • C. Cerebral infarction: No direct association with the patient’s symptoms.
    • D. Narcolepsy: Presents with excessive daytime sleepiness, cataplexy, sleep paralysis, and hypnagogic hallucinations but not violent dream enactment.

Test-taking tip: Recognize that certain sleep disorders can be prodromal symptoms of neurodegenerative diseases. REM sleep behavior disorder, in particular, is a big red flag for Parkinson’s disease and other synucleinopathies on the USMLE.

Reference:

  • Le, T., Bhushan, V., & Sochat, M. (2023). First Aid for the USMLE Step 1 (p. 508, 587). McGraw-Hill Education.
  • Le, T., Bhushan, V., Ascha, M., Bhardwaj, A., Boushra, M., & Griffin, D. (2023). First Aid for the USMLE Step 2 CK, eleventh edition (11th ed.). (p. 617 – 619). McGraw-Hill Education.

Question 59

A 21-year-old man comes to student health services because of a 6-month history of increasingly frequent episodes of moderate chest pain. The first episode occurred while he was sitting in traffic and feeling stressed because he was late for a college class. At that time, he had the sudden onset of moderate chest pain, a rapid heartbeat, sweating, and nausea. He says he felt as though he were going to die…………

Answer: A Agoraphobia

  1. Why the correct answer is right:
    • This patient’s sudden intense fear and somatic symptoms, such as rapid heartbeat and sweating, which last for a short duration, are characteristic of panic attacks.
    • For USMLE/NBME, understand that when patients start avoiding places or situations due to fear of these attacks, it becomes agoraphobia.
    • He avoids social activities and spending time outside, fearing another attack, which indicates agoraphobia.
  2. Illness script/Test-taking strategy:
    • Panic attacks are acute episodes of intense fear, while agoraphobia is the avoidance behavior stemming from the fear of having an attack in a situation where escape might be difficult.
    • When encountering a USMLE question describing a patient with panic attacks who avoids situations due to fear of recurrence, lean towards agoraphobia.
  3. Why other answers are wrong:
    • B. Generalized anxiety disorder: Characterized by chronic worry about multiple areas of life, not specific panic attacks.
    • C. Illness anxiety disorder: Excessive worry about having a serious illness, not fear of panic attacks.
    • D. Social anxiety disorder: Intense fear of social situations where others might scrutinize one.
    • E. Somatic symptom disorder: Physical symptoms causing distress with excessive thoughts, feelings, or behaviors.

Test-taking tip: For USMLE questions on anxiety disorders, focus on the specifics of the patient’s fear and behavior. Avoiding places due to the fear of panic attacks is key to diagnosing agoraphobia.

Reference:

  • Le, T., Bhushan, V., & Sochat, M. (2023). First Aid for the USMLE Step 1 (p. 582). McGraw-Hill Education.
  • Le, T., Bhushan, V., Ascha, M., Bhardwaj, A., Boushra, M., & Griffin, D. (2023). First Aid for the USMLE Step 2 CK, eleventh edition (11th ed.). (p. 595 – 596). McGraw-Hill Education.

Question 60

A 25-year-old woman, gravida 3, para 2, at 36 weeks’ gestation, comes to the emergency department because of heavy vaginal bleeding following sexual intercourse 3 hours ago. She has received no prenatal care. She has no history of serious illness or operative procedures. Vital signs are within normal limits………..

Answer: E Transabdominal ultrasonography

  1. Why the correct answer is right:
    • In a patient with late-pregnancy vaginal bleeding, placenta previa is a key differential.
    • It is essential to determine the location of the placenta before considering any digital examination or manipulation.
    • For USMLE/NBME, understand that transabdominal Ultrasonography is the first step to diagnose or rule out placenta previa when there is unexplained third-trimester bleeding.
  2. Illness script/Test-taking strategy:
    • A pregnant patient with painless, bright red vaginal bleeding, especially after intercourse, in the third trimester should raise suspicion for placenta previa.
    • For USMLE scenarios, the absence of pain differentiates placenta previa from other conditions like placental abruption.
  3. Why other answers are wrong:
    • A. Cesarean delivery now: A diagnosis should be made first; premature cesarean might not be necessary
    • B. Digital examination of the cervix: Contraindicated initially in unexplained third-trimester bleeding because of the risk of provoking massive hemorrhage in the placenta previa.
    • C. Magnesium sulfate: Used for preeclampsia or tocolysis, not relevant here.
    • D. Oxytocin: Used to induce labor, not indicated without a diagnosis.

Test-taking tip:

When presented with a third-trimester pregnant patient having unexplained vaginal bleeding, always think of placenta previa and avoid any invasive procedures until it is ruled out using ultrasound.

Reference:

  • Le, T., Bhushan, V., & Sochat, M. (2023). First Aid for the USMLE Step 1 (p. 659). McGraw-Hill Education.
  • Le, T., Bhushan, V., Ascha, M., Bhardwaj, A., Boushra, M., & Griffin, D. (2023). First Aid for the USMLE Step 2 CK, eleventh edition (11th ed.). (p. 447t, 448f). McGraw-Hill Education.

Question 61

A 13-year-old girl is brought to the office for evaluation of short stature. Since birth, she has been below the 3rd percentile for height and at the 3rd percentile for weight. She has never had a menstrual period………

Answer: C X-rays of the left hand and wrist

  1. Why the correct answer is right:
    • The girl’s delay in growth and puberty suggests a possible bone age delay.
    • A delayed bone age compared to chronological age indicates potential for future growth with delayed pubertal development.
    • For USMLE/NBME, it is essential to recognize that X-rays of the left hand and wrist (bone age study) are used to assess skeletal maturity and predict future growth potential.
  2. Illness script/Test-taking strategy:
    • Delayed growth and puberty with a family history of late bloomers suggest constitutional growth delay.
    • Always relate clinical features (short stature + no menstruation + delayed breast development) to the most appropriate diagnostic step: bone age assessment.
  3. Why other answers are wrong:
    • A. Complete blood count and serum chemistry panel: Useful for metabolic issues but not specific for evaluating growth delay.
    • B. Measurement of urine-free cortisol concentration: Pertains to Cushing’s syndrome, not described here.
    • D. X-rays of the long bones and spine: More related to evaluating skeletal dysplasias or pathologies, not to assessing growth potential.
    • E. X-ray of the skull: This would evaluate cranial abnormalities irrelevant to the presented issue.

Test-taking tip:

For USMLE, when faced with growth and puberty delay, consider constitutional growth delay and use family history as a clue. Bone age assessment is key in differentiating pathologic growth disorders from constitutional growth delay.

Reference:

  • Le, T., Bhushan, V., & Sochat, M. (2023). First Aid for the USMLE Step 1 (p. 332, 333). McGraw-Hill Education.
  • Le, T., Bhushan, V., Ascha, M., Bhardwaj, A., Boushra, M., & Griffin, D. (2023). First Aid for the USMLE Step 2 CK, eleventh edition (11th ed.). (p. 471, 472t, 512). McGraw-Hill Education.

Question 62

A 37-year-old man comes to the office because of a 2-year history of mild to moderate fatigue. The fatigue waxes and wanes without relation to activity. He has no history of serious illness and takes no medications. He drinks three glasses of whiskey weekly…………….

Answer: C Hepatocellular carcinoma

  1. Why the correct answer is right:
    • The laboratory findings indicate chronic hepatitis B infection (HBsAg positive and IgG anti-HBc positive).
    • The patient’s longstanding fatigue and liver enzyme elevations corroborate liver pathology, likely secondary to hepatitis B.
    • For USMLE/NBME, recognize that chronic hepatitis B infection is a significant risk factor for developing hepatocellular carcinoma (HCC).
  2. Illness script/Test-taking strategy:
    • Chronic hepatitis B can present subtly with fatigue and elevated liver enzymes.
    • Associate hepatitis B, especially in endemic regions like China, with the risk of hepatocellular carcinoma.
  3. Why other answers are wrong:
    • A. Amyloidosis: Not directly related to hepatitis B.
    • B. Essential mixed cryoglobulinemia: More associated with hepatitis C than hepatitis B.
    • D. Membranoproliferative glomerulonephritis: While it can be related to hepatitis B, it does not carry the same high risk as HCC.
    • E. Polyarteritis nodosa: Though sometimes linked to hepatitis B, it is not as common as HCC in this context.
    • F. Sjögren syndrome: Not related to hepatitis B and presents with different clinical features.

Test-taking tip:

When encountering liver pathology in the context of positive HBsAg, especially in patients from endemic regions, always consider the long-term complications of hepatitis B. Hepatocellular carcinoma should be top of mind.

Reference:

  • Le, T., Bhushan, V., & Sochat, M. (2023). First Aid for the USMLE Step 1 (p. 171, 172). McGraw-Hill Education.
  • Le, T., Bhushan, V., Ascha, M., Bhardwaj, A., Boushra, M., & Griffin, D. (2023). First Aid for the USMLE Step 2 CK, eleventh edition (11th ed.). (p. 249 – 251). McGraw-Hill Education.

Question 63

A hospitalized 37-year-old woman has a 2- hour history of severe chest pain and blood1. streaked sputum 4 days after undergoing a hysterectomy and oophorectomy for ovarian cancer. Current medications are oxycodone, prophylactic low-molecularweight heparin, and a multivitamin…….

Answer: H Pulmonary infarction

  1. Why the correct answer is right:
    • The patient’s presentation of pleuritic chest pain, hemoptysis, recent surgery, and pleural-based triangular density (Hampton’s hump) on imaging suggest pulmonary infarction.
    • For USMLE/NBME, be aware that deep vein thrombosis (DVT) is the common precursor for pulmonary embolism (PE), but it is not always detectable
    • Pleuritic chest pain combined with the described radiographic finding following a surgical procedure should lead to suspicion for PE and subsequent infarction.
  2. Illness script/Test-taking strategy:
    • Recognize that major surgery is a significant risk factor for PE.
    • While PE may not always cause noticeable symptoms, subsequent pulmonary infarction is characterized by localized pain and signs of lung tissue damage.
  3. Why other answers are wrong:
    • A. Air embolism: Usually seen immediately after procedures and not days later, it can lead to sudden cardiovascular collapse.
    • B. Empyema: Would expect fever and purulent sputum, not blood-streaked.
    • C. Hemothorax: Would cause massive hemoptysis and shock, not localized pain.
    • D. Pericarditis: The x-ray finding does not correlate, and there is no mention of pericardial friction rub.
    • E. Pneumonia: Would expect fever, productive cough, and other systemic symptoms.
    • F. Pulmonary hemorrhage: Typically results from severe inflammation, and there is no supportive history here.
    • G. Pulmonary hypertension: A chronic condition that would not cause acute chest pain.

Test-taking tip:

In USMLE, remember that post-operative patients have a heightened risk for thromboembolic events. Even if a DVT is not found, the clinical scenario can point towards PE and related complications like infarction. Always link the clinical presentation with recent significant events or procedures.

Reference:

  • Le, T., Bhushan, V., & Sochat, M. (2023). First Aid for the USMLE Step 1 (p. 300, 693). McGraw-Hill Education.
  • Le, T., Bhushan, V., Ascha, M., Bhardwaj, A., Boushra, M., & Griffin, D. (2023). First Aid for the USMLE Step 2 CK, eleventh edition (11th ed.). (p. 644 – 645). McGraw-Hill Education.

Question 64

A 68-year-old man comes to the office because of a 3-month history of moderate low back pain that extends into both legs. He first noticed the pain while standing in the ticket line at a theater. At that time, the pain was accompanied by tingling in both legs. The symptoms recurred a few days later as he was standing in the grocery checkout line…..

Answer: D Spinal stenosis

  1. Why the correct answer is right:
    • The patient’s symptoms of back pain radiating into both legs, exacerbated by standing or walking, and relieved by sitting or flexing the spine are typical for neurogenic claudication due to lumbar spinal stenosis.
    • Age and degenerative changes are common causes of lumbar spinal stenosis.
    • For USMLE/NBME, understand that spinal stenosis presents as pain or tingling in the lower extremities, worsened by lumbar extension (e.g., standing or walking) and relieved by lumbar flexion (e.g., sitting).
  2. Illness script/Test-taking strategy:
    • Always differentiate between neurogenic (spinal stenosis) and vascular claudication; the relief with flexion and the pain with prolonged standing or walking are hallmarks of the former.
    • Recognize that the onset of symptoms during activities involving extended standing or walking is a clue for spinal stenosis.
  3. Why other answers are wrong:
    • A. Abdominal aortic aneurysm: Typically asymptomatic; does not explain leg symptoms.
    • B. Osteoporosis: Can cause back pain but would not cause leg pain/tingling.
    • C. Peripheral neuropathy: Causes distal symmetric pain/numbness, not positional.
    • E. Vascular claudication: Pain is relieved by rest and is not positiondependent. Also, the ankle-brachial index is normal.

Test-taking tip:

On USMLE, differentiate neurogenic from vascular claudication by noting the circumstances of pain onset and relief. Spinal stenosis pain is relieved by lumbar flexion, whereas vascular claudication is relieved simply by rest.

Reference:

  • Le, T., Bhushan, V., Ascha, M., Bhardwaj, A., Boushra, M., & Griffin, D. (2023). First Aid for the USMLE Step 2 CK, eleventh edition (11th ed.). (p. 349 – 350). McGraw-Hill Education.

Question 65

A 27-year-old woman, gravida 3, para 2, at 12 weeks’ gestation comes to the office for her first prenatal visit. Her first child was delivered at 41 weeks’ gestation and weighed 4167 g (9 lb 3 oz). Her second child was delivered at 37 weeks’ gestation and weighed 4309 g (9 lb 8 oz). She has no history of serious illness…………….

Answer: C Fasting glucose tolerance test

  1. Why the correct answer is right:
    • Given the patient’s history of delivering large-for-gestational-age infants, her obesity (BMI of 38 kg/m^2), and a family history of type 2 diabetes, she is at an increased risk for gestational diabetes mellitus (GDM).
    • For USMLE/NBME, it is vital to recognize that women with risk factors for GDM should be screened at their first prenatal visit.
    • GDM screening typically involves an initial glucose challenge, followed, if abnormal, by a more definitive glucose tolerance test.
  2. Illness script/Test-taking strategy:
    • Key triggers for early GDM testing are prior GDM, obesity, prior large babies, and strong family history of diabetes.
    • Be wary of the ‘typical’ screening time (24-28 weeks) for GDM; highrisk patients like this one should be screened earlier.
  3. Why other answers are wrong:
    • A. Antiphospholipid antibody screening: Indicated with recurrent unexplained miscarriages, not stillbirth alone.
    • B. Chest x-ray: No indication of her presentation.
    • D. Measurement of serum α-fetoprotein: Used for neural tube defect screening, typically between 15-20 weeks’ gestation.
    • E. Measurement of serum-free thyroxine: Only indicated if there is a suspicion of thyroid dysfunction; no such evidence here.

Test-taking tip:

In USMLE, always weigh risk factors for conditions that have standard screening timelines. High-risk patients may need earlier or more aggressive testing.

Reference:

  • Le, T., Bhushan, V., & Sochat, M. (2023). First Aid for the USMLE Step 1 (p. 73, 654). McGraw-Hill Education.
  • Le, T., Bhushan, V., Ascha, M., Bhardwaj, A., Boushra, M., & Griffin, D. (2023). First Aid for the USMLE Step 2 CK, eleventh edition (11th ed.). (p. 439 – 441). McGraw-Hill Education.

Question 66

A 22-year-old man comes to the office for a health maintenance examination. He feels well and has not noticed any health issues. He is in the US Army. Two months ago, he returned from a 10-month deployment to Southwest Asia, during which he started smoking one pack of cigarettes daily. The patient says he is happy to be home…………..

Answer: C Excessive alcohol use

  1. Why the correct answer is right:
    • The patient’s consumption of an average of six beers daily is indicative of excessive alcohol use, which is a well-known reversible cause of hypertension.
    • The elevation in blood pressure may return to baseline upon reducing alcohol intake.
    • For USMLE/NBME, understand that chronic excessive alcohol consumption, defined as more than two drinks per day for men and more than one drink per day for women, can cause elevated blood pressure.
  2. Illness script/Test-taking strategy:
    • Recognize lifestyle factors that can contribute to hypertension. Excessive alcohol use is a major modifiable risk factor.
    • Always consider secondary causes of hypertension in young adults, especially when they present without other risk factors for primary hypertension.
  3. Why other answers are wrong:
    • A. Cigarette smoking: Not typically associated with chronic hypertension.
    • B. Essential hypertension: While common, primary or essential hypertension is usually a diagnosis of exclusion.
    • D. Pheochromocytoma: Typically presents with episodic hypertension, headache, sweating, and tachycardia.
    • E. Renal artery stenosis: Should be considered in younger patients with hypertension.

Test-taking tip:

For young patients with newly identified hypertension on USMLE, always explore secondary causes before settling on a diagnosis of essential hypertension. The patient’s history can provide crucial clues.

Reference:

  • Le, T., Bhushan, V., & Sochat, M. (2023). First Aid for the USMLE Step 1 (p. 304). McGraw-Hill Education.
  • Le, T., Bhushan, V., Ascha, M., Bhardwaj, A., Boushra, M., & Griffin, D. (2023). First Aid for the USMLE Step 2 CK, eleventh edition (11th ed.). (p. 57 – 59). McGraw-Hill Education.

Question 67

A 28-year-old man comes to the clinic because of a 2-day history of a red, severely painful lesion on his left foreal He says that he felt a stinging pain as he put his shirt on in the dark two days ago; the pain was so severe he immediately threw down the shirt and saw a spider running across the floor………..

Answer: A Areas of eschar at the center of the wound site

  1. Why the correct answer is right:
    • The patient’s presentation suggests a bite from the spider, which can lead to necrotic arachnidism.
    • The characteristic development of an eschar at the center of the wound site often follows the initial reaction and represents tissue necrosis.
    • For USMLE/NBME, recognize that brown recluse spider bites can lead to local tissue necrosis due to the venom’s enzymes.
  2. Illness script/Test-taking strategy:
    • In the context of a suspected spider bite in a region known for the presence of brown recluse spiders, expect a progression of local skin changes, including the development of an eschar.
    • Focus on the location and characteristic evolution of the bite site when considering a brown recluse spider bite.
  3. Why other answers are wrong:
    • B. Gangrene of the distal aspect: While tissue necrosis can occur, it is typically localized.
    • C. Generalized upper extremity edema: This would indicate a more systemic reaction or a significant infection.
    • D. Palpable purpuric lesions: This can be seen in vasculitis or certain infections.
    • E. Rapidly spreading erythema: This suggests bacterial cellulitis or a more aggressive infection.
    • F. Tender, erythematous streaks: Indicative of lymphangitis, typically seen with bacterial infections.

Test-taking tip:

On USMLE, focus on the regional and environmental clues provided in the vignette. The geographic location and the bite description can lead to the right diagnosis.

Reference:

  • Le, T., Bhushan, V., & Sochat, M. (2023). First Aid for the USMLE Step 1 (p. 111). McGraw-Hill Education.
  • Le, T., Bhushan, V., Ascha, M., Bhardwaj, A., Boushra, M., & Griffin, D. (2023). First Aid for the USMLE Step 2 CK, eleventh edition (11th ed.). (p. 343 – 344). McGraw-Hill Education.

Question 68

A 19-year-old man is brought to the emergency department (ED) by ambulance 15 minutes after he was stabbed in the chest during a fight. The stab wound is located medially to the left nipple………..

Answer: D Pericardiocentesis

  1. Why the correct answer is right:
    • The patient presents with cardiac tamponade. This is due to blood accumulation in the pericardial space from the injury, leading to decreased ventricular filling and cardiac output.
    • Immediate treatment involves pericardiocentesis to relieve the pressure and improve cardiac output.
    • The FAST (focused assessment with sonography for trauma) examination would most likely reveal pericardial fluid in penetrating chest trauma.
  2. Illness script/Test-taking strategy:
    • Recognize the classic triad of Beck’s triad (hypotension, muffled heart sounds, and jugular venous distention) in the context of penetrating chest trauma to suspect cardiac tamponade.
    • The FAST exam is crucial for rapid diagnosis in trauma patients, especially when suspecting intra-abdominal or pericardial fluid.
  3. Why other answers are wrong:
    • A. CT scan of the chest and abdomen: Not the initial choice in an unstable trauma patient.
    • B. Exploratory laparotomy: Addresses intra-abdominal injuries, but the patient’s symptoms suggest a cardiac issue.
    • C. Needle decompression and chest tube placement: This treatment is for tension pneumothorax.
    • E. Peritoneal lavage: An older method for evaluating intra-abdominal injuries but has been largely replaced by FAST in trauma settings.

Test-taking tip:

When approaching trauma scenarios in USMLE, always prioritize the ABCs (Airway, Breathing, and Circulation). Given the patient’s hemodynamic instability and the clinical picture suggesting cardiac tamponade, immediate intervention (pericardiocentesis) is critical.

Reference:

  • Le, T., Bhushan, V., & Sochat, M. (2023). First Aid for the USMLE Step 1 (p. 317). McGraw-Hill Education.
  • Le, T., Bhushan, V., Ascha, M., Bhardwaj, A., Boushra, M., & Griffin, D. (2023). First Aid for the USMLE Step 2 CK, eleventh edition (11th ed.). (p. 67, 753). McGraw-Hill Education.

Question 69

Two weeks after undergoing open cholecystectomy for gangrenous cholecystitis, a 47-year-old woman comes to the clinic for the removal of surgical staples. She was treated with oral vancomycin for Clostridioides difficile colitis during hospitalization. She has type 2 diabetes mellitus…………..

Answer: B Washing hands thoroughly with soap and water

  1. Why the correct answer is right:
    • Clostridioides difficile (formerly Clostridium difficile) spores resist alcohol-based hand sanitizers.
    • Patients with a history of C. difficile infection, especially recently, should prompt providers to wash hands with soap and water to remove spores physically
    • Hand hygiene with soap and water is preferred for C. difficile prevention when hands are visibly soiled.
  2. Illness script/Test-taking strategy:
    • C. difficile infection is associated with antibiotic use and results in diarrhea, often recurrent.
    • Consider C. difficile in post-operative patients with diarrhea, especially after antibiotic use.
    • Focus on describing the patient’s symptoms and recent history to recognize the red flag for C. difficile.
  3. Why other answers are wrong:
    • A. Alcohol gel: Not effective against C. difficile spores.
    • C. Surgical mask: C. difficile is not transmitted through respiratory droplets.
    • D. Sterile gloves and chlorhexidine: The concern is about C. difficile transmission, not a surgical site infection.
    • E. No additional precautions: Given the patient’s history of C. difficile, precautions are necessary.

Test-taking tip:

On the USMLE, when dealing with infection prevention or control, always consider the mode of transmission of the infectious agent. For C. difficile, the concern is spore transmission, which mandates soap and water hand hygiene.

Reference:

  • Le, T., Bhushan, V., & Sochat, M. (2023). First Aid for the USMLE Step 1 (p. 182). McGraw-Hill Education.
  • Le, T., Bhushan, V., Ascha, M., Bhardwaj, A., Boushra, M., & Griffin, D. (2023). First Aid for the USMLE Step 2 CK, eleventh edition (11th ed.). (p. 231 – 232). McGraw-Hill Education.

Question 70

An 18-month-old boy is brought to the emergency department by his mother 30 minutes after he fell from his bed onto the floor. Two months ago, he sustained a fracture of the right humerus when he fell while playing in the park…………….

Answer: A Audiography

  1. Why the correct answer is right:
    • The child’s recurrent fractures with minimal trauma and bluish sclera suggest Osteogenesis Imperfecta (OI), a collagen synthesis disorder.
    • One of the common complications of OI is sensorineural hearing loss, often beginning in early childhood or adolescence.
    • Regular audiography is recommended to detect and manage hearing loss early in patients with OI.
  2. Illness script/Test-taking strategy:
    • OI is a hereditary disorder with key features including increased bone fragility, bluish sclera, and hearing loss.
    • When presented with a pediatric patient with recurrent fractures and bluish sclera, think of OI and its associated complications.
    • In clinical scenarios, always correlate the findings to known complications of the described condition.
  3. Why other answers are wrong:
    • B. DEXA scan: While OI affects bone strength, DEXA is mainly used for diagnosing osteoporosis in postmenopausal women.
    • C. Echocardiography: Heart abnormalities are not a primary concern in OI.
    • D. Retinal examination: OI does not have primary retinal manifestations.
    • E. Serum calcium and vitamin D: These would be checked for metabolic bone diseases or concerns about vitamin D deficiency but are not a primary concern in OI.

Test-taking tip:

On the USMLE, unique physical findings combined with a specific clinical history (like recurrent fractures with bluish sclera) often point towards a specific diagnosis. Always remember the common complications of such diagnoses for decision-making.

Reference:

  • Le, T., Bhushan, V., & Sochat, M. (2023). First Aid for the USMLE Step 1 (p. 48 – 49, 495). McGraw-Hill Education.
  • Le, T., Bhushan, V., Ascha, M., Bhardwaj, A., Boushra, M., & Griffin, D. (2023). First Aid for the USMLE Step 2 CK, eleventh edition (11th ed.). (p. 578). McGraw-Hill Education.

Question 71

A 38-Year-old woman comes to the clinic for a follow-up examination. One has opioid use disorder and currently takes methadone 40 mg daily as maintenance therapy. She says the methadone is causing mild sedation interfering with her job. She would like to discontinue methadone maintenance therapy but is concerned about heroin relapse…………….

Answer: E There is an unclear difference in efficacy between the two drugs at these doses

  1. Why the correct answer is right:
    • Buprenorphine and methadone are both effective opioid agonist treatments for opioid use disorder.
    • Studies have shown variability in outcomes, and there is no clear-cut advantage of one over the other in terms of efficacy at medium to high doses.
    • For USMLE/NBME, it is crucial to recognize that both drugs have proven benefits, but head-to-head comparisons at different doses do not demonstrate a consistent winner in efficacy.
  2. Illness script/Test-taking strategy:
    • Both buprenorphine and methadone are mainstay treatments for opioid use disorder, with their side effect profiles and considerations
    • Differentiate between the two based on side effects, drug interactions, and patient preference rather than absolute efficacy.
    • Remember the dangers of combining opioids with benzodiazepines due to the risk of respiratory depression.
  3. Why other answers are wrong:
    • A. Specific percentages about efficacy differences are not widely accepted, which seems arbitrary
    • B. Using buprenorphine in combination with benzodiazepines can increase the risk of fatal respiratory depression.
    • C. Both methadone and buprenorphine have shown benefits in treatment retention, and neither is universally superior.
    • D. Both methadone and buprenorphine are more effective than placebo in treating opioid use disorder.

Test-taking tip:

On USMLE, when comparing two well-established treatments, avoid extreme answers that overly favor one option unless the clinical scenario provides a clear reason. Here, knowing the comparable efficacy of the two drugs is essential.

Reference:

  • Le, T., Bhushan, V., & Sochat, M. (2023). First Aid for the USMLE Step 1 (p. 569, 590). McGraw-Hill Education.
  • Le, T., Bhushan, V., Ascha, M., Bhardwaj, A., Boushra, M., & Griffin, D. (2023). First Aid for the USMLE Step 2 CK, eleventh edition (11th ed.). (p. 610, 614). McGraw-Hill Education.

Question 72

A 38-year-old woman comes to the clinic for a follow-up examination. She has opioid use disorder and currently takes methadone 40 mg daily as maintenance therapy. She says the methadone is causing mild sedation that is interfering with her job. She would like to discontinue methadone maintenance therapy but is concerned about heroin relapse……..

Answer: E. Inclusion of trials with an intervention duration of 2 weeks

  1. Why the correct answer is right:
    • Buprenorphine and methadone are both effective opioid agonist treatments for opioid use disorder.
    • Maintenance therapy for opioid use disorder often requires long-term treatment; thus, a 2-week study does not reflect real-world treatment scenarios.
    • For USMLE/NBME, understanding the limitations of study durations is key; especially for chronic conditions, longer studies provide better insights.
  2. Illness script/Test-taking strategy:
    • Opioid maintenance therapy is a prolonged treatment strategy, so any study duration less than what is clinically relevant can skew results.
    • Correlate the chronic nature of opioid use disorder with the need for longer trial durations to get meaningful conclusions
    • Remember that a trial’s design and duration can affect its results’ generalizability.
  3. Why other answers are wrong:
    • A. exclusion of patients using methadone for acute detoxification: A different treatment paradigm than maintenance.
    • B. exclusion of pregnant patients: While this limits generalizability, it is often done to avoid potential fetal risks.
    • C. Inclusion of randomized controlled trials only: RCTs are the gold standard for determining causality and efficacy; thus, it strengthens confidence.
    • D. Inclusion of trials that compared active drug with placebo: Active drug vs. placebo can demonstrate efficacy but may not directly compare two active drugs.
    • F. Use of urinalysis to determine treatment retention: Urinalysis is a direct method to assess abstinence from illicit opioids, so it is a strong measure of treatment success.

Test-taking tip:

On the USMLE, always assess the duration of a study relative to the disease’s nature. Short study durations can be a major limitation for chronic diseases or conditions requiring long-term management.


Question 73

Which of the following is most likely to bias the results of this study?………

Answer: C Likelihood that more trials with positive results will be published

  1. Why the correct answer is right:
    • The scenario highlights a study examining methadone versus buprenorphine and potential biases affecting study results.
    • For USMLE/NBME, “publication bias” is important; studies with positive results are more likely to get published than those with negative or null results.
    • Thus, a review may inadvertently over-represent positive outcomes if they primarily capture published studies.
  2. Illness script/Test-taking strategy:
    • Publication bias skews systematic reviews because the positive results get more attention and are more likely to be published.
    • For exam scenarios, bias questions often hint at systemic issues in study collection or analysis.
  3. Why other answers are wrong:
    • A. Earlier diagnosis of relapse: This would be a detection bias, but nothing in the scenario specifically suggests an earlier diagnosis in one group.
    • B. Exclusion criteria: While this could introduce bias, the scenario does not provide evidence pointing to it specifically.
    • D. Source of funding: Although funding can introduce bias, there is no indication in the scenario about the funding source causing bias.
    • E. Variety of sample sizes: While different sample sizes can influence study results, they do not inherently introduce bias like publication bias.

Test-taking tip:

On the USMLE, when assessing potential study biases, always tie back to the specifics of the scenario and look for broad systemic issues like publication bias, which can fundamentally skew results.


Question 74

A 65-year-old woman comes to the office because of a 6-month history of progressive shortness of breath and a 1-year history of nonproductive cough. Initially, the shortness of breath occurred only with exertion but now occurs at rest. It does not worsen when she lies down but is still present. Her symptoms have limited her ability to travel internationally with the Peace Corps……….

Answer: E Pulmonary function testing

  1. Why the correct answer is right:
    • The patient’s complaints and physical exam findings (end-expiratory wheezes and prolonged expiratory phase) strongly suggest chronic obstructive pulmonary disease (COPD).
    • PFTs will provide diagnostic and severity information about the patient’s possible COPD
    • For USMLE/NBME, it is important to understand that the first diagnostic test for suspected COPD is spirometry (part of pulmonary function testing) to demonstrate airflow obstruction.
  2. Illness script/Test-taking strategy:
    • A classic COPD patient on the USMLE will be a long-term smoker with symptoms of chronic cough, shortness of breath, and wheezing.
    • When thinking of COPD diagnosis, always prioritize spirometry/PFTs.
  3. Why other answers are wrong:
    • A. CT scan of the chest: Useful for many pulmonary conditions but not the first line for suspected COPD.
    • B. Echocardiography: This evaluates cardiac structures and function, not primarily for pulmonary issues.
    • C. Exercise stress testing: This is primarily for evaluating cardiac ischemia.
    • D. PPD skin testing and chest x-ray: This is for tuberculosis suspicion, and the presentation does not fit TB.

Test-taking tip:

For patients with longstanding smoking history presenting with respiratory symptoms, always think of COPD and its management pathway. In USMLE, spirometry is the primary diagnostic tool for this.

Reference:

  • Le, T., Bhushan, V., & Sochat, M. (2023). First Aid for the USMLE Step 1 (p. 176). McGraw-Hill Education.
  • Le, T., Bhushan, V., Ascha, M., Bhardwaj, A., Boushra, M., & Griffin, D. (2023). First Aid for the USMLE Step 2 CK, eleventh edition (11th ed.). (p. 630 – 632). McGraw-Hill Education.

Question 75

A 28-year-old man is admitted to the intensive care unit 3 hours after undergoing resection and anastomosis of a small-bowel injury and nephrectomy because of a right renal laceration sustained in a motor vehicle collision. He also has bilateral, minimally displaced pubic rami fractures. He is intubated and mechanically ventilated……….

Answer: D Ultrasonography of the bladder

  1. Why the correct answer is right:
    • The patient’s drop in urine output post-operatively with bilateral pubic rami fractures suggests the possibility of a bladder injury or a post-renal cause for acute kidney injury.
    • Ultrasound of the bladder can quickly assess for bladder distension, rupture, or other issues.
    • On the USMLE/NBME, recognize that bladder injuries can result from pelvic fractures, and the rapid decrease in urine output paired with the fractures makes it likely.
  2. Illness script/Test-taking strategy:
    • Bilateral pubic rami fractures are a red flag for potential bladder injury in trauma patients.
    • Always evaluate post-operative changes in clinical status, especially drastic changes in urine output, for potential complications or secondary injuries.
  3. Why other answers are wrong:
    • A. CT scan of the abdomen and pelvis: Time-consuming and may not immediately address the urgent concern of urine output drop.
    • B. Repeat surgical exploration of the abdomen: No indication of an intraabdominal catastrophe to justify this invasive step.
    • C. Transfusion of packed red blood cells: While the urine is bloody, there is no clear indication of active bleeding.

Test-taking tip:

On the USMLE, always associate the clinical findings with the context. A drastic drop in urine output post-op in a patient with pelvic fractures should make you think of bladder involvement.

Reference:

  • Le, T., Bhushan, V., & Sochat, M. (2023). First Aid for the USMLE Step 1 (p. 647). McGraw-Hill Education.
  • Le, T., Bhushan, V., Ascha, M., Bhardwaj, A., Boushra, M., & Griffin, D. (2023). First Aid for the USMLE Step 2 CK, eleventh edition (11th ed.). (p. 764). McGraw-Hill Education.

Question 76

A 23-year-old nulligravid woman comes to the office because she has been unable to conceive during the past year. She and her husband have had regular unprotected intercourse during this time. The husband has no children. Prior to attempting to conceive, the patient regularly used depot medroxyprogesterone…………….

Answer: A Female factor infertility

  1. Why the correct answer is right:
    • The patient’s history of pelvic inflammatory disease (PID) puts her at risk for tubal scarring, which is a common cause of female factor infertility.
    • Hysterosalpingography would likely show tubal occlusion or other abnormalities if PID had caused significant tubal damage.
    • On the USMLE/NBME, PID is frequently linked to long-term reproductive complications due to tubal damage.
  2. Illness script/Test-taking strategy:
    • The history of PID is an important clue for potential infertility due to tubal factors.
    • In a patient presenting with infertility and a PID history, always consider tubal obstruction or damage as a potential etiology.
  3. Why other answers are wrong:
    • B. Hydatidiform mole: There is no indication from the information provided that she is at an increased risk for molar pregnancies.
    • C. Recurrent spontaneous abortions: While PID can increase the risk of ectopic pregnancy, it does not directly raise the risk for recurrent spontaneous abortions.
    • D. Successful pregnancy within the next year: Given her history of PID and likely tubal damage shown on hysterosalpingography, this is less probable without treatment.

Test-taking tip:

When the USMLE provides a detailed patient history, always correlate potential complications of past medical events (like PID) with the patient’s current presentation (like infertility).

Reference:

  • Le, T., Bhushan, V., & Sochat, M. (2023). First Aid for the USMLE Step 1 (p. 140, 677). McGraw-Hill Education.
  • Le, T., Bhushan, V., Ascha, M., Bhardwaj, A., Boushra, M., & Griffin, D. (2023). First Aid for the USMLE Step 2 CK, eleventh edition (11th ed.). (p. 489 – 490, 711 – 712). McGraw-Hill Education

Question 77

A 72-year-old man comes to the clinic because of a 3-week history of visual changes in his right eye. During this period, he has noticed that the lines of his daily crossword puzzle look curved, and the blinds in his apartment appear wavy. He wears magnifying lenses for reading. He has not had pain in his eye or photophobia…………..

Answer: D Macular degeneration

  1. Why the correct answer is right:
    • The patient’s presentation highly suggests macular degeneration, specifically age-related macular degeneration (AMD)
    • The macula, the central part of the retina, is responsible for central vision; degeneration can cause lines to appear curved.
    • On the USMLE/NBME, recognize that AMD frequently presents with metamorphopsia (warped vision) without pain.
  2. Illness script/Test-taking strategy:
    • Elderly patients with changes in central vision, especially with distortion like wavy lines, should prompt consideration of AMD.
    • Remember, AMD affects central vision, so tasks like reading or detailed work may become difficult.
  3. Why other answers are wrong:
    • A. Cataracts: Typically presents with painless, gradual blurring or dimming of vision without distortion.
    • B. Central retinal artery occlusion: Presents abruptly with painless vision loss but no distortion.
    • C. Closed-angle glaucoma: Presents with acute, painful vision loss, often with nausea and a red eye.
    • E. Temporal arteritis: Typically in older patients, but presents with headache, jaw claudication, and can have sudden vision loss, not distortion.

Test-taking tip:

When presented with visual symptoms on the USMLE, consider the age and specific type of visual change to narrow down the diagnosis. AMD’s classic description involves distorted central vision without pain.

Reference:

  • Le, T., Bhushan, V., & Sochat, M. (2023). First Aid for the USMLE Step 1 (p. 554). McGraw-Hill Education.
  • Le, T., Bhushan, V., Ascha, M., Bhardwaj, A., Boushra, M., & Griffin, D. (2023). First Aid for the USMLE Step 2 CK, eleventh edition (11th ed.). (p. 412f, 418 – 419). McGraw-Hill Education.

Question 78

A 50-year-old man comes to the office for a health maintenance examination. He says he has felt well and reports no symptoms. Medical history is unremarkable. He takes no medications. He uses up to three cans of chewing tobacco weekly and occasionally sleeps with tobacco in his mouth…………

Answer: B Surgical biopsy of the oral lesion

  1. Why the correct answer is right:
    • The patient’s long-term use of chewing tobacco and a whitish lesion on the buccal mucosa are worrisome for leukoplakia, a premalignant lesion.
    • A surgical biopsy is the definitive diagnostic step to evaluate for dysplasia or carcinoma in these lesions.
    • On the USMLE/NBME, recognize that smokeless tobacco users are at an increased risk of developing oral cancers and precancerous lesions like leukoplakia.
  2. Illness script/Test-taking strategy:
    • A chronic, white, non-scrapable lesion in a smokeless tobacco user should raise concern for leukoplakia or oral cancer.
    • To assess associated risks, always take a thorough tobacco history, including smokeless products.
  3. Why other answers are wrong:
    • A. Prescribing oral nystatin suspension: Suggests treating for oral candidiasis; no mention of immunosuppression.
    • C. Swabbing of the white area and sending for cytology: While cytology can detect malignant cells, a biopsy is a more definitive diagnostic tool for suspected oral cancers.
    • D. Observation only: Inactive management is risky, given the patient’s tobacco use and a suspicious lesion.

Test-taking tip:

On the USMLE, consider the highest risk factors and most concerning diagnoses first, especially when a combination of high-risk behavior (like chewing tobacco) with a suspicious physical finding.

Reference:

  • Le, T., Bhushan, V., & Sochat, M. (2023). First Aid for the USMLE Step 1 (p. 174). McGraw-Hill Education.
  • Le, T., Bhushan, V., Ascha, M., Bhardwaj, A., Boushra, M., & Griffin, D. (2023). First Aid for the USMLE Step 2 CK, eleventh edition (11th ed.). (p. 199f, 200). McGraw-Hill Education.

Question 79

A 60-year-old woman comes to the office for a health maintenance examination. She says she feels well. She has no history of serious illness and takes no medications.Vital signs are within normal limits. Observation of the tympanic membranes is limited because of cerumen filling both external auditory canals…………

Answer: E No further management is indicated at this time

  1. Why the correct answer is right:
    • Cerumen (earwax) is naturally produced and acts as a protective barrier; its presence, even in significant amounts, does not necessarily indicate pathology.
    • Unnecessary manipulation can lead to complications such as ear canal trauma or even infection.
    • For the USMLE/NBME, note that asymptomatic cerumen impaction often requires no intervention, especially in patients who report no hearing difficulty.
  2. Illness script/Test-taking strategy:
    • Cerumen impaction requiring intervention usually presents with symptoms such as hearing loss, pain, ringing, or fullness.
    • If a patient presents with cerumen but reports no symptoms and has no complications, conservative management is often best.
  3. Why other answers are wrong:
    • A. Irrigation of the ear canals by the physician: Not indicated if the patient is asymptomatic.
    • B. Manual removal of the cerumen by an otorhinolaryngologist: Typically reserved for symptomatic cases or for the need to visualize the tympanic membrane.
    • C. Recommendation for daily use of cotton swabs: Can push cerumen deeper into the ear canal, exacerbate impaction, and cause trauma.
    • D. Recommendation for the use of a ceruminolytic agent: Not needed if the patient is asymptomatic.

Test-taking tip:

When faced with a scenario involving a common finding (like cerumen in the ear canal) on the USMLE, always consider the patient’s symptoms and potential complications before selecting an intervention. Not all common findings require intervention.


Question 80

A 59-year-old man is brought to the emergency department because of a 1-hour history of weakness of his left arm and leg and mild headache. He says that, when he was gardening with his wife, he suddenly became unable to hold his gardening tool, lift his left arm, or walk. Two weeks ago, he had a nonproductive cough and nasal congestion that resolved spontaneously………

Answer: C Small vessel disease

  1. Why the correct answer is right:
    • The patient’s acute-onset weakness, mild headache, and CT scan findings of a large area of increased attenuation in the right putamen region indicate an ischemic stroke.
    • On the USMLE/NBME, small vessel disease is a common cause of lacunar infarcts, which often occur in the basal ganglia (like the putamen) and present with pure motor or sensory deficits.
  2. Illness script/Test-taking strategy:
    • The patient with acute-onset hemiparesis, headache, and a basal ganglia lesion on imaging suggests a lacunar infarct.
    • Always consider the location of neurological deficits when narrowing down stroke etiologies.
  3. Why other answers are wrong:
    • A. Amyloid angiopathy: Typically presents with lobar hemorrhages, not ischemic strokes.
    • B. Embolism: Could cause ischemic stroke, but the patient’s clinical presentation and imaging are more suggestive of small vessel disease.
    • D. Vasculitis: Could potentially cause a stroke, but not as likely as small vessel disease in this case.
    • E. Venous thrombosis: Usually presents with headache, papilledema, and focal neurological deficits, but not the classic basal ganglia lesion seen here.

Test-taking tip:

When interpreting neuroimaging on the USMLE, always combine the findings with the patient’s clinical presentation. In this case, the acute-onset weakness and imaging findings suggest a lacunar infarct due to small vessel disease.

Reference:

  • Le, T., Bhushan, V., & Sochat, M. (2023). First Aid for the USMLE Step 1 (p. 527). McGraw-Hill Education.
  • Le, T., Bhushan, V., Ascha, M., Bhardwaj, A., Boushra, M., & Griffin, D. (2023). First Aid for the USMLE Step 2 CK, eleventh edition (11th ed.). (p. 365 – 367). McGraw-Hill Education.

Block 3

Question 81

A 10-year-old boy is brought to the office because of moderate right knee pain. He is in no distress. Medical history is remarkable for hemophilia A. Examination of the right knee shows warmth and swelling; there is pain with flexion……………

Answer: B Factor VIII concentrate

  1. Why the correct answer is right:
    • The patient presents with hemarthrosis, a characteristic complication of hemophilia A, caused by a deficiency in factor VIII.
    • Replacing the deficient factor is crucial to stop bleeding and prevent further joint damage.
    • For USMLE/NBME, recognize that factor VIII concentrate is the immediate treatment for bleeding episodes in hemophilia A.
  2. Illness script/Test-taking strategy:
    • When presented with a patient having a history of hemophilia A and symptoms of joint bleeding, such as swelling, warmth, and limited range of motion, suspect hemarthrosis.
    • Hemophilia A = Factor VIII deficiency; Hemophilia B = Factor IX deficiency.
  3. Why other answers are wrong:
    • A. ADH (vasopressin): Used in central diabetes insipidus and some cases of esophageal variceal bleeding.
    • C. Factor IX concentrate: Used in Hemophilia B (or Christmas disease), not Hemophilia A.
    • D. Ferrous sulfate: Used for iron-deficiency anemia.
    • E. Fresh frozen plasma: Contains all clotting factors, but factor concentrates are preferred in hemophilia for precision and safety

Test-taking tip:

When choosing a therapy, be sure to match the type of hemophilia (A or B) with the appropriate deficient factor (VIII or IX). Hemarthrosis is a classic presentation in hemophilia patients, prompting factor replacement.

Reference:

  • Le, T., Bhushan, V., & Sochat, M. (2023). First Aid for the USMLE Step 1 (p. 431). McGraw-Hill Education.
  • Le, T., Bhushan, V., Ascha, M., Bhardwaj, A., Boushra, M., & Griffin, D. (2023). First Aid for the USMLE Step 2 CK, eleventh edition (11th ed.). (p. 274 – 276). McGraw-Hill Education.

Question 82

A 47-year-old woman is scheduled to undergo right knee arthroscopy and partial medial meniscectomy. Medical history is otherwise unremarkable. At her preoperative examination 2 weeks ago, her surgeon discussed the risks and benefits of the procedure, and both parties signed the operative consent form. On the patient’s arrival at the hospital, the paperwork is reviewed…….

Answer: C Do not proceed, and remove the patient from the operating room

  1. Why the correct answer is right:
    • Informed consent is essential before any surgical procedure, and not having the physical documentation in the operating room poses a medicolegal risk.
    • A surgeon’s memory or recollection is insufficient evidence of a patient’s informed consent.
    • For USMLE/NBME, recognize that even with prior discussions and signing, the actual consent form must be present and accessible during the procedure.
  2. Illness script/Test-taking strategy:
    • Informed consent is an ethical and legal requirement to ensure the patient understands and agrees to the procedure’s risks, benefits, and alternatives.
    • Always prioritize patient autonomy and safety; if there is uncertainty about the signed consent, the procedure should be halted.
  3. Why other answers are wrong:
    • A. Relying on a sedated patient’s recall is unreliable and not legally sound.
    • B. The patient has already been sedated; cannot provide informed consent.
    • D. The husband is not the primary decision-maker unless the patient is incapacitated.
    • E. Proceeding without confirmed consent can result in legal and ethical repercussions.

Test-taking tip:

On the USMLE, always prioritize patient autonomy and safety. When faced with questions involving ethics, legality, or patient rights, choose the option that ensures the highest patient care and protection standard.

Reference:

  • Le, T., Bhushan, V., & Sochat, M. (2023). First Aid for the USMLE Step 1 (p. 267, 268). McGraw-Hill Education.
  • Le, T., Bhushan, V., Ascha, M., Bhardwaj, A., Boushra, M., & Griffin, D. (2023). First Aid for the USMLE Step 2 CK, eleventh edition (11th ed.). (p. 187 – 189). McGraw-Hill Education.

Question 83

A 48-year-old man is brought to the emergency department by ambulance 45 minutes after he collapsed at home. He did not have a loss of consciousness. He has a 6- week history of shortness of breath and severe fatigue when he walks 20 to 30 feet. During the past 6 weeks, he also has had a 5-kg (11-Ib) weight gain despite a decreased appetite……….

Answer: B Dilated cardiomyopathy

  1. Why the correct answer is right:
    • The patient’s symptoms (shortness of breath, weight gain despite decreased appetite, fatigue) and physical examination findings (bilateral crackles, S3, liver span, bilateral edema) are consistent with congestive heart failure.
    • Dilated cardiomyopathy may present with an enlarged heart, reduced ejection fraction, and all four chambers dilated.
    • For USMLE/NBME, link these findings to dilated cardiomyopathy, especially in a patient with a history of chronic alcohol consumption.
  2. Illness script/Test-taking strategy:
    • Always correlate the clinical presentation with risk factors. Here, chronic alcohol use is a significant risk factor for dilated cardiomyopathy.
    • An S3 is a typical finding in dilated cardiomyopathy and indicates elevated left ventricular end-diastolic volumes.
  3. Why other answers are wrong:
    • A. A large pericardial effusion would likely cause tamponade.
    • C. Left ventricular hypertrophy usually results from chronic hypertension.
    • D. Paradoxical septal motion is seen in constrictive pericarditis or after cardiac surgery.
    • E. Regional wall motion abnormality is seen in ischemic heart disease; the patient’s presentation is more consistent with global heart dysfunction.

Test-taking tip:

On USMLE, linking a patient’s symptoms, physical findings, and risk factors (like alcohol use) can guide you to the correct diagnosis, even before considering the answer choices.

Reference:

  • Le, T., Bhushan, V., & Sochat, M. (2023). First Aid for the USMLE Step 1 (p. 315). McGraw-Hill Education.
  • Le, T., Bhushan, V., Ascha, M., Bhardwaj, A., Boushra, M., & Griffin, D. (2023). First Aid for the USMLE Step 2 CK, eleventh edition (11th ed.). (p. 40 – 46). McGraw-Hill Education.

Question 84

Patient Information Age: 18 years Gender: F. self-identified Race/Ethnicity: unspecified Site of Care: emergency department History Reason for Visit/Chief Concern: “I’m coughing up green and yellow mucus.” History of Present Illness: • cystic fibrosis with two to three exacerbations yearly for past 5 years…………

Answer: D Sputum culture

  1. Why the correct answer is right:
    • The patient’s chest x-ray showing bilateral basilar consolidations with increased interstitial markings suggests pneumonia or an infectious process.
    • Sputum cultures allow for tailoring antibiotic treatment, especially if the patient does not improve on initial broad-spectrum antibiotics.
    • For USMLE/NBME, always remember that, in a patient with suspected bacterial pneumonia, a sputum culture is critical to guide antibiotic therapy by identifying the causative organism and its sensitivities.
  2. Illness script/Test-taking strategy:
    • Pneumonia, especially in community settings, often presents with fever, cough, and radiographic evidence of consolidation.
    • Sputum cultures are crucial for antibiotic stewardship and to ensure the most effective treatment.
  3. Why other answers are wrong:
    • A. CT angiography of the chest is primarily used to evaluate pulmonary embolism.
    • B. Serum immunoglobulin concentrations are for assessing immune deficiencies.
    • C. Spirometry with lung volume measurement assesses obstructive or restrictive lung diseases.
    • E. Transbronchial lung biopsy is invasive and is used for non-infectious causes or specific pathogens not detectable by culture.

Test-taking tip: On USMLE, always look for the least invasive yet effective evaluation step, especially when managing common conditions like pneumonia. The scenario often provides enough clues to guide the diagnostic approach.

Reference:

  • Le, T., Bhushan, V., & Sochat, M. (2023). First Aid for the USMLE Step 1 (p. 176, 703). McGraw-Hill Education.
  • Le, T., Bhushan, V., Ascha, M., Bhardwaj, A., Boushra, M., & Griffin, D. (2023). First Aid for the USMLE Step 2 CK, eleventh edition (11th ed.). (p. 650 – 652). McGraw-Hill Education.

Question 85

A 30-year-old man comes to the office to establish primary care. Medical history is unremarkable, and he takes no medications. His father had a myocardial infarction at age 48 years. The patient has smoked one-half pack of cigarettes daily for 10 years. He does not drink alcoholic beverages. He is 168 cm (5 ft 6 in) tall and weighs 82 kg………….

Answer: B Fasting serum lipid studies

  1. Why the correct answer is right:
    • The patient has a family history of premature cardiovascular disease, with his father having a myocardial infarction at age 48.
    • Assessing and managing dyslipidemia is crucial for the primary prevention of cardiovascular disease in high-risk individuals.
    • For USMLE/NBME, remember that individuals with a family history of premature heart disease and personal risk factors (e.g., smoking, obesity) should undergo lipid screening to assess for dyslipidemia.
  2. Illness script/Test-taking strategy:
    • Patients with family histories of early heart disease are at increased risk; early screening for modifiable factors is a cornerstone of prevention.
    • Always relate the patient’s risk factors to the most appropriate preventive strategy; the relationship between lipids and cardiovascular risk is central here.
  3. Why other answers are wrong:
    • A. ECG: Not recommended without specific indications.
    • C. Serum chemistry profile: Not specifically indicated as a screening for CVD.
    • D. No screening studies are indicated: Given the patient’s family history and risk factors, clear indications for lipid screening.

Test-taking tip: On USMLE, preventive care questions often provide patient histories that hint toward specific guidelines. Consider a patient’s risk factors when deciding on appropriate screening tests.

Reference:

  • Le, T., Bhushan, V., & Sochat, M. (2023). First Aid for the USMLE Step 1 (p. 308 – 314). McGraw-Hill Education.
  • Le, T., Bhushan, V., Ascha, M., Bhardwaj, A., Boushra, M., & Griffin, D. (2023). First Aid for the USMLE Step 2 CK, eleventh edition (11th ed.). (p. 21, 49 – 54). McGraw-Hill Education.

Question 86

A 42-year-old nulligravid woman comes to the office because of a 6-month history of hot flashes and intermittent palpitations. Her last menstrual period was 6 months ago. Menses previously occurred at regular 29-day intervals but gradually had occurred less frequently during the past 2 years. She underwent ovarian cystectomy 13 years ago for a small benign teratoma. She used an oral contraceptive for 12 years………….

Answer: A Measurement of serum follicle-stimulating hormone concentration

  1. Why the correct answer is right:
    • This patient’s presentation with hot flashes, palpitations, and amenorrhea suggests the onset of menopause.
    • Elevated FSH is due to decreased inhibin production and decreased estradiol feedback from the ovaries.
    • For USMLE/NBME, understand that menopause is diagnosed clinically, but if confirmation is required, elevated follicle-stimulating hormone (FSH) is the most indicative.
  2. Illness script/Test-taking strategy:
    • Menopause typically occurs between ages 45-55 and can present with vasomotor symptoms like hot flashes and irregular menstruation.
    • Link decreased ovarian reserve with increased FSH due to reduced inhibin and estradiol feedback.
  3. Why other answers are wrong:
    • B. Measurement of serum luteinizing hormone concentration: LH also increases during menopause, but FSH is more commonly used for confirmation.
    • C. Measurement of serum testosterone concentration: Not used for diagnosing menopause.
    • D. ECG: No cardiovascular symptoms; palpitations are related to menopause.
    • E. Pelvic ultrasonography: Not indicated without specific gynecologic symptoms; the menopause diagnosis is largely clinical.

Test-taking tip: On the USMLE, focus on the primary presentation and symptoms. Vasomotor symptoms + amenorrhea in the age range of 45-55 should point you toward menopause, with FSH being the main marker for confirmation.

Reference:

  • Le, T., Bhushan, V., & Sochat, M. (2023). First Aid for the USMLE Step 1 (p. 655). McGraw-Hill Education.
  • Le, T., Bhushan, V., Ascha, M., Bhardwaj, A., Boushra, M., & Griffin, D. (2023). First Aid for the USMLE Step 2 CK, eleventh edition (11th ed.). (p. 486). McGraw-Hill Education.

Question 87

A 5-year-old boy is brought to the office by his parents in July because of a 2-day history of an itchy rash. The rash began as small red bumps on the right forearm and spread to the left cheek and right ankle within the same day. He has not had fever, vomiting, or diarrhea. No other family members have similar symptoms. Medical history is unremarkable…………. Answer: B Flora

  1. Why the correct answer is right:
    • The patient’s localized itching and rash symptoms that spread to exposed areas are consistent with contact dermatitis due to poison ivy, oak, or sumac exposure (all are types of flora).
    • Picnics or outdoor activities in wooded areas increase the exposure to these plants.
    • For USMLE/NBME, recognize the pattern of the rash as it often appears linearly, reflecting the pattern in which the plant brushes against the skin.
  2. Illness script/Test-taking strategy:
    • Contact dermatitis due to flora like poison ivy presents with pruritic, erythematous, linear streaks or clusters of vesicles.
    • Relate a history of outdoor activity with the typical rash presentation to poison ivy, oak, or sumac exposure.
  3. Why other answers are wrong:
    • A. Dog: Pet allergies usually manifest with respiratory symptoms or generalized hives rather than a localized rash.
    • C. Peanut butter: Food allergies might present with generalized urticaria, angioedema, GI, or respiratory symptoms.
    • D. Perfumed bath soap: Contact dermatitis from soap would be widespread, especially in areas with maximum contact, like the trunk or extremities.
    • E. Sun: Photosensitivity reactions might be generalized to all sun-exposed areas.

Test-taking tip: On the USMLE, environmental exposures and timing are key. If a patient recently spent time outdoors and develops a linear, itchy rash, think of plants like poison ivy. Always focus on the pattern and distribution of the rash.

Reference:

  • Le, T., Bhushan, V., & Sochat, M. (2023). First Aid for the USMLE Step 1 (p. 111). McGraw-Hill Education.
  • Le, T., Bhushan, V., Ascha, M., Bhardwaj, A., Boushra, M., & Griffin, D. (2023). First Aid for the USMLE Step 2 CK, eleventh edition (11th ed.). (p. 91). McGraw-Hill Education.

Question 88

A 12-month-old girl is brought to the office by her father because of a 3-day history of fever and mildly decreased appetite and activity. She has not had cough, vomiting, or diarrhea. She receives no medications. Vaccinations are up-to-date. She is at the 35th percentile for height, 50th percentile for weight, and 60th percentile for head circumference……… Answer: D Ultrasonography of the kidneys and bladder

  1. Why the correct answer is right:
    • The patient’s presentation and urinalysis findings are consistent with a urinary tract infection (UTI) in a febrile infant.
    • Ultrasonography of the kidneys and bladder is the initial imaging modality to evaluate anatomical abnormalities.
    • For USMLE/NBME, remember that a febrile UTI in a child younger than two years warrants further imaging to rule out anatomical abnormalities or vesicoureteral reflux that might predispose to infections.
  2. Illness script/Test-taking strategy:
    • Pediatric patients with febrile UTIs should be evaluated for structural or functional urinary tract abnormalities.
    • Recall that the first step in evaluation is ultrasonography, which is non-invasive and free from radiation exposure.
  3. Why other answers are wrong:
    • A. Ciprofloxacin prophylaxis: Not the first step following diagnosis.
    • B. Clindamycin prophylaxis: Usually used for UTI prophylaxis or treatment.
    • C. CT urography: Not the first choice for imaging in children due to radiation exposure.
    • E. Voiding cystourethrography (VCUG): It can identify vesicoureteral reflux, it is done after an abnormal ultrasound or recurrent UTIs.

Test-taking tip: For the USMLE, in pediatric patients with their first febrile UTI, always consider doing an ultrasound first. It is a gentle approach to initial imaging, especially in children.

Reference:

  • Le, T., Bhushan, V., & Sochat, M. (2023). First Aid for the USMLE Step 1 (p. 179, 194). McGraw-Hill Education.
  • Le, T., Bhushan, V., Ascha, M., Bhardwaj, A., Boushra, M., & Griffin, D. (2023). First Aid for the USMLE Step 2 CK, eleventh edition (11th ed.). (p. 706 – 711). McGraw-Hill Education.

Question 89

A 27-year-old woman, gravida 1, para 1, comes to the emergency department because of a 1- day history of moderate pain in her right breast. Three weeks ago, she underwent cesarean delivery of a healthy newborn at term because of breech presentation. She is breast-feeding. but the newborn is not latching properly……….

Answer: B Dicloxacillin therapy

  1. Why the correct answer is right:
    • The patient’s presentation of fever, erythema, and breast tenderness in the context of breastfeeding suggests acute mastitis.
    • Dicloxacillin or cephalexin is typically used as first-line therapy for acute mastitis.
    • For USMLE/NBME, understand that staphylococci, particularly Staphylococcus aureus, are the most common causative organisms in acute mastitis.
  2. Illness script/Test-taking strategy:
    • Acute mastitis in breastfeeding women can arise from milk stasis and the introduction of bacteria into cracked nipples, presenting with erythema and tenderness.
    • Recognize the clinical presentation and start empiric antibiotic therapy without waiting for culture results.
  3. Why other answers are wrong:
    • A. Application of a breast binder on the right: Binders may exacerbate milk stasis; continued breastfeeding or pumping is recommended.
    • C. Fluconazole therapy: Indicated for Candida infection, presents with burning nipple pain and shiny areolas.
    • D. Fine-needle aspiration of the erythematous area: Not necessary for the initial management of acute mastitis.
    • E. No further management is indicated: This would neglect the patient’s symptoms and risk complications like abscess formation.

Test-taking tip: On the USMLE, acute mastitis should be a top differential when faced with a postpartum breastfeeding patient with localized breast erythema and tenderness. Immediate management with antibiotics effective against staphylococci is essential.

Reference:

  • Le, T., Bhushan, V., & Sochat, M. (2023). First Aid for the USMLE Step 1 (p. 669). McGraw-Hill Education.
  • Le, T., Bhushan, V., Ascha, M., Bhardwaj, A., Boushra, M., & Griffin, D. (2023). First Aid for the USMLE Step 2 CK, eleventh edition (11th ed.). (p. 465 – 466). McGraw-Hill Education.

Question 90

A 58-year-old woman comes to the office for a routine examination. She has type 2 diabetes mellitus. During the past 3 months, her fingerstick blood glucose concentrations have ranged from 80 mg/dL to 230 mg/dL; previously, they ranged from 80 mg/dL to 140 mg/dL. Medications are metformin and aspirin…………….

Answer: A increase Glucose; increase insulin; decrease insulin receptor responsiveness

  1. Why the correct answer is right:
    • The patient’s clinical presentation suggests worsening insulin resistance typical of type 2 diabetes mellitus.
    • Over time, the pancreatic beta cells produce more insulin to overcome the resistance, but eventually, they cannot keep up, leading to elevated glucose levels.
    • For USMLE/NBME, remember that type 2 diabetes mellitus is characterized by a combination of insulin resistance and relative insulin deficiency.
  2. Illness script/Test-taking strategy:
    • Patients with type 2 diabetes have increased insulin resistance, which is an initial compensatory increase in insulin secretion by the pancreas.
    • Consider both the functional capacity of the pancreatic beta cells and the peripheral tissues’ response to insulin when evaluating diabetes.
    • The presence of peripheral neuropathy (decreased sensation) and an erythematous lesion on the foot highlight the chronicity and complications of poorly controlled diabetes.
  3. Why other answers are wrong:
    • B. Increase Glucose; Increase Insulin; Normal Insulin Receptor Responsiveness: This would not account for the elevated glucose levels seen in type 2 diabetes as there would be no resistance.
    • C. Increase Glucose; Normal Insulin; Decrease Insulin Receptor Responsiveness: In the presence of insulin resistance, there is an initial compensatory increase in insulin secretion, so insulin levels would not be “normal.”
    • D. Increase Glucose; Normal Insulin; Normal Insulin Receptor Responsiveness: Suggest an insulin deficiency without resistance.
    • E. Normal Glucose; Normal Insulin; Normal Insulin Receptor Responsiveness: Suggest a non-diabetic state.

Test-taking tip: In the USMLE, when evaluating diabetes mellitus type 2, always think of the dual pathology: initial insulin resistance with compensatory hyperinsulinemia, followed by beta-cell exhaustion leading to relative insulin deficiency. The progression of the disease will manifest as increasing glucose levels due to decreased insulin effectiveness and insulin receptor responsiveness.

Reference:

  • Le, T., Bhushan, V., & Sochat, M. (2023). First Aid for the USMLE Step 1 (p. 338, 350). McGraw-Hill Education.
  • Le, T., Bhushan, V., Ascha, M., Bhardwaj, A., Boushra, M., & Griffin, D. (2023). First Aid for the USMLE Step 2 CK, eleventh edition (11th ed.). (p. 124 – 128). McGraw-Hill Education.

Question 91

A 55-year-old woman comes to the office for a follow-up examination 3 months after sustaining an uncomplicated myocardial infarction in the distribution of the left anterior descending artery. She has participated in an outpatient rehabilitation program since discharge from the hospital 6 weeks ago…………..

Answer: B It is likely to double her risk for death during the next 3 months

  1. Why the correct answer is right:
    • This patient’s symptoms are indicative of major depressive disorder, which is a common complication after myocardial infarction (MI).
    • The mechanisms linking post-MI depression to poor outcomes may include non-adherence to medical recommendations, increased platelet aggregation, and autonomic dysregulation.
    • For USMLE/NBME, understand that post-MI depression has been associated with increased morbidity and mortality, including doubling the risk of death in the months following the event.
  2. Illness script/Test-taking strategy:
    • Recognize that depression after an MI is not just a psychological reaction but also significantly impacts a patient’s physical health.
    • Remember that screening for and managing depression after MI is crucial, given its impact on prognosis.
  3. Why other answers are wrong:
    • A. It is likely to decrease her perceived quality of life but have no impact on her mortality: While depression does decrease her perceived quality of life, it also has a tangible impact on mortality.
    • C. It is likely to decrease her risk for ischemia because she is likely to rest and avoid socializing: Simply resting does not mitigate the risk factors associated with post-MI depression.
    • D. It likely will have no significant effect on her physical health: Post-MI depression has been associated with worse physical health outcomes.

Test-taking tip: On the USMLE, always integrate mental and physical health, especially in post-MI patients. The consequences of untreated depression in such patients extend beyond mental well-being, affecting physical health and survival.

Reference:

  • Le, T., Bhushan, V., & Sochat, M. (2023). First Aid for the USMLE Step 1 (p. 308 – 314). McGraw-Hill Education.
  • Le, T., Bhushan, V., Ascha, M., Bhardwaj, A., Boushra, M., & Griffin, D. (2023). First Aid for the USMLE Step 2 CK, eleventh edition (11th ed.). (p. 49 – 54). McGraw-Hill Education.

Question 92

A 27-year-old woman comes to the office because of a 3-week history of nasal congestion, cough productive of sputum, intermittent moderate headache, and intermittent moderate pain over her cheeks. She has had similar symptoms three to four times yearly during the past 5 years. The symptoms last 2 to 8 weeks and generally resolve spontaneously. She has had two episodes of bacterial pneumonia during the past 5 years; both episodes resolved with antibiotic………..

Answer: E Monthly immune globulin replacement therapy

  1. Why the correct answer is right:
    • The patient has a history of recurrent sinopulmonary infections and decreased IgA, IgG, and IgM levels, suggesting common variable immunodeficiency (CVID).
    • Treatment of CVID involves immune globulin replacement to provide passive immunity.
    • For USMLE/NBME, remember that CVID is characterized by recurrent bacterial infections and decreased levels of most or all immunoglobulin isotypes.
  2. Illness script/Test-taking strategy:
    • CVID patients are more susceptible to bacterial infections, particularly of the respiratory tract.
    • Associate recurrent sinopulmonary infections with decreased immunoglobulins to CVID.
  3. Why other answers are wrong:
    • A. Daily inhaled tobramycin therapy: Used for chronic Pseudomonas infection in cystic fibrosis.
    • B. Daily intranasal glucocorticoid therapy: Helps with allergic rhinitis.
    • C. Daily trimethoprim-sulfamethoxazole therapy: Prophylaxis for Pneumocystis jirovecii pneumonia in immunocompromised patients.
    • D. Intranasal influenza virus vaccine administration: Used for influenza prevention, but not the primary issue in CVID.

Test-taking tip: On USMLE, when presented with a patient with recurrent infections and specific antibody deficiencies, think of primary immunodeficiency. CVID’s hallmark is decreased levels of multiple immunoglobulin types.

Reference:

  • Le, T., Bhushan, V., & Sochat, M. (2023). First Aid for the USMLE Step 1 (p. 52, 114, 116). McGraw-Hill Education.
  • Le, T., Bhushan, V., Ascha, M., Bhardwaj, A., Boushra, M., & Griffin, D. (2023). First Aid for the USMLE Step 2 CK, eleventh edition (11th ed.). (p. 542 – 546). McGraw-Hill Education.

Question 93

A 47-year-old woman comes to the office for a follow-up examination. Three days ago, she was brought to the emergency department because of cellulitis of her right lower extremity, and trimethoprim-

sulfamethoxazole therapy was begun. She has had no adverse reaction to the medication. She has kept her leg elevated, and her cellulitis has improved moderately…………

Answer: A Review for potential drug-drug interaction

  1. Why the correct answer is right:
    • The patient is on warfarin, and her INR is significantly elevated at 5.2, increasing her risk of bleeding; the most likely cause is her recent addition of trimethoprim-sulfamethoxazole, which potentiates the effects of warfarin.
    • Always be vigilant about drug-drug interactions, especially when introducing a new medication to a patient on warfarin.
    • For USMLE/NBME, recognize that trimethoprim-sulfamethoxazole can increase INR by inhibiting the hepatic metabolism of warfarin.
  2. Illness script/Test-taking strategy:
    • A patient with a mechanical valve is at risk for thrombosis and will be on anticoagulation.
    • Associate an elevated INR with potential drug-drug interactions, especially in patients taking warfarin.
  3. Why other answers are wrong:
    • B. Dietary counseling: While diet, particularly vitamin K intake, can affect warfarin dosing, there is no indication.
    • C. Echocardiography: Useful for evaluating cardiac structures, not for elevated INR.
    • D. Venous duplex ultrasonography: Assesses for deep vein thrombosis. Not pertinent here.
    • E. Discontinuation of allopurinol: Allopurinol is for gout, and there is no indication that it contributed to the elevated INR.

Test-taking tip: Consider potential drug interactions when a patient on a stable warfarin dose suddenly changes INR. Familiarize yourself with common medications that can potentiate or inhibit the effects of warfarin.

Reference:

  • Le, T., Bhushan, V., & Sochat, M. (2023). First Aid for the USMLE Step 1 (p. 431). McGraw-Hill Education.
  • Le, T., Bhushan, V., Ascha, M., Bhardwaj, A., Boushra, M., & Griffin, D. (2023). First Aid for the USMLE Step 2 CK, eleventh edition (11th ed.). (p. 270, 273t). McGraw-Hill Education.

Question 94

An 87-year-old man is admitted to the hospital because of a 4-day history of moderate abdominal cramps, nausea, and vomiting. He has not had a bowel movement or passed gas in the past 24 hours. He has colon cancer metastatic to the lungs, liver, and peritoneum. Eighteen months ago, he underwent a low anterior resection for T3N2 rectal cancer. Six months ago, exploratory laparotomy showed diffuse carcinomatosis…………

Answer: C Explain the futility of the operation to the patient and his family

  1. Why the correct answer is right:
    • This patient has a poor prognosis due to diffuse carcinomatosis and recurrent bowel obstruction. Previous surgical exploration deemed him unsuitable for future laparotomy due to the lack of therapeutic options.
    • The physician is responsible for advising against interventions that are unlikely to provide a benefit or may cause harm, despite patient or family requests.
    • For USMLE/NBME, understand that engaging in a goals-of-care discussion is crucial in cases with limited therapeutic options rather than offering non-beneficial treatments.
  2. Illness script/Test-taking strategy:
    • Recurrent bowel obstruction in the setting of metastatic colon cancer, especially after a recent surgery deemed further laparotomies non-beneficial, sets the stage for palliative care considerations.
    • Associate futility of care with prioritizing comfort and quality of life rather than aggressive interventions that may not provide benefit.
  3. Why other answers are wrong:
    • A. Abide by the wishes of the patient and his family: Not recommended.
    • B. Alter the chemotherapy regimen: This will not address the acute obstruction and the patient’s current state.
    • D. Offer to transfer the patient to another facility: Transferring does not solve the issue of the futility of surgery.
    • E. Suggest external beam radiation therapy: This might be used to palliate symptoms in specific situations.

Test-taking tip: For USMLE, when faced with clinical scenarios where aggressive interventions may not be beneficial, consider the principles of medical ethics and the importance of communicating the reality and options to patients.

Reference:

  • Le, T., Bhushan, V., & Sochat, M. (2023). First Aid for the USMLE Step 1 (p. 236 – 238). McGraw-Hill Education.

Question 95

A 20-year-old man comes to the office at the end of the summer because of a 2-month history of generalized malaise, fatigue, intermittent cough productive of whitish sputum, and decreased appetite. During this time, he has lost approximately 4 kg (10 lb). The patient previously was on the college track team but now feels winded when he tries to go running. He has a history of childhood asthma. He currently takes no medications. He does not smoke cigarettes, drink alcoholic beverages, or use other substances. He identifies as African American. He has been working at a lumber mill for the summer, feeding logs………….

Answer: C Hypersensitivity pneumonitis

  1. Why the correct answer is right:
    • The patient’s symptoms and chest x-ray findings suggest hypersensitivity pneumonitis (HP).
    • The presentation with upper lung involvement, restrictive and obstructive pulmonary test findings, and rapid onset after exposure supports HP.
    • For USMLE/NBME, remember that HP is an allergic inflammatory response to inhaled organic antigens, with lumber mill work posing exposure to wood dust.
  2. Illness script/Test-taking strategy:
    • HP results from repeated inhalation of specific environmental antigens leading to an inflammatory response in the lungs.
    • Associate diffuse crackles, reticulonodular pattern, and mixed obstructive and restrictive lung disease with HP, especially with relevant exposure.
  3. Why other answers are wrong:
    • A. Aspergillosis: Usually seen in immunocompromised or presents as a fungal ball (aspergilloma) in pre-existing lung cavities.
    • B. Asthma recurrence: Chronic inflammatory disorder but does not cause weight loss or reticulonodular pattern on chest x-ray.
    • D. Sarcoidosis: Noncaseating granulomas and often bilateral hilar lymphadenopathy are its hallmark; it is prevalent in African Americans.
    • E. Silicosis: Results from inhalation of silica dust (not wood); presents with upper lung nodularity.

Test-taking tip: In USMLE, when considering occupational lung diseases, always connect the patient’s occupation to the potential exposures and resultant diseases. The rapid onset of symptoms after a specific exposure is a key hint.

Reference:

  • Le, T., Bhushan, V., & Sochat, M. (2023). First Aid for the USMLE Step 1 (p. 696). McGraw-Hill Education.
  • Le, T., Bhushan, V., Ascha, M., Bhardwaj, A., Boushra, M., & Griffin, D. (2023). First Aid for the USMLE Step 2 CK, eleventh edition (11th ed.). (p. 634). McGraw-Hill Education.

Question 96

A 65-year-old man comes to the office because of a 2-year history of progressive shortness of breath on exertion and a 6-month history of nonproductive cough. He now has shortness of breath when walking to his mailbox. He has not had fever, weight loss, or chest pain. He has not had recent sick contacts………..

Answer: A Amiodarone

  1. Why the correct answer is right:
    • The patient’s clinical presentation suggests amiodarone-induced pulmonary toxicity (AIPT).
    • The bilateral nature of the lung findings, absence of other exposures, and time course of amiodarone use make AIPT a top consideration.
    • For USMLE/NBME, recognize that amiodarone, while effective for arrhythmias, has potential long-term toxic effects on the lungs.
  2. Illness script/Test-taking strategy:
    • Amiodarone has multiple potential side effects due to its long half-life and extensive tissue deposition, with the lungs being a major target.
    • Connect bilateral crackles and progressive dyspnea in the setting of amiodarone use to potential AIPT.
  3. Why other answers are wrong:
    • B. Lisinopril: Can cause cough secondary to increased bradykinin levels.
    • C. Propranolol: Beta-blockers can exacerbate asthma or COPD symptoms but do not directly cause the lung findings described.
    • D. Tiotropium: A bronchodilator for COPD; its side effects are typically dry mouth and urinary retention.
    • E. Warfarin: Used as an anticoagulant, its primary adverse effect is bleeding.

Test-taking tip: For USMLE, always correlate a drug’s known side effects with the clinical presentation. Long-term amiodarone use and new lung symptoms should raise a flag for potential pulmonary toxicity.

Reference:

  • Le, T., Bhushan, V., & Sochat, M. (2023). First Aid for the USMLE Step 1 (p. 250, 696). McGraw-Hill Education.
  • Le, T., Bhushan, V., Ascha, M., Bhardwaj, A., Boushra, M., & Griffin, D. (2023). First Aid for the USMLE Step 2 CK, eleventh edition (11th ed.). (p. 747t). McGraw-Hill Education.

Question 97

A colleague of a 47-year-old physician who is conducting patient evaluations during afternoon walk-in clinic hours notices that the physician appears groggy, has slurred speech, and smells of alcohol. Earlier in the day, the physician had attended a celebratory lunch for the graduation of one of the clinic’s employees from nursing school. The colleague has not witnessed similar findings in the physician in the past. The physician is scheduled to work at the clinic for the next 4 hours………….

Answer: E Report the physician to the clinic administrator and conduct the remainder of the clinic hours without him

  1. Why the correct answer is right:
    • Patient safety is paramount, and a physician under the influence poses a significant risk.
    • Reporting the physician to the clinic administrator ensures accountability, while continuing the clinic hours without him ensures patient care continuity.
    • For USMLE/NBME, always prioritize the immediate safety of the patients over potential repercussions to the involved physician.
  2. Illness script/Test-taking strategy:
    • When faced with ethical dilemmas on the USMLE, always choosing the option that prioritizes patient safety and well-being is a good rule of thumb.
    • In cases of impaired physicians, immediate action is necessary to prevent potential harm to patients.
  3. Why other answers are wrong:
    • A. Allowing the physician to continue jeopardizes patient safety.
    • B. Canceling clinic hours disrupts patient care without addressing the root issue.
    • C. While the physician assistant can aid in patient care, this does not address the immediate concern of an impaired physician.
    • D. Merely confronting and asking the physician to leave may not guarantee that the physician complies and does not address reporting.
    • F. Reporting to the state medical board is a more drastic step; the immediate concern is patient safety.

Test-taking tip: In USMLE, ethical questions often present scenarios testing your ability to balance patient welfare, professional responsibilities, and interpersonal challenges. Always prioritize patient safety and appropriate reporting in situations with immediate risk.

Reference:

  • Le, T., Bhushan, V., & Sochat, M. (2023). First Aid for the USMLE Step 1 (p. 272, 273). McGraw-Hill Education.

Question 98

A 32-year-old nulligravid woman comes to the office because she has been unable to conceive during the past 12 months. She and her husband have unprotected sexual intercourse three times weekly. Two years ago, she was involved in a motor vehicle collision; CT scan at that time showed absence of the left kidney………

Answer : In this scenario, the key clinical feature to consider is the patient’s history of a motor vehicle collision and subsequent finding of a missing left kidney. This medical history is significant as it implies potential congenital abnormalities or trauma-related consequences, which can impact pregnancy outcomes. Specifically, the absence of a kidney, whether due to congenital reasons or trauma, can be associated with reproductive anomalies like a bicornuate uterus. A bicornuate uterus, characterized by a heart-shaped, divided uterine cavity, is a known risk factor for preterm labor. Preterm labor, defined as labor that begins before 37 weeks of gestation, can be more prevalent in women with uterine malformations due to reduced uterine capacity and altered uterine anatomy.

  1. Illness script/Test-taking strategy:
    • It’s crucial to connect the dots between a patient’s past medical history, especially any congenital anomalies or significant trauma, and their potential impact on pregnancy. In cases of uterine anomalies like a bicornuate uterus, the altered shape and size of the uterus can lead to complications such as preterm labor.
    • Connect past medical history, especially significant trauma or congenital conditions, to potential pregnancy-related risks.
  2. Why other answers are wrong:
    • A. Fetal macrosomia: This condition is typically associated with maternal factors like diabetes or obesity, rather than uterine anomalies.
    • B. Multiple gestation: The likelihood of multiple gestations is influenced by factors unrelated to uterine shape, such as fertility treatments or genetic predispositions.
    • C. Oligohydramnios: While renal function can influence amniotic fluid volume, a single absent kidney usually doesn’t significantly impact amniotic fluid levels.
    • D. Preeclampsia: This condition is more commonly linked with maternal factors like hypertension and not directly with unilateral renal agenesis or uterine anomalies.

Test-taking tip: For exams like the USMLE or NBME, always consider how a patient’s comprehensive medical history, including any physical anomalies or past traumas, might influence their current condition or risk factors. In this case, a past traumatic event and a significant anatomical finding (absence of a kidney) hint at potential risks in pregnancy, such as preterm labor due to a possible associated uterine anomaly.

Reference:

  • Le, T., Bhushan, V., & Sochat, M. (2023). First Aid for the USMLE Step 1 (p. 598). McGraw-Hill Education.

Question 99

A 62-year-old woman is brought to the emergency department (ED) by her husband because of a 3-day history of increasing confusion. She has a 20-year history of bipolar disorder well controlled with medication for the past 5 years. Her husband forgot to bring her medication bottles and cannot remember which Medication………………..

Answer: C Lithium

  1. Why the correct answer is right:
    • The patient’s presentation is suggestive of lithium toxicity.
    • Dehydration in the context of lithium use, especially with increased physical activity and sun exposure, can lead to increased lithium levels.
    • For USMLE/NBME, recognize that lithium toxicity can be precipitated by factors that lead to reduced renal clearance, such as dehydration.
  2. Illness script/Test-taking strategy:
    • Lithium is used for bipolar disorder and has a narrow therapeutic index, making it easy to reach toxic levels, especially in settings where renal clearance is compromised.
    • Consider medication side effects or toxicity in patients with a long history of chronic drug use and new neurological symptoms.
  3. Why other answers are wrong:
    • A. Acetaminophen: Overdose can cause hepatotoxicity, not confusion, and ECG changes.
    • B. Bupropion: While it can lower the seizure threshold, it does not typically cause AV blocks.
    • D. Risperidone: Can cause extrapyramidal side effects but not the constellation of symptoms seen here.
    • E. Topiramate: Can cause cognitive side effects and kidney stones but not AV blocks.

Test-taking tip: On USMLE, drug toxicities often present with multiple system involvement. Recognizing the full spectrum of a drug’s side effects can be key to arriving at the right diagnosis. In the context of bipolar disorder and dehydration, lithium should be a top differential for toxicity.

Reference:

  • Le, T., Bhushan, V., & Sochat, M. (2023). First Aid for the USMLE Step 1 (p. 589). McGraw-Hill Education.
  • Le, T., Bhushan, V., Ascha, M., Bhardwaj, A., Boushra, M., & Griffin, D. (2023). First Aid for the USMLE Step 2 CK, eleventh edition (11th ed.). (p. 606t). McGraw-Hill Education.

Question 100

A 3-year-old boy is brought to the office prior to undergoing dental work under general anesthesia for extensive dental caries. He feels well. He has a history of a heart murmur. Last year, he developed urticarial and facial swelling after he was administered penicillin for streptococcal pharyngitis………………

Answer: E No antibiotic prophylaxis is recommended

  1. Why the correct answer is right:
    • Current guidelines recommend antibiotic prophylaxis for dental procedures only in patients with high-risk cardiac conditions, such as prosthetic heart valves, a History of infective endocarditis, certain congenital heart conditions, and cardiac transplant recipients with valvulopathy.
    • The benefits of prophylaxis in patients without high-risk conditions do not outweigh the risks of antibiotic resistance and allergic reactions.
    • For USMLE/NBME, remember that most patients, including those with simple heart murmurs, do not need antibiotic prophylaxis for dental procedures.
  2. Illness script/Test-taking strategy:
    • Although this patient was allergic to penicillin, the focus should be on his underlying cardiac condition.
    • Without other risk factors, heart murmurs alone do not require antibiotic prophylaxis for dental procedures.
  3. Why other answers are wrong:
    • A. Azithromycin: Used as an alternative for penicillin-allergic patients.
    • B. Cephalexin: A first-generation cephalosporin that could cross-react with penicillin.
    • C. Clindamycin: An option for patients allergic to penicillin.
    • D. Vancomycin: A broad-spectrum antibiotic, unnecessary here.

Test-taking tip: On USMLE, familiarity with updated guidelines is essential. The overuse of antibiotics can lead to resistance, and thus their use is limited only to high-risk cases for specific conditions, like infective endocarditis prophylaxis.

Reference:

  • Le, T., Bhushan, V., & Sochat, M. (2023). First Aid for the USMLE Step 1 (p. 185). McGraw-Hill Education.
  • Le, T., Bhushan, V., Ascha, M., Bhardwaj, A., Boushra, M., & Griffin, D. (2023). First Aid for the USMLE Step 2 CK, eleventh edition (11th ed.). (p. 347, 748t). McGraw-Hill Education.

Question 101

A 30-year-old man comes to the physician because he and his wife have been unable to conceive during the past 2 years. Previous evaluation of his wife showed no abnormalities. He has not been exposed to pesticides, heavy metals, radiation, or testicular overheating. Physical examination shows no abnormalities…………….

Answer: A Ask the patient about his alcohol intake, smoking, and stress level

  1. Why the correct answer is right:
    • Oligospermia refers to a sperm count that is below the normal threshold. Lifestyle factors like excessive alcohol consumption, smoking, and high stress can negatively impact sperm production.
    • Inquiring about the patient’s lifestyle factors can potentially identify a reversible cause of his oligospermia.
    • For USMLE/NBME, when confronted with a fertility issue, a comprehensive assessment of potential reversible or modifiable factors should be the initial approach.
  2. Illness script/Test-taking strategy:
    • Before moving on to specific interventions or treatments for oligospermia, it is essential to identify and address modifiable risk factors.
    • Recognize that common lifestyle factors, including stress, smoking, and excessive alcohol, can significantly affect male infertility.
  3. Why other answers are wrong:
    • B. Educate the patient about declining sperm counts with increased age: While sperm count can decrease with age, the patient is only 30.
    • C. Prescribe a phosphodiesterase inhibitor: These are used mainly for erectile dysfunction.
    • D. Provide a topical testosterone gel: External testosterone can further suppress intratesticular testosterone and sperm production.
    • E. Recommend in vitro fertilization: This is premature without first addressing modifiable factors.

Test-taking tip: On USMLE, always consider addressing reversible or modifiable factors before progressing to pharmacological interventions or advanced procedures. This approach is both patient-centric and cost-effective.

Reference:

  • Le, T., Bhushan, V., & Sochat, M. (2023). First Aid for the USMLE Step 1 (p. 407). McGraw-Hill Education.

Question 102

An 8-year-old boy is brought to the urgent care center by his mother while on a family vacation in Florida because of a 3-day history of an itchy, burning rash. The ambient temperature during the past 3 days has been 37.8°C (100.0°F). The mother does not recall any recent exposures for her son, and none of her other children have a rash………..

Answer: B Heat avoidance

  1. Why the correct answer is right:
    • The patient’s presentation of clear, tiny vesicles resembling water droplets after high ambient temperatures is consistent with miliaria crystallina.
    • The best management is to avoid further heat exposure and allow the body to cool.
    • For USMLE/NBME, recognize miliaria crystallina as a sweat gland blockage condition caused by excessive sweating and that it is the mildest form of heat rash.
  2. Illness script/Test-taking strategy:
    • When encountering skin conditions in the context of environmental exposures, consider the direct effects of that environment, such as heat leading to blockage of sweat ducts.
    • Miliaria crystallina presents without significant erythema or pustules, differentiating it from other types of miliaria.
  3. Why other answers are wrong:
    • A. Antifungal cream: There is no indication of fungal infection here.
    • C. Oral antibiotic therapy: The rash is not infectious or bacterial in nature.
    • D. Oral antiviral therapy: There is no indication of a viral infection.
    • E. Oral retinoid therapy: Used for severe acne or psoriasis, not relevant here.
    • F. Ultraviolet therapy: Not indicated for miliaria crystallina and can exacerbate the condition.

Test-taking tip: On USMLE, always correlate clinical findings with the provided context. Here, the context of high temperatures and the specific appearance of the rash makes miliaria crystallina the most likely diagnosis.

Reference:

  • Le, T., Bhushan, V., & Sochat, M. (2023). First Aid for the USMLE Step 1 (p. 178). McGraw-Hill Education.
  • Le, T., Bhushan, V., Ascha, M., Bhardwaj, A., Boushra, M., & Griffin, D. (2023). First Aid for the USMLE Step 2 CK, eleventh edition (11th ed.). (p. 528t). McGraw-Hill Education.

Question 103

A 62-year-old woman comes to the office because of a 3-month history of anxiety, insomnia, and frequent bowel movements. She has had a 9-kg (20-Ib) weight loss during this period. Pulse is 90/min and irregularly irregular, respirations are 22/min, and blood pressure is 150/65 mm Hg. She is restless. The thyroid gland is diffusely enlarged…………

Answer: E Thyrotoxicosis

  1. Why the correct answer is right:
    • The clinical picture is consistent with thyrotoxicosis.
    • The presentation of widened palpebral fissures, an irregularly irregular heartbeat, and the described skin changes all point towards thyroid hormone excess.
    • For USMLE/NBME, it is essential to recognize the systemic manifestations of thyroid hormone excess.
  2. Illness script/Test-taking strategy:
    • In a patient with nervousness, weight loss, heat intolerance, cardiac changes (like atrial fibrillation), and a diffusely enlarged thyroid, always consider thyrotoxicosis.
    • Including specific findings like nail changes and skin changes help solidify the diagnosis.
  3. Why other answers are wrong:
    • A. Addison disease: Present with weakness, hyperpigmentation, and low blood pressure.
    • B. Carcinoid syndrome: Manifests with flushing, diarrhea, and heart valve lesions due to serotonin secretion.
    • C. Cushing syndrome: Features include central obesity, moon face, purple striae, and hirsutism due to excess cortisol.
    • D. Pheochromocytoma: Presents with episodic hypertension, headache, palpitations, and sweating due to a catecholamine-producing tumor.

Test-taking tip: On USMLE, endocrine disorders can manifest with various systemic symptoms. Careful attention to systemic and specific physical findings will help narrow the differential. In this scenario, the findings, like palpable thyroid with a bruit and skin changes, provide key diagnostic clues for thyrotoxicosis.

Reference:

  • Le, T., Bhushan, V., & Sochat, M. (2023). First Aid for the USMLE Step 1 (p. 315, 335). McGraw-Hill Education.
  • Le, T., Bhushan, V., Ascha, M., Bhardwaj, A., Boushra, M., & Griffin, D. (2023). First Aid for the USMLE Step 2 CK, eleventh edition (11th ed.). (p. 130 – 134). McGraw-Hill Education.

Question 104

A randomized placebo-controlled clinical trial is conducted to assess the effectiveness of a selective estrogen receptor modulator (SERM) for chemoprophylaxis of breast cancer. A subset of the results…………….

Answer: B. 2.6

  1. Why the correct answer is right:
    • To determine the absolute risk reduction (ARR), you subtract the event rate in the treatment group from the event rate in the control group.
    • In this case, the breast cancer rate in the placebo group is 3.6 per 1000 woman-years.
    • The breast cancer rate in the SERM group is 0.97 per 1000 womanyears.
  2. Illness script/Test-taking strategy:
    • Calculate the Incidence in each group (treated and untreated).
    • ARR = Incidence in the untreated group (or placebo) – Incidence in the treated group.
    • Here, ARR = 3.6 – 0.97 = 2.63, which is closest to 2.6 per 1000 womanyears.
  3. Why other answers are wrong:
    • A. 0.28: Does not represent the difference between the two incidence rates.
    • C. 3.6: This is the Incidence in the placebo group, not the ARR.
    • D. 6: This is an overestimation of the ARR.
    • E. 7: This is the number of cases in the SERM group, not the ARR.

Test-taking tip: For questions involving ARR on the USMLE, always calculate the event rate for both groups first and then determine the difference. Remember, the ARR is the difference between the incidence rates of the control group (usually placebo) and the treatment group

Reference:

  • Le, T., Bhushan, V., & Sochat, M. (2023). First Aid for the USMLE Step 1 (p. 258). McGraw-Hill Education.
  • Le, T., Bhushan, V., Ascha, M., Bhardwaj, A., Boushra, M., & Griffin, D. (2023). First Aid for the USMLE Step 2 CK, eleventh edition (11th ed.). (p. 160). McGraw-Hill Education.

Question 105

A 16-year-old boy comes to the emergency department (ED) because of moderate left knee pain and abrasions sustained 2 hours ago in a skateboarding collision. He has a history of occasional injuries sustained while skateboarding, including an injury to the same knee 6 months ago. He says his knee had felt fine until today’s injury, and he has been fit and active. The patient has an EXT mutation, which is also present in his father. The patient takes no medications. Vital signs are within normal limits…………….

Answer: E Recommendation to bear weight as tolerated; no further workup needed

  1. Why the correct answer is right:
    • The patient has multiple osteochondromas due to the EXT mutation, consistent with hereditary multiple exostoses (HME).
    • HME is typically benign; the patient’s unchanged x-ray over 6 months supports this benign nature.
    • In the absence of malignant features like rapid growth, pain unrelated to trauma, or changes in the overlying skin, further workup, like a biopsy, is unnecessary
  2. Illness script/Test-taking strategy:
    • Osteochondromas (bone polyps) are benign bone tumors that appear in childhood and stop growing after skeletal maturity.
    • Always link the EXT mutation to hereditary multiple exostoses, presenting with multiple osteochondromas.
    • Consider the benign nature of osteochondromas unless presented with suspicious malignant features.
  3. Why other answers are wrong:
    • A-B. Biopsy of the polyps: No suspicious features to warrant an invasive biopsy.
    • C-D. MRI of the knee: An unchanged X-ray over 6 months with no concerning features makes MRI unnecessary.
    • F. Recommendation for no weight bearing: The patient’s ability to walk, even with a limp, and the absence of instability suggest weight-bearing is safe.

Test-taking tip: In USMLE, when given a known genetic mutation (like EXT) with a typical clinical picture, focus on the related condition (in this case, HME) and its usual presentation and management. Avoid invasive measures without clear indication.

Reference:

  • Le, T., Bhushan, V., & Sochat, M. (2023). First Aid for the USMLE Step 1 (p. 470). McGraw-Hill Education.
  • Le, T., Bhushan, V., Ascha, M., Bhardwaj, A., Boushra, M., & Griffin, D. (2023). First Aid for the USMLE Step 2 CK, eleventh edition (11th ed.). (p. 324). McGraw-Hill Education.

Question 106

A 2-year-old girl is brought to the office by her parents for a well-child examination. She was adopted 2 months ago. She has no history of serious illness and receives no medications. Vital signs are within normal limits…………….

Answer: B As early as possible, even if she cannot process the whole experience

  1. Why the correct answer is right:
    • Discussing adoption openly from an early age promotes trust and understanding between the child and the adoptive parents.
    • It allows the child to integrate the concept of adoption into their selfidentity gradually.
    • Parents can start the conversation early to ensure the child learns about their adoption in a supportive and loving environment.
  2. Illness script/Test-taking strategy:
    • Transparency and openness in families about adoption prevent feelings of betrayal or mistrust as the child grows up.
    • Recognize that the developmental understanding of adoption evolves, and discussions will be adjusted according to age.
    • Even if not fully understood, early discussions lay the groundwork for more complex conversations as the child matures.
  3. Why other answers are wrong:
    • A. After she turns 18 years of age: Waiting too long can result in feelings of mistrust or betrayal.
    • C. Once the seal on the adoption records has been lifted: Telling is not necessarily tied to bureaucratic processes.
    • D. Right before she is enrolled in kindergarten: This is an arbitrary time that might delay the conversation unnecessarily.
    • E. When she learns that she is not biologically related to her parents: Waiting for this realization can lead to confusion and emotional turmoil.

Test-taking tip: In USMLE, psychosocial questions prioritize the patient’s emotional well-being and transparency. Always look for options that emphasize open communication and trust.

Reference:

  • Le, T., Bhushan, V., & Sochat, M. (2023). First Aid for the USMLE Step 1 (p.256). McGraw-Hill Education.

Question 107

A 74-year-old woman comes to the clinic 2 days after the sudden onset of severe left foot pain. She has had difficulty walking during this time and has been using a cane. She has not sustained trauma to the foot. She has type 2 diabetes mellitus and hypertension. Medications are metformin, insulin glargine hydrochlorothiazide, lisinopril, and atorvastatin…….

Answer: B Gout

  1. Why the correct answer is right:
    • The sudden onset of severe pain in a joint, especially the great toe, with hyperuricemia suggests an acute gouty attack.
    • Patients with diabetes and hypertension and on medications like thiazide diuretics (hydrochlorothiazide) have a higher risk for gout.
    • For USMLE/NBME, always think of gout with the abrupt onset of severe joint pain in hyperuricemia.
  2. Illness script/Test-taking strategy:
    • Gout classically presents with a painful, swollen, red joint, often the first metatarsophalangeal joint (“podagra”).
    • Recall the connection between thiazide diuretics and increased uric acid levels leading to gout.
  3. Why other answers are wrong:
    • A. Cellulitis: Typically involves erythema, warmth, and swelling but not acute severe pain.
    • C. Infected mucoid cyst: Common on the dorsum of the wrist, not typically the great toe.
    • D. Osteoarthritis: Chronic condition; does not cause sudden severe pain.
    • E. Pseudogout: Presents similarly but mostly affects larger joints like the knee.
    • F. Rheumatoid arthritis: Typically involves multiple joints symmetrically.

Test-taking tip: On the USMLE, hyperuricemia with sudden severe joint pain, especially in the big toe, should prompt consideration of gout, particularly if associated with risk factors or certain medications.

Reference:

  • Le, T., Bhushan, V., & Sochat, M. (2023). First Aid for the USMLE Step 1 (p. 496). McGraw-Hill Education.
  • Le, T., Bhushan, V., Ascha, M., Bhardwaj, A., Boushra, M., & Griffin, D. (2023). First Aid for the USMLE Step 2 CK, eleventh edition (11th ed.). (p. 346 – 348). McGraw-Hill Education.

Question 108

A 24-year-old primigravid woman at 10 weeks gestation comes to the office for a scheduled dilatation and suction curettage. Ultrasonography 2 days ago showed a fetus consistent in size with a 9-week gestation with absent cardiac activity. Two weeks ago, the patient had an upper respiratory tract infection that resolved spontaneously. One year ago, she was diagnosed with herpes simplex virus (HS) 2……………

Answer: D Type of vaginal discharge

  1. Why the correct answer is right:
    • The homogenous white-gray vaginal discharge with a pH of 5.5 suggests bacterial vaginosis (BV), which increases the risk of postoperative pelvic infections.
    • BV disrupts the normal vaginal flora and increases susceptibility to infections.
    • For USMLE/NBME, consider BV a risk factor for post-operative infections following gynecological procedures.
  2. Illness script/Test-taking strategy:
    • Bacterial vaginosis presents with a characteristic fishy odor, homogeneous gray-white discharge, and an elevated vaginal pH (>4.5).
    • Use clinical clues: in a gynecological context, recognize altered vaginal discharge as indicative of an underlying infection that can predispose to further complications.
  3. Why other answers are wrong:
    • A. history of upper respiratory tract infection: It resolved spontaneously and is unrelated to pelvic infections.
    • B. HSV 2: While HSV 2 can cause genital ulcers, it is not directly related to post-operative infections unless active lesions are present.
    • C. Obesity: While obesity can be a general risk factor for surgery, it is not the most serious predisposing risk in this case.
    • E. Vulvar shaving: While it can introduce skin microtears, the presence of BV is a more significant risk for pelvic infection.

Test-taking tip:

On the USMLE, when assessing risk factors for post-operative infections, always consider the underlying health of the surgical site. In gynecological scenarios, the status of vaginal flora plays a crucial role.

Reference:

  • Le, T., Bhushan, V., & Sochat, M. (2023). First Aid for the USMLE Step 1 (p. 155, 179). McGraw-Hill Education.
  • Le, T., Bhushan, V., Ascha, M., Bhardwaj, A., Boushra, M., & Griffin, D. (2023). First Aid for the USMLE Step 2 CK, eleventh edition (11th ed.). (p. 488, 489). McGraw-Hill Education.

Question 109

A 15-year-old boy is brought to the office 2 hours after he awoke with weakness of the right side of his face. He has a 2-week history of intermittent headache, sensitivity to light, and mild to moderate neck pain. He has no history of serious illness. His only medication is acetaminophen as needed. He lives in New Jersey and has not traveled outside of the state during the past 6 months. extremities………….

Answer: B Measurement of serum Lyme (Borrelia burgdorfen) antibody concentration

  1. Why the correct answer is right:
    • The patient’s presentation of facial nerve palsy (Bell’s palsy) combined with a history of headache, photophobia, and meningismus in an endemic region (New Jersey) suggests Lyme disease.
    • Lyme disease can present with neurologic manifestations such as facial nerve palsy and lymphocytic meningitis.
    • For USMLE/NBME, linking the clinical findings of aseptic meningitis and facial palsy in an endemic region to Lyme disease is crucial.
  2. Illness script/Test-taking strategy:
    • Think of Lyme disease with the triad of facial nerve palsy, meningitis symptoms, and a history of possible tick exposure in an endemic region.
    • The importance of geography: Always consider Lyme disease in patients from endemic areas presenting with neurological and musculoskeletal symptoms.
  3. Why other answers are wrong:
    • A. Measurement of serum angiotensin-converting enzyme activity: Investigates sarcoidosis, not indicated here.
    • C. Polymerase chain reaction test for cytomegalovirus: CMV can cause mononucleosis-like symptoms but not this presentation.
    • D. Serum antinuclear antibody assay: Evaluates for autoimmune conditions, not this scenario.
    • E. Serum protein electrophoresis: Used to assess plasma cell disorders such as multiple myeloma.

Test-taking tip:

On the USMLE, a combination of a clinical scenario with geographic context often points towards specific diagnoses. Always consider Lyme disease when presented with neurological symptoms in a tick-endemic area.

Reference:

  • Le, T., Bhushan, V., & Sochat, M. (2023). First Aid for the USMLE Step 1 (p. 144). McGraw-Hill Education.
  • Le, T., Bhushan, V., Ascha, M., Bhardwaj, A., Boushra, M., & Griffin, D. (2023). First Aid for the USMLE Step 2 CK, eleventh edition (11th ed.). (p. 357, 722 – 723). McGraw-Hill Education.

Question 110

Patient Information Age: 56 years Gender: F, self-identified Race/Ethnicity: mixed race (White and Honduran) Site of Care: clinic History Reason for Visit/Chief Concern: “I’m tired and I don’t feel well.” History of Present Illness: • fatigue and malaise for 3 months; tires quickly with physical activity • early satiety Past Medical History: •5-year history of hypertension •5-year history of hyperlipidemia……………

Answer: D Chronic myeloid leukemia

  1. Why the correct answer is right:
    • The patient’s presentation suggests Chronic Myeloid Leukemia (CML).
    • CML typically presents in middle-aged individuals and is associated with marked leukocytosis and the presence of all stages of granulocyte maturation in the blood.
    • For USMLE/NBME, remember that CML is a myeloproliferative disorder characterized by the Philadelphia chromosome resulting from a t(9;22) translocation.
  2. Illness script/Test-taking strategy:
    • Consider the age group and constellation of clinical and lab findings; in this case, an older adult with fatigue, splenomegaly, and leukocytosis with left shift.
    • CML often presents in the chronic phase with nonspecific symptoms like fatigue, malaise, and sometimes splenomegaly.
    • When seeing a marked leukocytosis with a left shift but without the predominance of blasts, think of CML.
  3. Why other answers are wrong:
    • A. Acute lymphocytic leukemia: Present with a higher percentage of lymphoblasts in the blood and is more common in children.
    • B. Acute myeloid leukemia: Would have a significant percentage of myeloblasts.
    • C. Chronic lymphocytic leukemia: Typically seen in older patients and would present with lymphocytosis.
    • E. Monoclonal gammopathy of undetermined significance: No evidence of plasma cell disorder exists in the given data.

Test-taking tip:

On the USMLE, consider CML as a top differential when you see a mature leukocytosis with a left shift (especially with metamyelocytes and myelocytes) combined with splenomegaly in a middle-aged or older patient.

Reference:

  • Le, T., Bhushan, V., & Sochat, M. (2023). First Aid for the USMLE Step 1 (p. 437). McGraw-Hill Education.
  • Le, T., Bhushan, V., Ascha, M., Bhardwaj, A., Boushra, M., & Griffin, D. (2023). First Aid for the USMLE Step 2 CK, eleventh edition (11th ed.). (p. 302 – 303). McGraw-Hill Education.

Question 111

1 A 1-week-old male newborn is evaluated in the hospital nursery 4 days after undergoing operative repair of a myelomeningocele. He was born at 37 weeks’ gestation via uncomplicated, spontaneous vaginal delivery to an 18-year-old primigravid woman. The defect was diagnosed prenatally by ultrasonography, and the repair was done 72 hours after delivery…………

Answer: D Neurogenic bladder

  1. Why the correct answer is right:
    • Myelomeningocele, a type of spina bifida, can lead to a loss of neuromuscular function below the lesion’s level, commonly including bladder dysfunction, known as neurogenic bladder.
    • The lack of movement in the lower extremities suggests significant spinal involvement.
    • For USMLE/NBME, always consider bladder involvement when there is evidence of lower extremity neuromuscular deficit with myelomeningocele.
  2. Illness script/Test-taking strategy:
    • Myelomeningocele frequently leads to issues like hydrocephalus (enlarged ventricles as shown in the ultrasound) and neurogenic bladder due to nerve damage.
    • When provided a neurologic deficit involving the lower extremities in the context of spina bifida, think of associated bladder dysfunction.
  3. Why other answers are wrong:
    • A. Congenital heart malformation: Not directly linked to myelomeningocele.
    • B. Cryptorchidism: While spina bifida may have associations with other anomalies, cryptorchidism is not a primary one.
    • C. Impaired folate metabolism: While maternal folate deficiency is a risk factor for neural tube defects, it is not a direct consequence.
    • E. Severe cognitive impairment: Many children with myelomeningocele have normal intelligence, though there is a risk with associated hydrocephalus.

Test-taking tip:

On the USMLE, myelomeningocele, and other neural tube defects can have various complications. Link the level of the spinal defect (e.g., lower extremity involvement) with associated sequelae (e.g., bladder dysfunction). Always consider the anatomical and physiological consequences of the lesion’s location.

Reference:

  • Le, T., Bhushan, V., & Sochat, M. (2023). First Aid for the USMLE Step 1 (p. 539). McGraw-Hill Education.
  • Le, T., Bhushan, V., Ascha, M., Bhardwaj, A., Boushra, M., & Griffin, D. (2023). First Aid for the USMLE Step 2 CK, eleventh edition (11th ed.). (p. 701, 764). McGraw-Hill Education.

Question 112

A 14-year-old boy is brought to the office because of a 6-month history of changes in behavior. Two days ago, the patient and his friends were detained by police for trespassing on abandoned property and entering an unoccupied house. No alcohol or other substances were involved. The police officer who brought the patient home reported that there was evidence indicating this had not been the first time the boys entered the property. The mother says her son was polite…………

Answer: E Normal development

  1. Why the correct answer is right:
    • During adolescence, it is normal for teens to prioritize peer relationships over family, seek independence, and test boundaries, which may sometimes lead to rule-breaking behaviors.
    • For USMLE/NBME, it is essential to differentiate between normal developmental behaviors and pathological conditions
  2. Illness script/Test-taking strategy:
    • Adolescents often undergo phases of rebellion, seeking independence and forming stronger peer bonds. If the behaviors are not extreme or endangering, they can be considered part of the normal developmental process.
    • When faced with a question about a teenager with a sudden behavior change but no clear signs of a disorder, consider the impact of peer groups and the normal struggles of adolescence.
  3. Why other answers are wrong:
    • A. Adjustment disorder: Results from identifiable stressors and causes significant distress or impairment.
    • B. Attention-deficit/hyperactivity disorder: Presents with attention issues, hyperactivity, and impulsivity across multiple settings.
    • C. Conduct disorder: Involves a repetitive and persistent pattern of violating the basic rights of others or societal norms.
    • D. Oppositional defiant disorder: Entails a pattern of angry, irritable mood with defiant behavior.

Test-taking tip:

On the USMLE, always consider the context of the presented scenario. Adolescents commonly challenge boundaries, and not every behavioral change indicates a disorder. Differentiate between normative rebellion and pathological behaviors.


Question 113

A hospital with a large intensive care unit (ICU) would like to improve communication among team members who care for patients with complex conditions. Research has shown that team communication is most effective when one clinician is designated as the team leader and assumes responsibility for directing the care…………….

Answer: A Conduct weekly structured team briefings and daily huddles

  1. Why the correct answer is right:
    • Structured team briefings and daily huddles are designed to ensure that all team members are on the same page regarding patient care and are updated regularly, fostering a common mental model.
    • These communication strategies help the team in real-time assessment, leading to quicker recognition of any deviation in patient progress and necessary care adjustments.
    • For USMLE/NBME, recognize the importance of regular and structured team communication for patient safety and quality of care in complex settings.
  2. Illness script/Test-taking strategy:
    • Maintaining a common mental model means ensuring all team members have the same understanding and expectations. It is achieved best with regular and structured communication where everyone is present.
    • In complex care settings like ICUs, prompt recognition of changes and alignment in care approach is critical, making regular group communication key.
  3. Why other answers are wrong:
    • B. Educating on protocols is important but does not address the regular sharing of patient-specific information and adjustments to care.
    • C. Individual discussions can lead to miscommunication or missed information between team members.
    • D. Consistency in the ICU team is good but does not guarantee effective daily communication about specific patient updates.
    • E. A disease-specific ICU would focus on care but still requires regular communication for specific patient updates.

Test-taking tip:

When dealing with questions about communication in healthcare settings on USMLE, always lean towards options that promote regular, transparent, and collective team interactions. Effective communication is a cornerstone of patient safety and improved outcomes.


Question 114

A 40-year-old man comes to the office during the summer because of a 3-day history of fever, diffuse muscle aches, fatigue, and sore throat. He has had no sick contacts. Medical history is unremarkable and he takes no medications. He received the influenza virus vaccine this year…………..

Answer: D HIV RNA polymerase chain reaction testing

  1. Why the correct answer is right:
    • Recent travel to a region with high HIV prevalence (like parts of Malaysia), along with fever, fatigue, sore throat, and leukopenia, suggests acute retroviral syndrome (early HIV infection).
    • A mononucleosis-like illness in the setting of a negative monospot test in a patient with risk factors raises suspicion for acute HIV.
    • For USMLE/NBME, be vigilant of acute HIV presentation when a patient has flu-like symptoms after potential exposure, including travel to high prevalence areas.
  2. Illness script/Test-taking strategy:
    • Acute retroviral syndrome resembles infectious mononucleosis but is distinguished by a negative monospot test.
    • When interpreting symptoms, consider the patient’s epidemiological background, such as recent travel.
  3. Why other answers are wrong:
    • A. Cytomegalovirus serologic testing: CMV can mimic infectious mononucleosis but is seen in transplant recipients or immunocompromised patients.
    • B. Epstein-Barr virus serologic testing: While EBV causes infectious mononucleosis, a negative monospot test makes this diagnosis less likely.
    • C. Hemagglutination inhibition assay: Used to diagnose mycoplasma pneumonia or some viral infections.
    • E. Serum hepatitis B surface antigen testing: The patient’s presentation is not typical of acute hepatitis B.

Test-taking tip:

On the USMLE, consider the epidemiological background of the patient. Traveling to a high HIV prevalence area combined with mononucleosis-like symptoms and a negative monospot test should prompt consideration of acute HIV.

Reference:

  • Le, T., Bhushan, V., & Sochat, M. (2023). First Aid for the USMLE Step 1 (p. 164). McGraw-Hill Education.
  • Le, T., Bhushan, V., Ascha, M., Bhardwaj, A., Boushra, M., & Griffin, D. (2023). First Aid for the USMLE Step 2 CK, eleventh edition (11th ed.). (p. 725 – 735). McGraw-Hill Education.

Question 115

A 70-year-old woman is admitted to the hospital because of a 1-hour history of shortness of breath. She has lung cancer and dementia, Alzheimer’s type, and has had a decline in mental status during the past month. She lives with her boyfriend of 20 years……………..

Answer: E Son

  1. Why the correct answer is right:
    • Without an advance directive or health care power of attorney, medical decision-making authority typically falls to the next of kin.
    • Adult children generally precede long-term partners in the hierarchy of surrogate decision-makers unless otherwise specified legally.
    • For USMLE/NBME, understand the usual hierarchy of surrogate decision makers: spouse > adult children > parents > adult siblings > other relatives.
  2. Illness script/Test-taking strategy:
    • Surrogate decision-making without a legal directive usually follows a legal and ethical hierarchy, with family members typically taking precedence over non-relatives.
    • Consider the patient’s ability to understand her situation; if she cannot, the next most appropriate surrogate should be identified.
  3. Why other answers are wrong:
    • A. Boyfriend: Without legal marriage or specified documentation, he is generally superseded by immediate family.
    • B. Neighbor: Not a family member, thus lower on the hierarchy.
    • C. Patient: Given her dementia and inability to understand her prognosis, she cannot make informed medical decisions.
    • D. Physician: Physicians do not have the authority to make patient decisions unless no other surrogate is available.

Test-taking tip:

On the USMLE, consider the epidemiological background of the patient. Traveling to a high HIV prevalence area combined with mononucleosis-like symptoms and a negative monospot test should prompt consideration of acute HIV.


Question 116

An 18-year-old patient comes to the emergency department because of a 2-day history of severe abdominal pain. The pain began in the middle of the abdomen after a large meal but now is localized to the right lower quadrant. The patient also has a 2-month history of intermittent diarrhea. Three weeks ago, the patient had an upper respiratory tract infection that resolved spontaneously…………

Answer: C CT scan of the abdomen

  1. Why the correct answer is right:
    • The patient’s symptoms of abdominal pain localized to the right lower quadrant after an upper respiratory infection, along with a history of intermittent diarrhea, suggest Crohn’s disease, an inflammatory bowel disease.
    • A CT scan of the abdomen is an appropriate diagnostic step for evaluating complications of Crohn’s, such as abscesses, fistulas, or bowel obstruction.
    • For USMLE/NBME, when suspecting inflammatory bowel disease with potential complications, a CT scan provides detailed imaging to evaluate the full extent of the disease.
  2. Illness script/Test-taking strategy:
    • Crohn’s disease can affect any part of the gastrointestinal tract from mouth to anus and often presents with right lower quadrant pain mimicking appendicitis.
    • Always consider underlying chronic conditions when presented with an acute symptom picture.
  3. Why other answers are wrong:
    • A. Abdominal ultrasonography: CT offers more detailed imaging of all potential complications.
    • B. Air-contrast barium enema: Less sensitive and specific for small bowel disease.
    • D. Technetium 99m scan: Useful for detecting Meckel’s diverticulum.
    • E. Upper gastrointestinal series: More useful for upper GI tract pathology.

Test-taking tip:

On the USMLE, when an atypical presentation of a common condition (like appendicitis) is suspected, consider differential diagnoses that mimic it, like Crohn’s, and choose the best diagnostic tool accordingly.

Reference:

  • Le, T., Bhushan, V., & Sochat, M. (2023). First Aid for the USMLE Step 1 (p. 389, 390). McGraw-Hill Education.
  • Le, T., Bhushan, V., Ascha, M., Bhardwaj, A., Boushra, M., & Griffin, D. (2023). First Aid for the USMLE Step 2 CK, eleventh edition (11th ed.). (p. 230 – 231, 241 – 242). McGraw-Hill Education.

Question 117

A 70-year-old woman comes to the office because of three episodes of rectal bleeding during the past 4 days. During each episode, she had the sudden urge to have a bowel movement and then passed a bright red, bloody stool. She has not had chest, abdominal, or rectal pain, palpitations, dizziness, light-headedness, or black stools. Her most recent colonoscopy 8 years ago showed no abnormalities. She has osteoarthritis treated with daily ibuprofen………….

Answer: C Diverticulosis

  1. Why the correct answer is right:
    • The patient’s episodes of painless, bright red rectal bleeding in the absence of other systemic symptoms in the setting of chronic NSAID use suggest diverticulosis.
    • Diverticulosis is common in the elderly and can present with sudden, intermittent, painless lower gastrointestinal bleeding, often selflimited.
    • For USMLE/NBME, remember that chronic NSAID use increases the risk of diverticular bleeding.
  2. Illness script/Test-taking strategy:
    • Painless, bright red blood per rectum in an elderly patient, especially with NSAID use and without alarming systemic symptoms, typically points toward diverticulosis.
    • Use age, medication history, and presentation (painless versus painful bleeding) to differentiate potential causes of lower GI bleeding.
  3. Why other answers are wrong:
    • A. Anal fissure: Presents with pain during defecation and blood.
    • B. Colonic polyp: They may bleed, and the presentation is less typical.
    • D. Duodenal ulcer: Commonly present with melena (black, tarry stools) rather than bright red blood.
    • E. Gastritis: Also more associated with melena or coffee-ground emesis.
    • F. Ulcerative colitis: Chronic inflammatory condition with persistent symptoms like diarrhea, often with mucus or pus, and is less common in the elderly.

Test-taking tip:

On the USMLE, consider the patient’s age and nature of the bleeding (painful vs. painless, bright red vs. melena) to differentiate the most likely cause of lower GI bleeding.

Reference:

  • Le, T., Bhushan, V., & Sochat, M. (2023). First Aid for the USMLE Step 1 (p. 387, 390). McGraw-Hill Education.
  • Le, T., Bhushan, V., Ascha, M., Bhardwaj, A., Boushra, M., & Griffin, D. (2023). First Aid for the USMLE Step 2 CK, eleventh edition (11th ed.). (p. 232 – 233). McGraw-Hill Education.

Question 118

A 4-month-old female infant is recovering in the hospital 5 days after uncomplicated surgical repair of a ventricular septal defect via median sterotomy utilizing cardiopulmonary bypass. She was extubated on post-operative day 1 and by postoperative day 4………..

Answer: C Echocardiography

  1. Why the correct answer is right:
    • The infant’s presentation of acute decompensation with symptoms like cold extremities, delayed capillary refill, and decreased urine output after cardiac surgery suggests a cardiac cause, most likely related to the recent surgery.
    • Echocardiography is a non-invasive, readily available, and rapid modality to evaluate cardiac function, valvular issues, and potential pericardial effusions or tamponade postoperatively.
    • For USMLE/NBME, an echocardiogram is often the first diagnostic step when suspecting a complication from cardiac surgery in a pediatric patient.
  2. Illness script/Test-taking strategy:
    • Post-operative cardiac issues in children, especially after procedures like VSD repair, can present with signs of poor perfusion and cardiac output.
    • Be prepared to rapidly evaluate cardiac function in the post-operative setting using the most direct and non-invasive means available.
  3. Why other answers are wrong:
    • A. Cardiac catheterization: Invasive and not the first choice in an acutely decompensating infant.
    • B. CT angiography: Involves radiation and potentially nephrotoxic contrast agents.
    • D. Electrophysiology study: More relevant for arrhythmias.
    • E. MRI of the heart: A longer and less accessible test, not ideal for acute decompensation.

Test-taking tip:

For acutely ill patients on USMLE, especially post-operative, choose the rapid, readily available diagnostic method that provides immediate insight into the suspected pathology. In this case, echocardiography is the best choice for a cardiac issue in a pediatric patient.


Question 119

A 42-year-old woman comes to the office because of a 2-month history of episodes of light-headedness and loss of consciousness for 10 seconds. The episodes occur two to three times weekly at work. She says her light- headedness resolves after she eats snacks……….

Answer: E No abnormalities

  1. Why the correct answer is right:
    • The patient’s hypoglycemic episodes with increased insulin and normal C-peptide levels, without an exogenous insulin administration history, suggest the factitious insulin administration.
    • A normal C-peptide with elevated insulin levels indicates that the elevated insulin is not being produced endogenously by the pancreas.
    • For USMLE/NBME, it is essential to recognize factitious disorders where patients intentionally cause illness in themselves; in this scenario, it is self-administration of insulin.
  2. Illness script/Test-taking strategy:
    • Recognize hypoglycemia as a common presentation for various issues, but match the clinical context (weight gain, frequency, and relation to food) and lab values to pinpoint the cause.
    • The discrepancy between C-peptide and insulin levels can help differentiate endogenous from exogenous insulin sources.
  3. Why other answers are wrong:
    • A. Duodenal mass with multiple liver metastases: Reflects a neuroendocrine tumor (like carcinoid), but these typically secrete serotonin, not insulin.
    • B. Fluid-filled mass between the posterior aspect of the stomach and pancreas: Suggests a pancreatic pseudocyst unrelated to insulin production.
    • C. Low-density mass in the head of the pancreas obstructing the main duct: Indicates a pancreatic adenocarcinoma, not typically associated with insulin secretion.
    • D. Vascular mass in the neck of the pancreas: This might suggest an insulinoma, but these typically produce both insulin and C-peptide.

Test-taking tip:

In USMLE questions, always connect the clinical scenario with lab values. A mismatch between what the labs suggest and the clinical picture might indicate an external factor, such as factitious disorders.

Reference:

  • Le, T., Bhushan, V., & Sochat, M. (2023). First Aid for the USMLE Step 1 (p. 585, 352). McGraw-Hill Education.
  • Le, T., Bhushan, V., Ascha, M., Bhardwaj, A., Boushra, M., & Griffin, D. (2023). First Aid for the USMLE Step 2 CK, eleventh edition (11th ed.). (p. 129 – 130, 621). McGraw-Hill Education.

Question 120

A 22-year-old man is brought to the emergency department because of severe right lower leg pain since he fell in the woods during a camping trip 2 days ago. The pain has worsened during the past 12 hours. He sustained a small scratch below his knee as a result of the fall……….

Answer: D Surgical debridement

  1. Why the correct answer is right:
    • The clinical presentation suggests necrotizing fasciitis, a rapidly progressive soft tissue infection characterized by widespread subcutaneous tissue and fascia necrosis.
    • Early surgical debridement is essential in necrotizing fasciitis to remove dead tissue and stop disease progression.
    • For USMLE/NBME, remember that clinical findings suggest necrotizing fasciitis (pain out of proportion to physical findings, bullae, rapid progression) warrants emergent surgical intervention.
  2. Illness script/Test-taking strategy:
    • Necrotizing fasciitis presents as severe pain, edema, erythema, warmth, and often systemic symptoms such as fever and hypotension.
    • Always associate rapidly advancing skin changes with potential underlying necrotizing infections.
  3. Why other answers are wrong:
    • A. Fine-needle aspiration of the bulla: In the setting of suspected necrotizing fasciitis, this is too passive and may delay life-saving intervention.
    • B. Hyperbaric oxygen therapy: Immediate surgical intervention is the priority in necrotizing fasciitis.
    • C. MRI of the right lower extremity: May delay surgical management in a clear clinical picture of necrotizing fasciitis.
    • E. Hospital admission for observation only: This is too passive for a lifethreatening infection that requires immediate surgical intervention.

Test-taking tip:

When confronted with severe clinical scenarios on the USMLE, always prioritize lifesaving interventions over diagnostic ones.

Reference:

  • Le, T., Bhushan, V., & Sochat, M. (2023). First Aid for the USMLE Step 1 (p. 134, 144, 487). McGraw-Hill Education.
  • Le, T., Bhushan, V., Ascha, M., Bhardwaj, A., Boushra, M., & Griffin, D. (2023). First Aid for the USMLE Step 2 CK, eleventh edition (11th ed.). (p. 103, 105). McGraw-Hill Education.